Auf dieser Seite werden für Sie die Antworten auf die Verständnisfragen aus dem Lehrbuch - inklusive pdfs zum Download - zusammengestellt. Damit Sie direkt hier den Testing Effect anwenden können, sind die Antworten erst sichtbar, wenn Sie den Bereich unterhalb der Frage markieren.

Sie finden die Verständnisfragen im Anschluss, sobald die Antworten zu Kapitel 18 vorliegen, auch wie gewohnt auf der Hauptseite des Buchs unter dem Reiter Verständnisfragen.

[Kleinere Abweichungen im Text zwischen Buch, Website und pdfs - insbesondere bei den Genderformen - bitten wir zu entschuldigen.]

 

Inhaltsübersicht

 

Kapitel 1 - Prolog: Die Geschichte der Psychologie

Kapitel 2 - Kritisch denken mit wissenschaftlicher Psychologie

Kapitel 3 - Neurowissenschaft und Verhalten

Kapitel 4 - Bewusstsein und der zweigleisige Verstand

Kapitel 5 - Anlage, Umwelt und die Vielfalt der Menschen

Kapitel 6 - Entwicklung über die Lebensspanne

Kapitel 7 - Wahrnehmung

Kapitel 8 - Lernen

Kapitel 9 - Gedächtnis

Kapitel 10 - Denken und Sprache

Kapitel 11 - Intelligenz

Kapitel 12 - Quellen der Motivation: Hunger, Sex, Zugehörigkeit und Erfolg

Kapitel 13 - Emotionen, Stress und Gesundheit

Kapitel 14 - Sozialpsychologie

Kapitel 15 - Persönlichkeit

Kapitel 16 - Klinische Psychologie: Psychische Störungen

Kapitel 17 - Klinische Psychologie: Therapie

Kapitel 18 - Pädagogische Psychologie

Anhang A

 

 

Kapitel 1 - Prolog: Die Geschichte der Psychologie

 

1.1 – Was zeichnet die Psychologie als Wissenschaft aus und warum „hat die Ratte immer Recht“?

Die Erkenntnisse der auf einem empirischen Ansatz beruhenden Psychologie, sind das Ergebnis sorgfältiger Beobachtung und Evaluation. Die Unterscheidung von Nutzen und Nonsens erfordert eine wissenschaftliche Einstellung. Die „Ratte“ (z. B. im Labyrinth eines Psychologen oder einer Psychologin) hat immer Recht, denn Fakten sind Fakten, auch wenn wir sie überraschend finden.

 

1.2 – Was sind die drei Hauptkomponenten der wissenschaftlichen Haltung und auf welche Weise unterstützen sie wissenschaftliches Arbeiten?

Die wissenschaftliche Haltung befähigt uns, neugierig, skeptisch und bescheiden zu sein, wenn wir miteinander konkurrierende Ideen oder eigene Beobachtungen unter die Lupe nehmen. Neugierde bringt uns auf neue Ideen, Skepsis ermutigt. Und Bescheidenheit hilft uns, Vorhersagen zu verwerfen, die durch die Forschung nicht verifiziert werden können. Gemeinsam ermöglichen diese drei Hauptkomponenten die moderne Wissenschaft.

 

1.3 – Auf welche Weise fördert kritisches Denken eine wissenschaftliche Haltung? Und wie kann es uns im Alltag klüger machen?

Kritisches Denken stellt Ideen auf den Prüfstand, indem es Annahmen hinterfragt, die Quellen beurteilt, versteckte Verzerrungen aufdeckt, Evidenz evaluiert und die Schlussfolgerungen bewertet.

 

1.4 – Was sind die wichtigen Meilensteine in der frühen Entwicklung der Psychologie?

Wilhelm Wundt gründete 1879 in Deutschland das erste psychologische Forschungslabor. Zwei frühe Denkschulen der Psychologie waren der Strukturalismus und der Funktionalismus.

 

1.5 – Wie trieben Behaviorismus, Freud’sche Psychologie und humanistische Psychologie die Entwicklung der wissenschaftlichen Psychologie voran?

Frühe Forschende definierten Psychologie als „die Wissenschaft vom Seelenleben“. In den 1920er-Jahren, unter dem Einfluss von John B. Watson und den Behaviorist:innen, änderte sich der Forschungsschwerpunkt in Richtung „wissenschaftliche Untersuchung des beobachtbaren Verhaltens“. Der Behaviorismus wurde bis in die 1960er-Jahre hinein eine der beiden wichtigsten Strömungen der Psychologie. Die zweite große Bewegung, die Freud‘sche Psychologie, und der Einfluss der humanistischen Psychologie ließen das Interesse an der Erforschung geistiger Prozesse wieder aufleben.

 

1.6 – Inwiefern konzentriert sich die moderne Psychologie auf Kognition, Biologie und Erfahrung, Kultur und Geschlechtszugehörigkeit sowie die Entfaltung des Menschen?

Die kognitive Revolution in den 1960er-Jahren brachte die Psychologie zu ihrem früheren Interesse an der Seele sowie zu ihrer heutigen Definition als Wissenschaft vom Verhalten und der mentalen Prozesse zurück. Unser wachsendes Verständnis von Biologie und Erfahrung hat die nachhaltigste Debatte der Psychologie genährt. Die Frage nature vs. nurture dreht sich um den relativen Beitrag von Genen und Erfahrungen sowie deren Interaktion in einer bestimmten Umgebung. Charles Darwins Ansicht, dass die natürliche Selektion sowohl das Verhalten als auch den Körper formt, führte zur Evolutionspsychologie, die die Gemeinsamkeiten zwischen uns aufgrund unserer gemeinsamen Biologie und Evolutionsgeschichte untersucht, sowie zur Verhaltensgenetik, die sich auf die relative Stärke und die Grenzen der genetischen und umweltbedingten Einflüsse auf das Verhalten konzentriert. Interkulturelle und geschlechtsspezifische Studien haben die Annahmen der Psychologie ausdifferenziert und uns gleichzeitig an unsere Gemeinsamkeiten erinnert. Einstellungen und Verhaltensweisen mögen sich je nach Geschlecht oder Kultur etwas unterscheiden, aber aufgrund unserer gemeinsamen Abstammung sind die zugrundeliegenden Prozesse und Prinzipien eher interindividuell ähnlich als unterschiedlich. Der traditionelle Schwerpunkt der Psychologie – Verstehen und Behandeln von Schwierigkeiten – hat sich mit der Forderung der Positiven Psychologie nach mehr Forschung zu menschlichem Wohlbefinden und dem Versuch, Eigenschaften zu entdecken und zu fördern, die dem Menschen helfen, zu wachsen, erweitert.

 

1.7 – Was sind die zentralen Analyseebenen der Psychologie und welche Sichtweisen sind damit verbunden?

Der Biopsychosoziale Ansatz integriert Informationen aus drei unterschiedlichen, aber sich ergänzenden Analyseebenen: der biologischen, der psychologischen und der sozial-kulturellen Analyseebene. Dieser Ansatz ermöglicht ein umfassenderes Verständnis, als es normalerweise möglich wäre, wenn man sich nur auf eine der aktuellen theoretischen Perspektiven der Psychologie (Neurowissenschaften, Evolution, Verhaltensgenetik, Psychodynamik, Verhaltenstherapie, kognitive und soziokulturelle Ansätze) stützen würde.

 

1.8 – Was sind die Hauptteilfelder der Psychologie?

Innerhalb der psychologischen Wissenschaft können Forschende Grundlagenforschung betreiben, um die Wissensgrundlagen des Fachgebiets zu erweitern (häufig in der Biologischen, Entwicklungs-, Kognitions-, Persönlichkeits- und Sozialpsychologie), oder angewandte Forschung, um praktische Probleme zu lösen (in der Arbeits- und Organisationspsychologie und anderen Fachgebieten). Personen, die Psychologie als helfenden Beruf ausüben, können Menschen als beratende Psycholog:innen bei der Bewältigung von Herausforderungen und Krisen (einschließlich akademischer, beruflicher und beziehungsbezogener Probleme) und bei der Verbesserung des persönlichen und sozialen Funktionierens helfen, oder als klinische Psycholog:innen Menschen mit geistigen, emotionalen und Verhaltensstörungen diagnostizieren und behandeln. (Psychiater:innen beurteilen und behandeln ebenfalls Menschen mit Störungen, aber als Ärzt:innen können sie zusätzlich zur Psychotherapie auch Medikamente verschreiben). Gemeindepsycholog:innen arbeiten an der Schaffung eines gesunden sozialen und physischen Umfelds (z. B. in Schulen).

 

1.9 – Wie können Ihnen psychologische Prinzipien dabei helfen, Ihre Lern- und Merkfähigkeit oder gar Ihr Leben allgemein zu verbessern?

Der Testing Effect zeigt, dass Lernen und Erinnern durch aktives Abrufen und nicht durch einfaches Wiederholen von bereits gelerntem Material verbessert werden. Die Lernmethode SQ3R (Survey, Question, Read, Retrieve, Review) wendet Prinzipien aus der Gedächtnisforschung an und kann Ihnen helfen, den Stoff zu lernen und zu behalten. Vier weitere Tipps sind: (1) Lernzeit optimal einteilen; (2) kritisches Denken lernen; (3) Informationen aus dem Lernstoff aktiv verarbeiten; und (4) nicht zu viel lernen. Die psychologische Forschung hat gezeigt, dass Menschen, die ein glückliches und erfolgreiches Leben führen, ihre Zeit so einteilen, dass sie ausreichend Schlaf bekommen, sich ausreichend bewegen, sich langfristige Ziele setzen, die sie täglich erreichen wollen, eine wachstumsorientierte Denkweise haben und Beziehungen Priorität einräumen.

 

zurück zur Inhaltsübersicht

 

 

Kapitel 2 - Kritisch denken mit wissenschaftlicher Psychologie

 

2.1 – Auf welche Weise führt uns unser alltägliches Denken manchmal zu falschen Schlussfolgerungen?

Unser Alltagsdenken kann uns aufgrund von drei Phänomenen in die Irre führen. Die Rückschauverzerrung (das „Ich habe es schon immer gewusst“-Phänomen) ist die Tendenz zu glauben, dass wir, nachdem wir ein Ergebnis erfahren haben, es auch vorhergesehen hätten. Übermäßiges Selbstvertrauen ist oft das Ergebnis unserer Bereitschaft, selbstbewusster zu sein als es der Fall ist. Diese Tendenzen und unser Bestreben, Muster in zufälligen Ereignissen zu erkennen, führen dazu, dass wir das Potential des gesunden Menschenverstands überschätzen. Die wissenschaftliche Forschung kann uns helfen, solche Vorurteile und Defizite zu überwinden, auch wenn sie durch die überprüfbaren Fragen, die sie behandeln kann, begrenzt ist.

 

2.2 – Wie bringen Theorien die Psychologie als Wissenschaft voran?

Psychologische Theorien verwenden einen integrierenden Satz von Prinzipien, um Beobachtungen zu organisieren und Hypothesen aufzustellen. Durch die Überprüfung ihrer Hypothesen können Forschende ihre Theorien bekräftigen, verwerfen oder revidieren. Um anderen Forschenden die Möglichkeit zu geben, die Studien zu replizieren, berichten die Forschenden über ihre Studien unter Verwendung präziser operativer Definitionen ihrer Verfahren und Konzepte. Wenn andere Forschende ähnliche Ergebnisse erzielen, ist das Vertrauen in die Schlussfolgerung größer.

 

2.3 – Wie nutzen Psycholog:innen Einzelfallstudien,  Feldbeobachtungen und Umfragen, um Verhalten zu beobachten  und zu beschreiben, und weshalb ist es wichtig,  Zufallsstichproben zu ziehen?

Deskriptive Methoden, zu denen Fallstudien, natürliche Beobachtungen und Erhebungen gehören, zeigen uns, was vorkommen kann, und sie können Ideen für weitere Untersuchungen liefern. Die beste Grundlage für Generalisierungen über eine Population ist eine repräsentative Stichprobe; bei einer Zufallsstichprobe hat jede Person in der gesamten untersuchten Population die gleiche Chance zur Aufnahme in die Sticprobe. Deskriptive Methoden beschreiben Verhalten, erklären es aber nicht; sie können Ursache und Wirkung nicht aufzeigen, da die Forschenden die Variablen nicht kontrollieren können.

 

2.4 – Was bedeutet es, wenn zwei Merkmale miteinander korrelieren, und was sind positive und negative Korrelationen?

Eine Korrelation gibt an, inwieweit zwei Variablen miteinander zusammenhängen und wie gut die eine die andere vorhersagt. Bei einer positiven Korrelation nehmen zwei Variablen gemeinsam zu oder ab; bei einer negativen Korrelation nimmt die Ausprägung einer Variablen zu, wenn die andere abnimmt. Die Stärke und Richtung der Beziehung wird durch einen Korrelationskoeffizienten ausgedrückt, der von +1,00 (perfekte positive Korrelation) über 0 (keine Korrelation) bis -1,00 (perfekte negative Korrelation) reicht. Die Korrelation kann in einem Streudiagramm dargestellt werden, in dem jeder Punkt einen Wert für die beiden Variablen darstellt.

 

2.5 – Was sind illusorische Korrelationen und was ist eine Regression zur Mitte?

Scheinkorrelationen sind zufällige Ereignisse, die wir wahrnehmen und von denen wir fälschlicherweise annehmen, dass sie zusammenhängen. Die Regression zum Mittelwert ist die Tendenz extremer oder ungewöhnlicher Werte, sich ihrem Mittelwert anzunähern (Regression).

 

2.6 – Warum erlauben Korrelationen Vorhersagen, aber keine kausalen Erklärungen?

Korrelationen ermöglichen Vorhersagen, weil sie aufzeigen, wie zwei Faktoren miteinander in Beziehung stehen - entweder positiv oder negativ. Eine Korrelation kann auf die Möglichkeit einer Ursache-Wirkungs-Beziehung hinweisen, aber sie zeigt nicht die Richtung des Einflusses oder ob ein zugrunde liegender dritter Faktor die Korrelation erklären kann.

 

2.7 – Welches sind die Eigenschaften von Experimenten, die es möglich machen, Ursache und Wirkung auseinanderzuhalten?

Um Ursache-Wirkungs-Beziehungen zu entdecken, führen Psycholog:innen Experimente durch, bei denen sie eine oder mehrere interessierende Variablen manipulieren und andere Variablen kontrollieren. Durch die zufällige Zuweisung können sie Störvariablen minimieren, z. B. bereits bestehende Unterschiede zwischen der Versuchsgruppe (die der Behandlung ausgesetzt ist) und der Kontrollgruppe (die die Behandlung nicht erhält). Die unabhängige Variable ist der Faktor, den der oder Forschende manipuliert, um dessen Wirkung zu untersuchen; die abhängige Variable ist der Faktor, den der oder die Forschende misst, um etwaige Veränderungen als Reaktion auf die Manipulation der unabhängigen Variablen festzustellen. Bei Studien kann ein Doppelblindverfahren angewandt werden, um den Placebo-Effekt und die Voreingenommenheit der Forschenden zu vermeiden.

 

2.8 – Was gilt es bei der Wahl des Forschungsdesigns zu bedenken?

Psychologische Wissenschaftler:innen entwerfen Studien und wählen Forschungsmethoden, die am geeignetsten sind, um aussagekräftige Ergebnisse zu erzielen. Die Forschenden entwickeln überprüfbare Fragstellungen und überlegen dann sorgfältig, welches Forschungsdesign für die Untersuchung dieser Fragen am besten geeignet ist (experimentell, korrelativ, Fallstudie, naturalistische Beobachtung, Zwillingsstudie, Längsschnitt- oder Querschnittsstudie). Als Nächstes messen die Psycholog:innen die zu untersuchenden Variablen, und schließlich interpretieren sie ihre Ergebnisse, wobei sie mögliche Störvariablen im Blick behalten. (Die Online-Forschungsaktivitäten von Immersive Learning „How Would You Know?“ ermöglichen es Ihnen, in die Rolle des oder der Forschenden zu schlüpfen und zu entscheiden, wie sich interessante Fragen am besten untersuchen lassen).

 

2.9 – Können vereinfachte Laborversuche etwas über den Alltag aussagen?

Forschende schaffen gezielt eine kontrollierte, künstliche Umgebung im Labor, um allgemeine theoretische Grundsätze zu testen. Es sind die allgemeinen Grundsätze – nicht die spezifischen Ergebnisse –, die helfen, alltägliche Verhaltensweisen zu erklären.

 

2.10 – Warum führen Psycholog:innen Experimente mit  Tieren durch und welche ethischen Richtlinien schützen  menschliche und tierische Versuchsteilnehmende? Auf welche Weise beeinflussen die Wertvorstellungen von Psycholog:innen die Psychologie?

Einige Psycholog:innen interessieren sich in erster Linie für das Verhalten von Tieren; andere wollen die physiologischen und psychologischen Prozesse besser verstehen, die Menschen und andere Lebewesen gemeinsam haben. Behörden haben Standards für die Haltung und Unterbringung von Tieren festgelegt. Auch Berufsverbände und Kostenträger:innen haben Richtlinien zum Schutz des Wohlergehens von Tieren aufgestellt. Der APA-Ethikkodex umfasst Standards für den Schutz des Wohlergehens menschlicher Studienteilnehmender, einschließlich der Einholung ihrer informierten Zustimmung und der späteren Nachbesprechung mit ihnen. Die Werte von Psycholog:innen beeinflussen die Wahl ihrer Forschungsthemen, ihre Theorien und Beobachtungen, ihre Verhaltensbezeichnungen und ihre professionelle Beratung. Die Prinzipien der Psychologie werden vor allem im Dienst der Menschheit angewandt.

 

2.11 – Wie können wir Daten mit drei Maßen der zentralen Tendenz beschreiben und was ist der relative Nutzen der beiden Maße der Variabilität?

Forschende verwenden deskriptive Statistiken, um die Merkmale der untersuchten Gruppen zu messen und zu beschreiben. Ein Maß für die zentrale Tendenz ist ein einzelner Wert, der eine ganze Reihe von Werten repräsentiert. Drei solcher Maße, die wir zur Beschreibung von Daten verwenden, sind der Modalwert (der am Häufigsten vorkommende Wert), der Mittelwert (das arithmetische Mittel) und der Median (der mittlere Wert in einer Gruppe von Daten). Maße der zentralen Tendenz fassen die Daten übersichtlich zusammen; Variationsmaße geben Aufschluss darüber, wie unterschiedlich die Daten sind. Zwei Variationsmaße sind die Spanne (die den Abstand zwischen dem höchsten und dem niedrigsten Wert beschreibt) und die Standardabweichung (die angibt, wie stark die Werte um den Mittelwert oder Durchschnittswert schwanken). Die Werte bilden oft eine normalverteilte (oder glockenförmige) Kurve.

 

2.12 – Wie können wir herausfinden, ob sich ein beobachteter Unterschied auf eine andere Population übertragen lässt?

Forschende verwenden die Inferenzstatistik (die Verfahren zur Bestimmung der Zuverlässigkeit und Signifikanz eines beobachteten Unterschieds zwischen den Ergebnissen verschiedener Gruppen umfasst), um festzustellen, ob die Ergebnisse auf eine größere Population verallgemeinert werden können. Zuverlässige Unterschiede beruhen auf Stichproben, die für die untersuchte Grundgesamtheit repräsentativ sind, die im Durchschnitt eine geringe Variabilität aufweisen und die aus vielen Einzelfällen bestehen. Man kann sagen, dass ein beobachteter Unterschied statistisch signifikant ist, wenn die Durchschnittswerte der Stichproben zuverlässig sind und der Unterschied zwischen ihnen groß ist.

 

zurück zur Inhaltsübersicht

 

 

Kapitel 3 - Neurowissenschaft und Verhalten

 

3.1 – Wieso beschäftigen sich Psycholog:innen mit der menschlichen Biologie?

Psycholog:innen, die eine biologische Perspektive einnehmen, untersuchen die Zusammenhänge zwischen der Biologie und dem Verhalten. Der Mensch ist ein biopsychosoziales System, in dem biologische, psychologische und sozial-kulturelle Faktoren zusammenwirken und das Verhalten beeinflussen. 

 

3.2 – Wie spielen Biologie und Erfahrung zusammen?

Plastizität ermöglicht es unserem Gehirn, sich an neue Erfahrungen anzupassen, wobei es sowohl von den Genen als auch vom Lebensumfeld geformt wird. Diese Fähigkeit ist zwar lebenslang vorhanden, aber in der Kindheit ist die Plastizität am größten. Durch Erfahrung entwickelt unser Gehirn einzigartige Muster, die unsere Lebenserfahrungen widerspiegeln.

 

3.3 – Was sind Neurone und wie übermitteln sie Informationen?

Neurone sind die elementaren Bestandteile des Nervensystems, des schnellen elektrochemischen Informationssystems des Körpers. Ein aus einem Zellkörper und seinen verzweigten Fasern bestehendes Neuron empfängt Signale über seine oft buschartig verzweigten Dendriten und sendet Signale über seine Axone. Einige Axone sind von einer Myelinschicht umhüllt, die eine schnellere Übertragung ermöglicht. Gliazellen unterstützen, ernähren und schützen die Neuronen und spielen auch eine Rolle beim Lernen, Denken und Gedächtnis. Wenn die von einem Neuron empfangenen Signale eine Mindestschwelle überschreiten, wird das Neuron aktiv und sendet einen elektrischen Impuls (das Aktionspotenzial) durch einen chemischen Prozess entlang seines Axons. Die Reaktion des Neurons ist ein Alles-oder-Nichts-Prozess.

 

3.4 – Wie kommunizieren Nervenzellen miteinander?

Wenn Aktionspotenziale das Ende eines Axons (die Axonendigung) erreichen, stimulieren sie die Freisetzung von Neurotransmittern. Diese chemischen Botenstoffe übertragen eine Nachricht vom aussendenden Neuron über eine Synapse zu Rezeptorstellen auf einem empfangenden Neuron. Das sendende Neuron nimmt anschließend normalerweise die überschüssigen Neurotransmittermoleküle aus dem synaptischen Spalt wieder auf, was als „Reuptake“ bezeichnet wird. Wenn die eingehenden Signale stark genug sind, erzeugt das empfangende Neuron sein eigenes Aktionspotenzial und leitet die Nachricht an andere Zellen weiter.

 

3.5 – Wie beeinflussen Neurotransmitter das Verhalten und welche Auswirkungen haben Drogen und andere chemische Stoffe auf die neuronale Übertragung?

Neurotransmitter durchlaufen bestimmte Bahnen im Gehirn und können bestimmte Verhaltensweisen und Gefühle beeinflussen. Acetylcholin (ACh) ermöglicht Muskelbewegungen, Lernen und Erinnerungsvermögen. Endorphine sind natürliche Opiate, die als Reaktion auf Schmerz und Bewegung freigesetzt werden. Drogen und andere chemische Substanzen beeinflussen die Gehirnchemie an den Synapsen. Agonisten verstärken die Wirkung eines Neurotransmitters und können dies auf verschiedene Weise tun. Antagonisten verringern die Wirkung eines Neurotransmitters, indem sie die Produktion oder Freisetzung blockieren.

 

3.6 – Welche Funktionen haben die Hauptkomponenten des Nervensystems und wie heißen die drei Hauptgruppen von Neuronen?

Das zentrale Nervensystem (ZNS) – das Gehirn und das Rückenmark – ist die Entscheidungsinstanz des Nervensystems. Das periphere Nervensystem (PNS), welches das ZNS über Nerven mit dem übrigen Körper verbindet, sammelt Informationen und leitet die Informationen des ZNS an den übrigen Körper weiter. Die beiden wichtigsten Abteilungen des PNS sind das somatische Nervensystem (das die willkürliche Steuerung der Skelettmuskulatur ermöglicht) und das autonome Nervensystem (das die unwillkürlichen Muskeln und Drüsen mit Hilfe seiner sympathischen und parasympathischen Teilsysteme steuert). Die drei Arten von Neuronen sind in Arbeitsnetzwerken zusammengefasst: (1) Sensorische (afferente) Neuronen leiten die von den Körpergeweben und Sinnesrezeptoren eingehenden Informationen an das Gehirn und das Rückenmark weiter. (2) Motorische (efferente) Neuronen leiten ausgehende Informationen vom Gehirn und Rückenmark zu den Muskeln und Drüsen. (3) Interneuronen kommunizieren innerhalb des Gehirns und des Rückenmarks und verarbeiten Informationen zwischen den sensorischen Eingängen und den motorischen Ausgängen.

 

3.7 – Wie übermittelt das endokrine System Informationen und welche Wechselwirkung besteht mit dem Nervensystem?

Das endokrine System sondert Hormone in den Blutkreislauf ab, von wo aus sie in den Körper gelangen und andere Gewebe, einschließlich des Gehirns, beeinflussen. Die Hauptdrüse des endokrinen Systems, die Hypophyse, beeinflusst die Hormonausschüttung anderer Drüsen, einschließlich der Nebennieren. In einem komplizierten Rückkopplungssystem beeinflusst der Hypothalamus des Gehirns die Hypophyse, die wiederum andere Drüsen beeinflusst, welche ihrerseits Hormone freisetzen, die dann erneut das Gehirn beeinflussen.

 

3.8 – Wie untersuchen Neurowissenschaftler:innen die Verbindungen vom Gehirn zum Verhalten und zum Verstand bzw. Bewusstsein?

Klinische Beobachtungen und Läsionen geben Aufschluss über die allgemeinen Auswirkungen von Hirnschäden. Elektrische, chemische oder magnetische Stimulation kann auch Aufschluss über die Informationsverarbeitung im Gehirn geben. MRT-Scans zeigen die Gehirnanatomie. EEG-, MEG-, PET- und fMRI-Aufnahmen (funktionelle MRT) geben Aufschluss über die Gehirnfunktion.

 

3.9 – Aus welchen Strukturen besteht der Hirnstamm und welche Funktionen haben Hirnstamm, Thalamus, Formatio reticularis und Kleinhirn?

Der Hirnstamm, der älteste Teil des Gehirns, ist für die autonomen Überlebensfunktionen zuständig. Seine Bestandteile sind das Medulla (steuert den Herzschlag und die Atmung), Pons (hilft, Bewegungen zu koordinieren und den Schlaf zu kontrollieren) und Formatio reticularis (filtert eingehende Reize, leitet Informationen an andere Gehirnbereiche weiter beeinflusst die Erregung). Der Thalamus, der oberhalb des Hirnstamms sitzt, fungiert als sensorisches Kontrollzentrum des Gehirns. Das Cerebellum, das an der Rückseite des Hirnstamms sitzt, koordiniert Muskelbewegungen sowie Gleichgewicht und ermöglicht nonverbales Lernen und Gedächtnisleistungen.

 

3.10 – Was sind die Strukturen und Funktionen des limbischen Systems?

Das limbische System ist mit Emotionen, dem Gedächtnis und den Trieben verbunden. Zu seinen neuronalen Zentren gehören die Amygdala (beteiligt an Aggressions- und Angstreaktionen), der Hypothalamus (steuert verschiedene Körperfunktionen, hilft bei der Steuerung des endokrinen Systems und ist mit Emotionen und Belohnung verbunden) und der Hippocampus (hilft bei der Verarbeitung expliziter, bewusster Erinnerungen). Der Hypothalamus steuert die Hypophyse (die „Hauptdrüse“), indem er sie zur Ausschüttung von Hormonen anregt.

 

3.11 – Welche vier Lappen bilden den zerebralen Kortex und was sind die Funktionen des motorischen Kortex, des somatosensorischen Kortex und der Assoziationsfelder?

Der zerebrale Kortex besteht aus zwei Hemisphären, und jede Hemisphäre hat vier Hirnlappen: den Frontal-, Parietal-, Okzipital- und Temporallappen. Jeder dieser Lappen erfüllt zahlreiche Funktionen und interagiert mit anderen Bereichen des Kortex. Der motorische Kortex, der sich im hinteren Teil der Frontallappen befindet, steuert willkürliche Bewegungen. Der somatosensorische Kortex an der Vorderseite der Scheitellappen registriert und verarbeitet Berührungs- und Bewegungsempfindungen des Körpers. Die Körperteile, die eine präzise Steuerung erfordern (im motorischen Kortex) oder die besonders empfindlich sind (im somatosensorischen Kortex), nehmen den größten Raum ein. Der größte Teil der Hirnrinde - der größte Teil jedes der vier Hirnlappen - ist den sogenannten Assoziationsarealen gewidmet, die Informationen integrieren, die an höheren geistigen Funktionen wie Lernen, Erinnern, Denken und Sprechen beteiligt sind. Unsere mentalen Erfahrungen entstehen durch koordinierte Gehirnaktivitäten.

 

3.12 – Benutzen wir wirklich nur 10 Prozent unseres Gehirns?

Die Tatsache, dass die Assoziationsbereiche nicht auf elektrische Impulse reagieren, hat zu der falschen Behauptung geführt, dass wir nur 10 Prozent unseres Gehirns nutzen. Diese umfangreichen Areale des Gehirns sind jedoch dafür verantwortlich, sensorische Informationen zu interpretieren, zu integrieren und zu verarbeiten und sie mit gespeicherten Erinnerungen zu verknüpfen. Hirnschädigungen zeigen, dass die Neuronen in den Assoziationsbereichen mit höheren geistigen Funktionen beschäftigt sind.

 

3.13 – In welchem Ausmaß kann sich ein geschädigtes Gehirn neu organisieren? Was ist Neurogenese?

Zwar regenerieren sich die Neurone des Gehirns und des Rückenmarks in der Regel nicht, doch kann sich ein Teil des neuronalen Gewebes als Reaktion auf eine Schädigung umstrukturieren. Das geschädigte Gehirn kann sich plastisch verändern, vor allem bei Kleinkindern, indem neue Signalwege gebildet werden und Funktionen in andere Hirnregionen verlagert werden. Auch bei Blindheit und Taubheit oder als Folge von Schäden und Krankheiten kann es zu einer Neuzuordnung von Funktionen zu verschiedenen Bereichen des Gehirns kommen. Das Gehirn heilt sich manchmal selbst, indem es neue Neuronen bildet, ein Prozess, der als Neurogenese bekannt ist.

 

3.14 – Wie trägt die Forschung zur Trennung der Hemisphären zu einem besseren Verständnis der Funktionen unserer rechten und linken Hirnhälfte bei?

Die Split-Brain-Forschung (Untersuchungen an Menschen mit einem durchtrennten Corpus callosum) hat bestätigt, dass bei den meisten Menschen die linke Hemisphäre die verbalere ist. Die rechte Hemisphäre zeichnet sich durch visuelle Wahrnehmung, Emotionswahrnehmung und Schlussfolgerung aus und hilft uns, unsere Sprache zu modulieren und unsere Selbstwahrnehmung zu koordinieren. Studien an gesunden Menschen mit intakten Gehirnen bestätigen, dass jede Hemisphäre einen einzigartigen Beitrag zur integrierten Funktionsweise des Gehirns leistet.

 

 

zurück zur Inhaltsübersicht

 

Kapitel 4 - Bewusstsein und der zweigleisige Verstand

 

4.1 – Wo lässt sich das Bewusstsein in der Geschichte der Psychologie verorten?

Nachdem die Psychologie im 19. Jahrhundert zunächst das Bewusstsein als Forschungsgebiet für sich beansprucht hatte, gab sie es in der ersten Hälfte des 20. Jahrhunderts auf und wandte sich stattdessen der Erforschung des beobachtbaren Verhaltens zu, weil die Forschenden glaubten, dass das Bewusstsein wissenschaftlich zu schwer zu untersuchen sei. Seit 1960, unter dem Einfluss der kognitiven Psychologie, der Neurowissenschaft und der kognitiven Neurowissenschaft, hat unser bewusstes Erleben von uns selbst und unserer Umwelt - unser Bewusstsein - seinen Platz als wichtiges Forschungsgebiet zurückerobert.

 

4.2 – Wie lenkt selektive Aufmerksamkeit unsere Wahrnehmung?

Wir nehmen selektiv einen sehr begrenzten Teil der eingehenden Informationen wahr und verarbeiten diese, wobei wir viele Informationen ausblenden und oft den Schwerpunkt unserer Aufmerksamkeit von einer Sache auf eine andere verlagern. Wenn wir uns intensiv auf eine Aufgabe konzentrieren, sind wir oft unaufmerksam gegenüber anderen Ereignissen, einschließlich der Blindheit gegenüber Veränderungen in unserer Umgebung.

 

4.3 – Was versteht man in der Kognitiven Neurowissenschaft unter Parallelverarbeitung?

Wissenschaftler:innen, die sich mit den Gehirnmechanismen befassen, die dem Bewusstsein und der Kognition zugrunde liegen, haben herausgefunden, dass der Verstand Informationen auf zwei getrennten Wegen verarbeitet: auf einer bewussten Schiene (sequentielle Verarbeitung) und auf einer unbewussten Schiene (parallele Verarbeitung). Die parallele Verarbeitung kümmert sich um Routineaufgaben, während die sequentielle Verarbeitung am besten geeignet ist, um neue Probleme zu lösen, die unsere Aufmerksamkeit erfordern. Diese zweifache Verarbeitung - bewusst und unbewusst - wirkt sich auf unsere Wahrnehmung, unser Gedächtnis, unsere Einstellungen und andere Erkenntnisse aus.

 

4.4 – Was ist Schlaf?

Schlaf ist der wiederkehrende, natürliche Verlust des Bewusstseins - im Unterschied zur Bewusstlosigkeit, die aus einem Koma, einer Vollnarkose oder einem Winterschlaf resultiert. (Adaptiert nach Dement, 1999.)

 

4.5 – Wie beeinflussen unsere biologischen Rhythmen unser alltägliches Leben?

Unser Körper verfügt über eine innere biologische Uhr, die ungefähr mit dem 24-Stunden-Zyklus von Tag und Nacht synchronisiert ist. Dieser zirkadiane Rhythmus zeigt sich in unseren täglichen Zyklen von Körpertemperatur, Erregung, Schlaf und Wachsein. Alter und Erfahrung können diese Rhythmen verändern und unsere biologische Uhr zurücksetzen.

 

4.6 – Wie sieht der biologische Rhythmus unserer Schlaf- und Traumstadien aus?

Jüngere Erwachsene durchlaufen etwa alle 90 Minuten vier verschiedene Schlafstadien (bei älteren Erwachsenen wiederholt sich der Schlafzyklus häufiger). Nach den Alphawellen des wachen, entspannten Stadiums fallen wir in die unregelmäßigen Gehirnwellen des Non-REM-Stadiums 1 (NREM-1), in dem es häufig zu Halluzinationen kommt. Es folgt der NREM-2-Schlaf (in dem wir etwa die Hälfte unserer Schlafzeit verbringen), der etwa 20 Minuten dauert und dessen charakteristische Schlafwellen zu beobachten sind. Danach treten wir in den NREM-3-Schlaf ein, der etwa 30 Minuten dauert und große, langsame Deltawellen aufweist. Etwa eine Stunde nach dem Einschlafen tauchen wir aus unserem anfänglichen Tiefschlaf auf und beginnen mit Phasen des REM-Schlafs (Rapid Eye Movement). Der REM-Schlaf wird als paradoxes Schlafstadium beschrieben, das durch innere Erregung, aber äußere Ruhe (nahezu Lähmung) gekennzeichnet ist. In dieser Phase wird am meisten geträumt. Diese Phase verlängert sich im Laufe der Nacht. Während einer normalen Nacht verkürzt sich der NREM-3-Schlaf, und es verlängern sich der REM- und NREM-2-Schlaf.

 

4.7 – Wie wirken sich Biologie und Umfeld auf unsere Schlafmuster aus?

Unsere Biologie – unser zirkadianer Rhythmus sowie unser Alter und die körpereigene Melatoninproduktion (die vom Nucleus suprachiasmaticus des Gehirns beeinflusst wird) – steht in Wechselwirkung mit kulturellen Erwartungen und individuellen Verhaltensweisen, um den Schlaf- und Wachrhythmus zu bestimmen. Lichteinfall (oder Lichtmangel) stört unsere biologische 24-Stunden-Uhr; Nachtschichtarbeitende können unter chronischer Desynchronisation leiden.

 

4.8 – Welche Funktionen hat der Schlaf?

Der Schlaf könnte in der menschlichen Evolution eine schützende Rolle gespielt haben, indem er die Menschen in potenziell gefährlichen Zeiten in Sicherheit hielt. Schlaf hilft auch bei der Wiederherstellung und Reparatur beschädigter Neurone. Schlaf festigt unsere Erinnerungen, indem er das kürzlich Gelernte wiederholt und die neuronalen Verbindungen stärkt. Schlaf fördert das kreative Problemlösen am nächsten Tag. Während des Slow-Wave-Schlafs schüttet die Hypophyse ein Wachstumshormon aus, das für den Muskelaufbau notwendig ist.

 

4.9 – Welche Auswirkungen hat Schlafmangel und was sind die wesentlichen Schlafstörungen?

Schlafmangel führt zu Müdigkeit und Reizbarkeit und beeinträchtigt die Konzentration, Produktivität und Gedächtniskonsolidierung. Schlafentzug kann auch zu Depressionen, Fettleibigkeit, Gelenkschmerzen, einem geschwächten Immunsystem und verminderter Leistungsfähigkeit (mit erhöhter Unfallgefahr) führen. Zu den Schlafstörungen gehören Schlaflosigkeit (wiederkehrende Wachzustände), Narkolepsie (plötzliche, unkontrollierbare Schläfrigkeit, die manchmal direkt in den REM-Schlaf übergeht), Schlafapnoe (wiederholte Atemstillstände im Schlaf; vor allem bei Männern mit Fettleibigkeit verbunden), Nachtangst (hohe Erregung und der Anschein von Angst; NREM-3-Störung, die vor allem bei Kindern auftritt), Schlafwandeln (NREM-3-Störung, die ebenfalls vor allem bei Kindern auftritt) und Reden im Schlaf.

 

4.10 – Wovon träumen wir? Was sind Theoretiker:innen zufolge die Aufgaben von Träumen?

Normalerweise träumen wir von gewöhnlichen Ereignissen und Alltagserfahrungen, die meist mit Ängsten oder Unglück verbunden sind. Weniger als 10 Prozent der Träume von Männern (und noch weniger von Frauen) haben einen sexuellen Inhalt. Die meisten Träume treten während des REM-Schlafs auf; die Träume, die während des NREM-Schlafs auftreten, sind meist vage, flüchtige Bilder. Es gibt fünf Hauptauffassungen über die Funktion von Träumen. (1) Freuds Wunscherfüllung: Träume stellen ein psychisches „Sicherheitsventil“ dar, wobei der manifeste Inhalt (die Handlung) als verschlüsselte Version des latenten Inhalts (die zugrunde liegende Bedeutung, die unsere unbewussten Wünsche befriedigt) fungiert. (2) Informationsverarbeitung: Träume helfen uns, die Ereignisse des Tages zu sortieren und im Gedächtnis zu festigen. (3) Physiologische Funktion: Regelmäßige Hirnstimulation kann dazu beitragen, die neuronalen Bahnen im Gehirn zu entwickeln und zu erhalten. (4) Neuronale Aktivierung: Das Gehirn versucht, die neuronale Statik zu verstehen, indem es sie zu einer Geschichte verwebt. (5) Kognitive Entwicklung: Träume spiegeln den Entwicklungsstand des bzw. der Träumenden wider – sein bzw. ihr Wissen und Verständnis. Die meisten Schlaftheoretiker:innen sind sich einig, dass der REM-Schlaf und die damit verbundenen Träume eine wichtige Funktion erfüllen, wie der REM-Rebound zeigt, der nach REM-Entzug bei Menschen und anderen Spezies auftritt.

 

4.11 – Was versteht man unter Substanzkonsumstörungen?

Menschen mit einer Substanzkonsumstörung haben ein anhaltendes Verlangen nach Substanzen und konsumieren diese trotz erheblicher Störungen im Alltag und bzw. oder körperlicher Risiken. Psychoaktive Drogen verändern Wahrnehmungen und Stimmungen.

 

4.12 – Welche Rolle spielen Toleranz und Sucht bei Substanzkonsumstörungen und wie hat sich der Begriff „Sucht“ verändert?

Psychoaktive Substanzen können zu einer Toleranz führen, d. h., es werden höhere Dosen benötigt, um die gewünschte Wirkung zu erzielen, und es kann zu Entzugserscheinungen kommen, d. h. zu erheblichen Beschwerden, die auf ein starkes Verlangen nach der Droge zurückzuführen sind und mit dem Versuch einhergehen, mit dem Konsum aufzuhören. Die Sucht führt dazu, dass sich die Konsumierenden nach der Droge sehnen und den Konsum trotz bekannter negativer Folgen fortsetzen. Eine Therapie oder Gruppenunterstützung können helfen; es hilft zu erkennen, dass Süchte kontrollierbar sind und dass Menschen sich ändern können. Obwohl Psycholog:innen sich bemühen, den Begriff „Sucht“ nicht übermäßig zu verwenden, um getriebene, exzessive Verhaltensweisen zu bezeichnen, gibt es einige Verhaltenssüchte (wie z. B. die Spielsucht), bei denen das Verhalten zwanghaft und dysfunktional wird.

 

4.13 – Was sind dämpfende Substanzen und welche Auswirkungen haben sie?

Beruhigungsmittel wie Alkohol, Barbiturate und Opiate verringern die neuronale Aktivität und verlangsamen die Körperfunktionen. Alkohol enthemmt und erhöht die Wahrscheinlichkeit, dass wir unseren Impulsen nachgehen, egal ob sie schädlich oder hilfreich sind. Alkohol beeinträchtigt auch das Urteilsvermögen, indem er die neuronale Verarbeitung verlangsamt, Gedächtnisprozesse durch Unterdrückung des REM-Schlafs stört und das Selbstbewusstsein und die Selbstkontrolle verringert. Die Erwartungen der Konsumierenden beeinflussen die Auswirkungen des Alkohols auf das Verhalten stark. Bei Menschen mit Alkoholmissbrauchsstörung kann Alkohol das Gehirn schrumpfen lassen (gekennzeichnet durch Toleranz, Entzugserscheinungen, wenn der Konsum eingestellt wird, und den Drang, den problembehafteten Konsum fortzusetzen).

 

4.14 – Was sind Stimulanzien und welche Auswirkungen haben sie?

Stimulanzien – darunter Koffein, Nikotin, Kokain, Amphetamine, Methamphetamin und Ecstasy – regen die Nervenaktivität an und beschleunigen die Körperfunktionen, was zu Energie- und Stimmungsschwankungen führt. Alle diese Substanzen haben ein hohes Suchtpotenzial. Die Wirkung von Nikotin macht es schwierig, vom Tabakkonsum loszukommen, doch wiederholte Versuche, mit dem Rauchen aufzuhören, scheinen sich zu lohnen. Kokain verschafft den Konsumierenden einen schnellen Rausch, auf den kurz darauf ein Absturz folgt. Zu den Risiken gehören Herz-Kreislauf-Belastungen und Argwohn. Amphetamine stimulieren die neuronale Aktivität, was zu erhöhter Energie und Stimmung führt. Methamphetaminkonsum kann die Dopaminproduktion dauerhaft verringern. Ecstasy (MDMA) ist ein Kombinationspräparat aus Stimulans und mildem Halluzinogen, das Euphorie und Gefühle der Intimität hervorruft. Seine Konsumierenden riskieren eine Unterdrückung des Immunsystems, dauerhafte Beeinträchtigungen der Stimmung und des Gedächtnisses sowie (bei Einnahme während körperlicher Aktivität) Dehydrierung und steigende Körpertemperaturen.

 

4.15 – Was sind Halluzinogene und welche Auswirkungen haben sie?

Halluzinogene wie LSD und Marihuana verzerren die Wahrnehmung und lösen Halluzinationen aus. Die Wirkung von LSD hängt von der Stimmung und den Erwartungen der Konsumierenden ab. Häufige Erfahrungen sind jedoch Halluzinationen und Gefühle, die von Euphorie bis Panik reichen. Der Hauptbestandteil von Marihuana, THC, kann Gefühle der Enthemmung, Euphorie, Entspannung, Schmerzlinderung und eine hohe Reizempfindlichkeit auslösen, kann aber auch das Risiko von psychischen Störungen erhöhen und zu Lern- und Gedächtnisstörungen führen.

 

4.16 – Warum werden manche Menschen zu regelmäßigen Konsument:innen von bewusstseinsverändernden Drogen?

Manche Menschen sind möglicherweise biologisch vulnerabel für bestimmte Drogen. Psychische Faktoren (wie Stress, Depressionen und Hoffnungslosigkeit) und soziale Faktoren (wie Gruppendruck) führen dazu, dass viele Menschen mit Drogen experimentieren und manchmal auch drogenabhängig werden. Kulturelle und ethnische Gruppen weisen unterschiedliche Drogenkonsumraten auf. Jede Art von Einfluss – biologisch, psychisch und sozial-kulturell – bietet einen möglichen Ansatzpunkt für Programme zur Prävention und Behandlung von Drogenmissbrauch.

 

 

zurück zur Inhaltsübersicht

 

Kapitel 5 - Anlage, Umwelt und die Vielfalt der Menschen

 

5.1 – Was sind Chromosomen, DNA, Gene und menschliches Genom? Wie erklären Verhaltensgenetiker:innen unsere individuellen Unterschiede?

Gene sind die biochemischen Einheiten der Erbmasse, die die Chromosomen, die fadenförmigen Windungen der DNA, bilden. Wenn Gene exprimiert werden, liefern sie den Code für die Bildung der Proteine, welche die Bausteine unseres Körpers bilden. Die meisten menschlichen Eigenschaften werden durch das Zusammenspiel vieler Gene beeinflusst. Das menschliche Genom ist das gemeinsame genetische Profil, das den Menschen von anderen Spezies unterscheidet, und besteht auf individueller Ebene aus dem gesamten genetischen Material in den Chromosomen eines Organismus. Verhaltensgenetiker:innen untersuchen die relative Auswirkung und die Grenzen der genetischen (unsere Vererbung) und umweltbedingten Einflüsse auf das Verhalten. Die meisten unserer unterschiedlichen Eigenschaften werden von zahlreichen Genen und der Wechselwirkung unserer individuellen Umgebung mit diesen genetischen Veranlagungen beeinflusst.

 

5.2 – Wie helfen uns Zwillings- und Adoptionsstudien dabei, die Auswirkungen und das Zusammenspiel von Natur und Umwelt zu verstehen?

Studien mit eineiigen (monozygoten) Zwillingen im Vergleich zu zweieiigen (dizygoten) Zwillingen, getrennt aufwachsenden Zwillingen und biologischen Verwandten im Vergleich zu Adoptivkindern ermöglichen es den Forschenden, die Auswirkungen der gemeinsamen Umgebung und der gemeinsamen Gene zu untersuchen, was Aufschluss darüber gibt, wie Nature und Nurture unsere Eigenschaften beeinflussen. Das gemeinsame familiäre Umfeld hat erstaunlich wenig Einfluss auf die Persönlichkeit, auch wenn die Erziehung durch die Eltern einen Einfluss auf weitere Faktoren hat.

 

5.3 – Was haben Psycholog:innen über das Temperament gelernt?

Die Stabilität des Temperaments, d. h. der charakteristischen emotionalen Reaktivität und Intensität einer Person, von den ersten Lebenswochen an lässt auf eine genetische Veranlagung schließen. Der genetische Effekt zeigt sich in physiologischen Besonderheiten wie der Herzfrequenz und der Reaktionsfähigkeit des Nervensystems.

 

5.4 – Was ist Erblichkeit und in welchem Zusammenhang steht sie zu Individuen und Gruppen?

Die Erblichkeit beschreibt das Ausmaß, in dem die Unterschiede zwischen den Mitgliedern einer Gruppe auf die Gene zurückgeführt werden können. Vererbbare interindividuelle Unterschiede (bei Merkmalen wie Größe oder Intelligenz) müssen nicht unbedingt vererbbare Gruppenunterschiede bedeuten. Gene erklären meist, warum manche Menschen größer sind als andere, aber nicht, warum Menschen heute größer sind als vor einem Jahrhundert.

 

5.5 – Wie verändert die molekulargenetische Forschung unser Verständnis der Auswirkungen von Natur und Umwelt?

Unsere genetischen Dispositionen und unser Umfeld stehen in Wechselwirkung. Die Umwelt kann Genaktivitäten auslösen, und genetisch beeinflusste Merkmale können Reaktionen bei anderen Personen hervorrufen. Die Molekulargenetik erforscht die molekulare Struktur und Funktion von Genen, einschließlich solcher, die das Verhalten beeinflussen. Ein Ziel der molekularen Verhaltensgenetik ist die Identifizierung spezifischer Gene – oder, häufiger, von Genkombinationen –, die zusammen komplexe Merkmale (wie Körpergewicht, sexuelle Orientierung und Impulsivität) steuern oder Menschen einem höheren Risiko für Störungen aussetzen. Der Bereich der Epigenetik untersucht die molekularen Mechanismen, durch die die Umgebung die genetische Expression auslösen oder blockieren kann.

 

5.6 – Wie erklären Evolutionspsycholog:innen mithilfe der Prinzipien der natürlichen Selektion Verhaltenstendenzen?

Evolutionspsycholog:innen versuchen zu verstehen, wie unsere Eigenschaften und Verhaltenstendenzen durch die natürliche Selektion geformt werden, da genetische Variationen, die die Chancen der Fortpflanzung und des Überlebens in ihrer jeweiligen Umgebung erhöhen, am ehesten an künftige Generationen weitergegeben werden. Einige Variationen entstehen durch Mutationen, andere durch neue genetische Kombinationen bei der Befruchtung. Wir Menschen teilen ein genetisches Erbe und sind für Verhaltensweisen prädisponiert, die das Überleben und die Fortpflanzung unserer Vorfahr:innen begünstigten. Charles Darwins Evolutionstheorie ist eines der grundlegenden Konzepte der Biologie. Darwin nahm die heutige Anwendung evolutionärer Prinzipien in der Psychologie vorweg.

 

5.7 – Wie könnte ein Evolutionspsychologe Unterschiede zwischen Männern und Frauen in der Sexualität und in den Vorlieben bei der Partnerwahl erklären?

Frauen neigen dazu, bei der Wahl ihrer Sexualpartner wählerischer zu sein als Männer. Evolutionspsycholog:innen begründen dies damit, dass die Anziehungskraft der Männer auf mehrere gesunde, fruchtbar aussehende Partnerinnen ihre Chancen erhöht, ihre Gene weit zu verbreiten. Da Frauen Babys aufziehen und stillen, erhöhen sie ihre eigenen Überlebenschancen und die ihrer Kinder, indem sie nach Partnern suchen, die das Potenzial haben, langfristig in den gemeinsamen Nachwuchs zu investieren.

 

5.8 – Welches sind die Hauptkritikpunkte an evolutionstheoretischen Erklärungen menschlicher Sexualität und wie antworten Evolutionspsycholog:innen darauf?

Kritiker:innen argumentieren, dass Evolutionspsycholog:innen von einer Wirkung ausgehen und sich rückwärts zu einer Erklärung durcharbeiten, zeitgenössische soziale und kulturelle Einflüsse (einschließlich erlernter sozialer Skripte) minimieren und den Menschen die Verantwortung für ihr Sexualverhalten abnehmen. Evolutionspsycholog:innen entgegnen, dass sie die Bedeutung sozialer und kultureller Einflüsse anerkennen, aber den Wert von überprüfbaren Vorhersagen auf der Grundlage evolutionärer Prinzipien betonen: Wenn wir unsere Veranlagungen verstehen, können wir sie auch bewältigen.

 

5.9 – Wie können frühe Erfahrungen das Gehirn verändern?

Während sich das Gehirn eines Kindes entwickelt, werden die neuronalen Verbindungen zahlreicher und komplexer. Erfahrungen lösen dann einen Prozess des „Bereinigens“ aus, bei dem ungenutzte Verbindungen schwächer und stark genutzte stärker werden. Die frühe Kindheit ist ein wichtiger Zeitraum für die Entwicklung des Gehirns, aber dank der Plastizität verändert sich das Gehirn während unseres gesamten Lebens als Reaktion auf unser Lernen.

 

5.10 – Inwiefern formen Eltern und Gleichaltrige die Entwicklung eines Kinds?

Das familiäre Umfeld und die Erwartungen der Eltern können die Motivation und den künftigen Erfolg von Kindern beeinflussen. Die Persönlichkeit ist allerdings meist nicht dem Einfluss der Erziehung zuzuschreiben. Da Kinder versuchen, sich an ihre Altersgenossen anzupassen, neigen sie dazu, deren Kultur zu übernehmen - Stil, Akzent, Slang, Einstellungen. Indem sie die Nachbarschaft und die Schule ihrer Kinder auswählen, üben die Eltern einen gewissen Einfluss auf die Kultur der Gleichaltrigengruppe aus.

 

5.11 – Wie beeinflusst Kultur unser Verhalten?

Eine Kultur ist eine beständige Ansammlung von Verhaltensweisen, Ideen, Einstellungen, Werten und Traditionen, die von einer Gruppe geteilt und von einer Generation an die nächste weitergegeben werden. Kulturelle Normen sind verständliche Regeln, die die Mitglieder einer Kultur über akzeptierte und erwartete Verhaltensweisen informieren. Kulturen unterscheiden sich über Zeit und Raum.

 

5.12 – Wie unterscheiden sich individualistische und kollektivistische Kulturen in ihren Werten und Zielen?

Obwohl jede:r Einzelne individuell verschieden ist, tendieren verschiedene Kulturen dazu, entweder den Individualismus oder den Kollektivismus zu betonen. Kulturen, die sich auf einen selbstbestimmten Individualismus stützen, neigen dazu, persönliche Unabhängigkeit und individuelle Leistungen zu schätzen. Sie definieren Identität über Selbstwertgefühl, persönliche Ziele und Eigenschaften sowie persönliche Rechte und Freiheiten. Kulturen, die auf sozialem Kollektivismus basieren, legen eher Wert auf Gruppenziele, soziale Identität und Verpflichtungen. Sie definieren Identität durch gegenseitige Beziehungen, Tradition und Harmonie.

 

5.13 – Wie unterscheidet sich das soziale vom biologischen Geschlecht?

In der Psychologie bezieht sich der Begriff „Gender“ auf die gesellschaftlich und kulturell konstruierten Erwartungen, die damit verbunden sind, was es bedeutet, ein Junge, ein Mädchen, ein Mann oder eine Frau zu sein. „Sex“ bezieht sich auf unseren biologischen Status als Mann oder Frau, der durch unsere Chromosomen und Anatomie definiert ist. Man könnte sagen, dass unser Körper unser „Sex“ definiert, während unser Geist unser „Gender“ bestimmt.

 

5.14 – Welche biologischen und psychologischen Ähnlichkeiten sowie Unterschiede gibt es zwischen Männern und Frauen?

Dank unserer gemeinsamen genetischen Veranlagung sind wir uns eher ähnlich als verschieden - wir sehen, lernen und erinnern uns auf vergleichbare Weise und verfügen über vergleichbare Kreativität, Intelligenz und Emotionen. Männer und Frauen unterscheiden sich jedoch in Bezug auf Körpergröße, Zeitpunkt des Eintritts in die Pubertät, Lebenserwartung und Anfälligkeit für bestimmte Krankheiten. Männer geben mehr Aggressionen zu als Frauen, und sie sind eher körperlich (und nicht relational) aggressiv. Frauen legen mehr Wert auf soziale Bindungen; sie sind abhängiger und „tend and befriend“.

 

5.15 – Welche Faktoren tragen zu geschlechtsspezifischen Vorurteilen am Arbeitsplatz bei?

Unterschiede in der Wahrnehmung von Männern und Frauen, in der Entlohnung und in der familiären Verantwortung beeinflussen und spiegeln die geschlechtsspezifische Voreingenommenheit am Arbeitsplatz wider. In den meisten Gesellschaften haben Männer mehr soziale Macht, und ihr Führungsstil ist eher direktiv, während der von Frauen eher demokratisch ist. In ihrem alltäglichen Verhalten und ihren Interaktionen neigen Männer dazu, durchsetzungsfähiger und rechthaberischer zu sein, während Frauen eher unterstützend und entschuldigend auftreten.

 

5.16 – Wie beeinflussen Geschlechtshormone die pränatale und jugendliche geschlechtliche Entwicklung und was ist Intergeschlechtlichkeit?

Sowohl die Geschlechtschromosomen als auch die Sexualhormone beeinflussen die Entwicklung. Etwa sieben Wochen nach der Befruchtung löst ein Gen auf dem Y-Chromosom des Vaters – der entweder dieses oder ein X-Chromosom beisteuern kann (die Mutter steuert immer das letztere bei) – die Produktion von Testosteron aus. Dies fördert die Entwicklung der männlichen Geschlechtsorgane. Im vierten und fünften pränatalen Monat wird das Gehirn des Fötus von Sexualhormonen durchflutet, wobei sich aufgrund des höheren Testosteronspiegels des Mannes und der Eierstockhormone der Frau unterschiedliche Muster entwickeln. Wenn Frauen vor der Geburt ungewöhnlich hohen Mengen an männlichen Hormonen ausgesetzt sind, können sie später zu eher männlichen Interessen neigen. Eine weitere Hormonwelle tritt in der Pubertät auf und löst einen Wachstumsschub, die Entwicklung der primären und sekundären Geschlechtsmerkmale und die markanten Ereignisse der Menarche und Spermarche aus. Intersexuelle Menschen werden mit mittleren oder ungewöhnlichen Kombinationen von männlichen und weiblichen Chromosomen, Hormonen und Anatomie geboren.

 

5.17 – Wie unterscheiden sich Geschlechtsrollen und Geschlechtsidentität?

Die Geschlechterrollen, also die Verhaltensweisen, die eine Gesellschaft von Männern und Frauen erwartet, variieren je nach Ort und Zeit. Die Theorie des sozialen Lernens besagt, dass wir die Geschlechtsidentität – unser persönliches Gefühl, männlich, weiblich oder eine Kombination aus beidem zu sein – so lernen, wie wir auch andere Dinge lernen: durch Verstärkung, Bestrafung und Beobachtung. Kritische Stimmen argumentieren, dass auch die Kognition eine Rolle spielt, da die Geschlechtszugehörigkeit von Kind zu Kind unterschiedlich ist. Wir scheinen uns so anzupassen, wie wir uns wohlfühlen, ob das nun bedeutet, dass wir eine männliche Rolle, eine weibliche Rolle oder eine Mischung aus beidem (Androgynie) einnehmen. Die Geschlechtsidentität oder der Geschlechtsausdruck von Transgender-Personen unterscheiden sich von den Verhaltensweisen oder Merkmalen, die als typisch für das ihnen von Geburt an zugewiesene Geschlecht gelten. Ihre sexuelle Orientierung kann heterosexuell, homosexuell, bisexuell oder asexuell sein.

 

5.18 – Wie beeinflussen Natur, Umwelt und unsere eigenen Entscheidungen Geschlechtsrollen?

Die individuelle Entwicklung ist das Ergebnis des Zusammenspiels von biologischen, psychologischen und sozial-kulturellen Einflüssen. Zu den biologischen Einflüssen gehören unser gemeinsames menschliches Genom, individuelle Variationen, die vorgeburtliche Umwelt und geschlechtsspezifische Gene, Hormone und Physiologie. Zu den psychologischen Einflüssen gehören die Wechselwirkungen zwischen Genen und Umwelt, die Auswirkungen früher Erfahrungen auf neuronale Netze, Reaktionen, die durch unsere eigenen Merkmale wie Geschlecht und Temperament hervorgerufen werden, sowie persönliche Überzeugungen, Gefühle und Erwartungen. Zu den soziokulturellen Einflüssen gehören die Einflüsse von Eltern und Gleichaltrigen, kulturellen Traditionen und Werten sowie kulturellen Geschlechternormen.

 

 

zurück zur Inhaltsübersicht

 

Kapitel 6 - Entwicklung über die Lebensspanne

 

6.1 – Mit welchen drei Fragestellungen befassen sich Entwicklungspsycholog: innen?

Entwicklungspsycholog:innen untersuchen körperliche, kognitive und soziale Veränderungen während der gesamten Lebensspanne. Sie konzentrieren sich auf drei Themen: „Nature and Nurture“ (die Wechselwirkung zwischen unserem genetischen Erbe und unseren Erfahrungen), „Continuity and Stages“ (welche Entwicklungsphasen allmählich und kontinuierlich verlaufen und welche sich relativ abrupt verändern) und „Stability and Change“ (ob unsere Eigenschaften im Alter erhalten bleiben oder sich verändern).

 

6.2 – Wie verläuft die pränatale Entwicklung und wie wirken Teratogene auf diese Entwicklung?

Der Lebenszyklus beginnt mit der Empfängnis, wenn sich eine Samenzelle mit einer Eizelle zu einer Zygote vereint. Die inneren Zellen der Zygote werden zum Embryo, und die äußeren Zellen werden zur Plazenta. In den nächsten 6 Wochen entwickeln sich die ersten Körperorgane, und mit 9 Wochen ist der Fötus als Mensch erkennbar. Teratogene sind potenziell schädliche Stoffe, welche die Plazenta passieren und den sich entwickelnden Embryo oder Fötus schädigen können, wie beispielsweise beim fetalen Alkoholsyndrom.

 

6.3 – Über welche Fähigkeiten verfügen Neugeborene und wie untersuchen Forschende die mentalen Fähigkeiten von Säuglingen?

Babys werden mit Sinnesorganen und Reflexen geboren, die ihr Überleben und ihre sozialen Interaktionen mit Erwachsenen erleichtern. So lernen sie beispielsweise schnell, den Geruch ihrer Mutter zu identifizieren, und sie bevorzugen den Klang der menschlichen Stimme. Um die Fähigkeiten von Säuglingen zu erforschen, setzen Forschende Techniken ein, mit denen die Habituation getestet wird, wie z. B. das Novelty-Preference-Verfahren.

 

6.4 – Wie entwickeln sich unser Gehirn und unsere motorischen Fähigkeiten im Kleinkind- und Kindesalter?

Die Nervenzellen des Gehirns werden durch Veranlagung und Erfahrungen geformt. Während sich das Gehirn eines Kindes entwickelt, werden die neuronalen Verbindungen zahlreicher und komplexer. Die Erfahrungen lösen dann einen Prozess aus, bei dem ungenutzte Verbindungen schwächer und stark genutzte verstärkt werden. Die frühe Kindheit ist ein wichtiger Zeitraum für die Ausprägung des Gehirns, aber unser Gehirn verändert sich als Reaktion auf unser Lernen ein Leben lang. In der Kindheit entwickeln sich komplexe motorische Fähigkeiten - Sitzen, Stehen, Gehen - in einer vorhersehbaren Abfolge, wobei der Zeitpunkt dieser Abfolge von der individuellen Reifung und Kultur abhängt. Wir haben nur wenige oder gar keine bewussten Erinnerungen an Ereignisse, die vor dem Alter von etwa 4 Jahren stattfanden. Diese kindliche Amnesie ist zum Teil darauf zurückzuführen, dass wichtige Gehirnbereiche noch nicht ausgereift sind.

 

6.5 – Wie entwickelt sich der kindliche Verstand aus der Sicht von Piaget, Vygotsky und heutiger Wissenschaftler: innen?

In seiner Theorie der kognitiven Entwicklung schlug Jean Piaget vor, dass Kinder ihr Verständnis der Welt durch die Prozesse der Assimilation und Akkommodation aktiv konstruieren und verändern. Sie bilden Schemata, die ihnen helfen, ihre Erfahrungen zu organisieren. Ausgehend von der sensomotorischen Phase der ersten zwei Jahre, in der Kinder eine Objektpermanenz entwickeln, gehen sie zu komplexeren Denkweisen über. In der präoperationalen Phase (etwa im Alter von 2 bis etwa 6 oder 7 Jahren) entwickeln sie eine Theory of Mind. In der präoperationalen Phase sind Kinder egozentrisch und nicht in der Lage, einfache logische Operationen durchzuführen. Im Alter von etwa 7 Jahren treten sie in die konkret-operationale Phase ein und sind in der Lage, das Prinzip der Erhaltung zu verstehen. Im Alter von 12 Jahren treten Kinder in die formal-operationale Phase ein und können systematisch denken. Die Forschung stützt die von Piaget vorgeschlagene Abfolge, zeigt aber auch, dass junge Kinder fähiger sind und ihre Entwicklung kontinuierlicher verläuft, als er glaubte. Lev Vygotskys Studien zur kindlichen Entwicklung konzentrierten sich auf die Art und Weise, wie der kindliche Verstand durch die Interaktion mit dem sozialen Umfeld wächst. Seiner Ansicht nach bieten Eltern und Betreuungspersonen vorübergehende Strukturen, die es den Kindern ermöglichen, höhere Ebenen des Denkens zu erreichen.

 

6.6 – Was ist eine Autismus-Spektrum-Störung?

Die Autismus-Spektrum-Störung (ASS) ist eine Störung, die durch soziale Defizite und sich wiederholende Verhaltensweisen gekennzeichnet ist, wobei es unterschiedliche Schweregrade gibt. Kinder mit ASD haben eine gestörte Theory of Mind. Im Alter von acht Jahren wird heute bei einem von 68 US-Kindern eine ASD diagnostiziert (wobei die gemeldeten Werte je nach Ort variieren). Der Anstieg der Diagnosen wurde durch einen Rückgang der Zahl der Kinder mit einer „kognitiven Behinderung“ oder „Lernbehinderung“ ausgeglichen. Genetische Einflüsse, eine abnorme Gehirnentwicklung und die pränatale Umgebung – insbesondere, wenn sie durch Infektionen, Medikamente oder Hormone verändert wird – tragen wahrscheinlich zu ASD bei.

 

6.7 – Wie entsteht die Eltern-Kind-Bindung?

Mit etwa acht Monaten, kurz nach Entwicklung der Objektpermanenz, zeigen Kinder, die von ihren Bezugspersonen getrennt sind, Angst vor Fremden. Säuglinge gehen Bindungen mit Bezugspersonen ein, die ihre biologischen Bedürfnisse befriedigen, aber noch wichtiger ist, dass sie sich bei ihnen wohlfühlen, vertraut sind und auf sie eingehen. Viele Vögel und andere Tiere haben einen festeren Bindungsprozess, der als Prägung bezeichnet wird und in einer kritischen Phase stattfindet.

 

6.8 – Wie haben Forscher:innen Unterschiede in der Bindung untersucht und zu welchen Erkenntnissen kamen sie dadurch?

Bindung wurde in Experimenten mit fremden Situationen untersucht, die zeigen, dass einige Kinder sicher gebunden sind und andere unsicher (ängstlich oder vermeidend) gebunden sind. Die unterschiedlichen Bindungsstile von Kleinkindern spiegeln sowohl ihr individuelles Temperament als auch die Reaktionsfähigkeit ihrer Eltern und Betreuungspersonen wider. Die Beziehungen der Erwachsenen scheinen die Bindungsstile ihrer frühen Kindheit widerzuspiegeln, was Erik Eriksons Idee unterstützt, dass das Urvertrauen in der Kindheit durch die Erfahrungen mit aufmerksamen Bezugspersonen geprägt wird.

 

6.9 – Inwiefern beeinflussen Vernachlässigung oder Misshandlung das kindliche Bindungsverhalten?

Die meisten Kinder sind resilient, aber diejenigen, die von ihren Eltern stark vernachlässigt oder auf andere Weise daran gehindert wurden, in jungen Jahren Bindungen aufzubauen, haben möglicherweise ein Risiko für Bindungsprobleme. Extreme Traumata in der Kindheit können das Gehirn verändern, unsere Stressreaktionen beeinflussen oder epigenetische Spuren hinterlassen.

 

6.10 – Wie entwickelt sich das kindliche Selbstkonzept?

Das Selbstkonzept, also das Verständnis und die Bewertung dessen, wer wir sind, entwickelt sich allmählich. Mit 15 bis 18 Monaten erkennen Kinder sich selbst in einem Spiegel. Im Schulalter können sie viele ihrer eigenen Eigenschaften beschreiben, und im Alter von 8 oder 10 Jahren ist ihr Selbstbild stabil.

 

6.11 – Welche sind die vier wichtigsten Erziehungsstile?

Die wichtigsten Erziehungsstile sind autoritär (zwanghaft), permissiv (nicht einschränkend), vernachlässigend (unbeteiligt) und autoritativ (konfrontativ).

 

6.12 – Welche Auswirkungen gehen mit den jeweiligen Erziehungsstilen einher?

Autoritäre Erziehung wird mit höherem Selbstwertgefühl, Selbstvertrauen, Selbstregulierung und sozialer Kompetenz in Verbindung gebracht; autoritäre Erziehung mit geringerem Selbstwertgefühl, geringerer sozialer Kompetenz und Überreagieren auf Fehler; permissive Erziehung mit höherer Aggressivität und mangelnder Reife; und vernachlässigende Erziehung mit schlechteren schulischen und sozialen Leistungen. Eine Korrelation ist jedoch nicht gleichbedeutend mit Kausalität (es ist möglich, dass Kinder mit positiven Eigenschaften eher positive Erziehungsmethoden aufweisen).

 

6.13 – Wie wird Adoleszenz definiert und wie wirken sich die körperlichen Veränderungen auf die sich entwickelnden Jugendlichen aus?

Die Adoleszenz ist die Übergangszeit von der Kindheit zum Erwachsenenalter, die sich von der Pubertät bis zur sozialen Eigenständigkeit erstreckt. Jungen scheinen (wenn auch mit Risiken) von der „frühen“ Reifung zu profitieren, Mädchen von der „späten“ Reifung. In der Adoleszenz und in den frühen Zwanzigern reifen die Frontallappen des Gehirns und das Myelinwachstum nimmt zu, was zu einer besseren Urteilsfähigkeit, Impulskontrolle und langfristige Planung führt.

 

6.14 – Wie haben Piaget, Kohlberg und spätere Wissenschaftler die kognitive und moralische Entwicklung im Jugendalter beschrieben?

Piaget stellte die Theorie auf, dass Heranwachsende eine Fähigkeit zu formalen Operationen entwickeln und dass diese Entwicklung die Grundlage für moralische Urteile ist. Lawrence Kohlberg schlug eine Stufentheorie des moralischen Denkens vor, die von einer präkonventionellen Moral des Eigeninteresses über eine konventionelle Moral, die sich mit der Einhaltung von Gesetzen und sozialen Regeln befasst, bis hin zu einer (bei manchen Menschen) postkonventionellen Moral mit allgemeinen ethischen Grundsätzen reicht. Andere Forschende sind der Ansicht, dass die Moral nicht nur im Denken, sondern auch in der moralischen Intuition und im moralischen Handeln liegt. Kohlbergs Kritiker:innen stellen fest, dass die postkonventionelle Ebene kulturell begrenzt ist und nur aus der Perspektive einer individualistischen Gesellschaft Moral repräsentiert.

 

6.15 – Welche sind die sozialen Aufgaben und Herausforderungen des Jugendalters?

Erik Erikson schlug acht Stufen der psychosozialen Entwicklung über die gesamte Lebensspanne vor. Seiner Ansicht nach müssen wir die folgenden Aufgaben bewältigen: Vertrauen, Autonomie, Initiative, Kompetenz, Identität (in der Adoleszenz), Intimität (im jungen Erwachsenenalter), Generativität und Integrität. Jede Lebensphase hat ihre eigene psychosoziale Aufgabe. Die Festigung des Selbstbewusstseins in der Adoleszenz bedeutet, dass man eine Reihe verschiedener Rollen ausprobiert. Die soziale Identität ist der Teil des Selbstkonzepts, das sich aus den Gruppenzugehörigkeiten einer Person ergibt.

 

6.16 – Wie werden Jugendliche von ihren Eltern und Gleichaltrigen beeinflusst?

Während der Adoleszenz nimmt der elterliche Einfluss ab und der Einfluss der Gleichaltrigen zu. Jugendliche übernehmen die Art und Weise, wie sich Gleichaltrige kleiden, verhalten und kommunizieren. Die Eltern haben mehr Einfluss auf die Bereiche von Religion, Politik, Bildung und Berufswahl.

 

6.17 – Was bedeutet der Übergang ins Erwachsenenalter?

Aufgrund der früheren Geschlechtsreife und der späteren Eigenständigkeit dauert der Übergang von der Jugend zum Erwachsensein länger als früher. Das heranwachsende Erwachsenenalter ist der Zeitraum zwischen dem 18. und dem mittleren 20. Lebensjahr, in dem viele junge Menschen noch nicht völlig autonom sind. Diese Phase ist vor allem in den heutigen westlichen Kulturen anzutreffen.

 

6.18 – Wie verändern wir uns im mittleren und hohen Erwachsenenalter körperlich?

Muskelkraft, Reaktionszeit, sensorische Fähigkeiten und Herzleistung nehmen ab Mitte zwanzig fast unmerklich ab; dieser Prozess beschleunigt sich im mittleren und späten Erwachsenenalter. Die Zeit der Fruchtbarkeit endet bei Frauen mit der Menopause etwa im Alter von 50 Jahren, während sie bei Männern allmählich abnimmt. Im späten Erwachsenenalter wird das Immunsystem schwächer, was die Anfälligkeit für lebensbedrohliche Krankheiten erhöht. Die Chromosomenenden (Telomere) nutzen sich ab, wodurch sich die Chancen auf eine normale genetische Replikation verringern. Bei einigen Menschen sorgen jedoch die Gene, die die Langlebigkeit unterstützen, sowie ein geringer Stress und gute Gesundheitsgewohnheiten für eine bessere Gesundheit im späteren Leben.

 

6.19 – Wie verändert sich das Gedächtnis mit dem Alter?

Das Erinnerungsvermögen nimmt ab, vor allem bei unbedeutenden Informationen, aber das Wiedererkennungsgedächtnis bleibt leistungsfähig. Ältere Erwachsene verlassen sich mehr auf Zeitmanagement und Gedächtnisstützen, um sich an zeitbezogene und gewohnheitsmäßige Aufgaben zu erinnern. Entwicklungsforschende untersuchen altersbedingte Veränderungen, z. B. des Gedächtnisses, mit Hilfe von Querschnittsstudien (Vergleich von Personen unterschiedlichen Alters zu einem bestimmten Zeitpunkt) und Längsschnittstudien (erneute Untersuchung derselben Personen über einen Zeitraum von mehreren Jahren). Der „terminale Verfall“ beschreibt den kognitiven Abbau in den letzten Lebensjahren.

 

6.20 – Wie wirken sich neurokognitive Störungen und die Alzheimer-Krankheit auf die kognitiven Fähigkeiten aus?

Neurokognitive Störungen (NCD) sind erworbene (nicht dauerhafte) Störungen, die durch kognitive Defizite gekennzeichnet sind und häufig mit der Alzheimer-Krankheit, Hirnverletzungen oder -krankheiten oder Drogenmissbrauch zusammenhängen. Diese Schädigung der Gehirnzellen führt zu einer Beeinträchtigung der geistigen Fähigkeiten, die nicht typisch für den normalen Alterungsprozess ist. Die Alzheimer-Krankheit führt zu einer Verschlechterung des Gedächtnisses und anschließend des Denkens. Nach 5 bis 20 Jahren ist die betroffene Person emotional verflacht, desorientiert, enthemmt, inkontinent und schließlich geistig verwirrt.

 

6.21 – Welche Themen und Einflüsse sind entscheidend für unsere soziale Entwicklung zwischen dem frühen Erwachsenenalter und dem Tod?

Erwachsene durchlaufen keine festgelegte Abfolge altersbedingter sozialer Phasen. Zufällige Ereignisse können die Lebensplanung beeinflussen. Die soziale Agenda ist der von einer Kultur bevorzugte Zeitpunkt für Ereignisse wie Heirat, Elternschaft und Ruhestand. Die dominierenden Themen des Erwachsenenalters sind Liebe und Arbeit (Eriksons Intimität und Generativität).

 

6.22 – Wie verändert sich unser Wohlbefinden über die Lebensspanne?

Unser Selbstvertrauen und unser Identitätsgefühl nehmen im Laufe des Lebens tendenziell zu. Studien zeigen, dass die Lebenszufriedenheit bis zur Phase des Lebensendes nicht vom Alter abhängt. Nach der mittleren Lebensphase nehmen die positiven Emotionen zu und die negativen ab; mit dem Alter nehmen die extremen Gefühle und Stimmungen ab.

 

6.23 – Welche Reaktionen kann der Tod einer nahestehenden Person auslösen?

Die Menschen trauern nicht in vorhersehbaren Phasen, wie früher angenommen wurde. Starke Gefühlsäußerungen führen nicht zur Linderung der Trauer, und eine Trauertherapie ist nicht wesentlich effektiver als eine Trauerarbeit ohne derartige Unterstützung. Das Leben kann auch nach einem Todesfall bejaht werden, insbesondere von Menschen, die das erleben, was Erikson ein Gefühl der Integrität nannte - das Gefühl, dass das eigene Leben sinnvoll ist.

 

 

zurück zur Inhaltsübersicht

 

Kapitel 7 - Wahrnehmung

 

7.1 – Was verstehen wir unter Sinnesempfindung und Wahrnehmung? Und was unter „Top-down“- und „Bottom-up“-Verarbeitung?

Empfindung ist der Prozess, durch den unsere Sinnesrezeptoren und unser Nervensystem Stimuli aus unserer Umwelt empfangen und verarbeiten. Wahrnehmung ist der Prozess der Organisation und Interpretation dieser Informationen, der das Erkennen von bedeutsamen Ereignissen ermöglicht. Empfindung und Wahrnehmung sind ein kontinuierlicher Prozess. Die Bottom-up-Verarbeitung ist die sensorische Analyse, die auf der Eingangsebene beginnt, wobei die Informationen von den Sinnesrezeptoren zum Gehirn fließen. Die Top-down-Verarbeitung ist eine Informationsverarbeitung, die von mentalen Prozessen auf höherer Ebene gesteuert wird, z. B. wenn wir Wahrnehmungen konstruieren, indem wir Informationen durch unsere Erfahrungen und Erwartungen filtern.

 

7.2 – Welche drei Schritte sind grundlegend für alle sensorischen Systeme?

Unsere sensorischen Systeme (1) nehmen sensorische Reize auf (oft mit Hilfe spezialisierter Rezeptorzellen), (2) wandeln diese Reize in neuronale Impulse um und (3) leiten die neuronalen Informationen an das Gehirn weiter. Transduktion ist der Prozess der Umwandlung einer Energieform in eine andere. Psychophysiker:innen untersuchen die Beziehungen zwischen den physikalischen Eigenschaften von Reizen und unseren psychologischen Erfahrungen mit den Reizen.

 

7.3 – Was versteht man unter absoluter Schwelle und Unterschiedsschwelle?

Unser absoluter Schwellenwert für einen Reiz ist der Mindestreiz, der notwendig ist, damit wir ihn in 50 Prozent der Fälle bewusst wahrnehmen können. Die Signalentdeckungstheorie sagt voraus, wie und wann wir einen schwachen Reiz inmitten von Hintergrundgeräuschen wahrnehmen können. Die absoluten Schwellenwerte sind individuell unterschiedlich und hängen von der Stärke des Signals sowie von unserer Erfahrung, unseren Erwartungen, unserer Motivation und unserer Wachsamkeit ab. Unsere Unterschiedsschwelle (auch „just noticeable difference“ oder „jnd“ genannt) ist der Unterschied, den wir in der Hälfte der Fälle zwischen zwei Reizen wahrnehmen können. Das Webersche Gesetz besagt, dass sich zwei Reize um einen konstanten Mindestprozentsatz (nicht um einen konstanten Betrag) unterscheiden müssen, um als unterschiedlich wahrgenommen zu werden.

 

7.4 – Wie werden wir von subliminalen Reizen beeinflusst?

Wir nehmen einige Reize unterschwellig wahr - in weniger als 50 Prozent der Fälle - und können durch diese Empfindungen beeinflusst werden. Aber auch wenn wir vorbereitet (priming) werden können, haben unterschwellige Reize keinen starken, nachhaltigen Einfluss.

 

7.5 – Welchen Nutzen ziehen wir aus sensorischer Adaptation?

Die sensorische Anpassung (unsere verringerte Empfindlichkeit gegenüber konstanten oder regelmäßigen Gerüchen, Geräuschen und Berührungen) lenkt unsere Aufmerksamkeit auf informative Veränderungen in unserer Umgebung.

 

7.6 – Wie nehmen unsere Erwartungen, Kontexte, Motivation und Emotionen Einfluss auf unsere Wahrnehmung?

Ein Wahrnehmungsschema ist eine mentale Prädisposition, die als Linse fungiert, durch die wir die Welt wahrnehmen. Unsere erlernten Konzepte (Schemata) veranlassen uns dazu, mehrdeutige Reize auf bestimmte Weise zu organisieren und zu interpretieren. Unsere Motivation sowie unser physischer und emotionaler Zustand können Erwartungen wecken und unsere Interpretation von Ereignissen und Verhaltensweisen beeinflussen.

 

7.7 – Was sind die Eigenschaften der Energie, die wir als sichtbares Licht sehen? Welche Strukturen in unserem Auge helfen dabei, diese Energie zu bündeln?

Was wir als Licht sehen, ist nur ein kleiner Bruchteil des breiten Spektrums der elektromagnetischen Energie. Der für den Menschen sichtbare Teil reicht von den blau-violetten bis zu den roten Lichtwellenlängen. Nachdem das Licht durch die Hornhaut in das Auge eingetreten ist, die Pupille und die Iris passiert hat und von der Linse gebündelt wurde, treffen die Lichtenergieteilchen auf die innere Oberfläche des Auges, die Netzhaut. Der Farbton, den wir in einem Licht wahrnehmen, hängt von seiner Wellenlänge ab, und seine Helligkeit von seiner Intensität.

 

7.8 – Wie verarbeiten die Stäbchen und Zapfen Informationen und auf welchem Weg gelangen diese Informationen vom Auge zum Gehirn?

Licht, das in das Auge eintritt, löst chemische Veränderungen aus, wodurch Lichtenergie in Nervenimpulse umgewandelt wird. Die Zapfen und Stäbchen auf der Rückseite der Netzhaut haben jeweils eine besondere Sensitivität - die Zapfen für Details und Farben, die Stäbchen für schwaches Licht und periphere Bewegungen. Nach der Verarbeitung durch Bipolar- und Ganglienzellen wandern die Nervenimpulse von der Netzhaut über den Sehnerv zum Thalamus und weiter zum visuellen Kortex.

 

7.9 – Wie nehmen wir Farben in unserer Umwelt wahr?

Nach der Drei-Farben-Theorie von Young-Helmholtz befinden sich auf der Netzhaut drei Arten von Farbrezeptoren. Die heutige Forschung hat drei Arten von Zapfen gefunden, die jeweils am empfindlichsten für die Wellenlängen einer der drei Grundfarben des Lichts (rot, grün oder blau) sind. Nach der Hering'schen Theorie der gegensätzlichen Prozesse gibt es drei weitere Farbprozesse (rot-gegen-grün, blau-gegen-gelb, schwarz-gegen-weiß). Die Forschung hat bestätigt, dass Nervenzellen in der Netzhaut und im Thalamus auf dem Weg zum Gehirn die Farbinformationen der Zapfen in Paare von Gegenfarben kodieren. Diese beiden Theorien und die sie stützende Forschung zeigen, dass die Farbverarbeitung in zwei Stufen erfolgt.

 

7.10 – Wo befinden sich Merkmalsdetektoren und was ist ihre Aufgabe?

Merkmalsdetektoren, spezialisierte Nervenzellen im visuellen Kortex, reagieren auf bestimmte Merkmale des visuellen Reizes, wie Form, Winkel oder Bewegung. Die Merkmalsdetektoren leiten Informationen an andere kortikale Bereiche weiter, wo Superzellencluster auf komplexere Muster reagieren.

 

7.11 – Auf welche Weise nutzt unser Gehirn Parallelverarbeitung, um visuelle Wahrnehmung zu konstruieren?

Durch Parallelverarbeitung verarbeitet das Gehirn mehrere Aspekte des Sehens (Farbe, Bewegung, Form und Tiefe) gleichzeitig. Andere neuronale Einheiten integrieren die Ergebnisse, vergleichen sie mit gespeicherten Informationen und ermöglichen so die Wahrnehmung.

 

7.12 – Wie haben die Gestaltpsycholog:innen Wahrnehmungsorganisation verstanden und was tragen das Figur-Grund-Prinzip und Gruppierungsprinzipien zu unserer Wahrnehmung bei?

Die Gestaltpsycholog:innen suchten nach Prinzipien, nach denen das Gehirn Teile von Sinneseindrücken zu Gestalten oder sinnvollen Formen zusammensetzt. Sie wiesen darauf hin, dass das Ganze mehr sein kann als die Summe seiner Teile, und stellten fest, dass wir sensorische Informationen filtern und unsere Wahrnehmungen konstruieren. Um ein Objekt zu erkennen, müssen wir es zunächst wahrnehmen (es als Figur sehen) und es von seiner Umgebung (dem Boden) unterscheiden. Wir bringen Ordnung und Form in die Reize, indem wir sie in sinnvolle Gruppen einteilen und dabei Regeln wie Nähe, Kontinuität und Geschlossenheit befolgen.

 

7.13 – Wie benutzen wir binokulare und monokulare Hinweise, um die Welt in drei Dimensionen wahrzunehmen und Bewegung wahrzunehmen?

Die Tiefenwahrnehmung ist unsere Fähigkeit, Objekte in dreidimensionaler Form zu sehen und die Entfernung zu beurteilen. Die visuelle Klippe und andere Forschungen zeigen, dass viele Spezies die Welt bei oder sehr bald nach der Geburt in drei Dimensionen wahrnehmen. Binokulare Wahrnehmungen, wie z. B. die Netzhautdisparität, sind Tiefenwahrnehmungen, die sich auf die Informationen beider Augen stützen. Monokulare Anhaltspunkte (wie z. B. relative Höhe, relative Größe, Interposition, relative Bewegung, lineare Perspektive sowie Licht und Schatten) lassen uns die Tiefe anhand von Informationen beurteilen, die nur von einem Auge übermittelt werden. Wenn sich Objekte bewegen, gehen wir davon aus, dass sich Objekte, die sich verkleinern, zurückziehen und Objekte, die sich vergrößern, sich nähern. Das Gehirn berechnet Bewegungen nur unzureichend, und besonders bei kleinen Kindern besteht die Gefahr, dass sie herannahende Gefahren wie Fahrzeuge falsch wahrnehmen. Eine schnelle Abfolge von Bildern auf der Netzhaut kann eine Illusion von Bewegung erzeugen, wie bei der stroboskopischen Bewegung oder dem Phi-Phänomen.

 

7.14 – Wie helfen uns Wahrnehmungskonstanzen, unsere Empfindungen als bedeutungsvolle Wahrnehmungen zu organisieren?

Wahrnehmungskonstanzen wie Farbe, Helligkeit, Form oder Größe ermöglichen es uns, Objekte trotz des sich verändernden Bildes, das sie auf unsere Netzhaut werfen, als stabil wahrzunehmen. Unser Gehirn konstruiert unsere Erfahrung mit der Farbe oder Helligkeit eines Objekts durch Vergleiche mit anderen Objekten in der Umgebung. Wenn wir die Größe eines Objekts kennen, erhalten wir Hinweise auf seine Entfernung; wenn wir seine Entfernung kennen, erhalten wir Hinweise auf seine Größe, aber manchmal deuten wir monokulare Entfernungsangaben falsch und ziehen falsche Schlüsse, wie bei der Mond-Illusion.

 

7.15 – Welchen Beitrag hat die Forschung zur Wiederherstellung des Sehvermögens, zur sensorischen Deprivation und zur Wahrnehmungsadaptation zu unserem Verständnis davon geleistet, wie Erfahrungen unsere Wahrnehmung beeinflussen?

Erfahrung leitet unsere Wahrnehmungsinterpretationen. Menschen, die von Geburt an sehbehindert sind und nach einer Operation wieder sehen können, fehlt die entsprechende Erfahrung, um Formen und Gestalten visuell zu erkennen. Die Forschung zur Sinneseinschränkung deutet darauf hin, dass es für einige Aspekte der Sinnes- und Wahrnehmungsentwicklung einen kritischen Zeitraum gibt. Ohne frühe Stimulation entwickelt sich die neuronale Organisation des Gehirns nicht normal. Menschen, die eine Brille tragen, die die Welt leicht nach links oder rechts oder sogar auf den Kopf stellt, erleben eine Anpassung ihrer Wahrnehmung. Sie sind zunächst desorientiert, aber es gelingt ihnen, sich an den neuen Zustand anzupassen.

 

7.16 – Welche Eigenschaften haben Luftdruckwellen, die wir als Klang erleben?

Bei den Schallwellen handelt es sich um Bewegungen der komprimierten und expandierten Luft. Unsere Ohren nehmen diese Veränderungen des Luftdrucks wahr und wandeln sie in Nervenimpulse um, die das Gehirn als Schall entschlüsselt. Schallwellen variieren in ihrer Amplitude, die wir als unterschiedliche Lautstärke wahrnehmen, und in ihrer Frequenz, die wir als unterschiedliche Tonhöhe empfinden.

 

7.17 – Wie wandelt das Ohr Schallenergie in neuronale Botschaften um?

Das Mittelohr ist die Kammer zwischen Trommelfell und Hörschnecke; das Innenohr besteht aus der Hörschnecke, den Bogengängen und dem Gleichgewichtsorgan („Saculus vestibularis“). Schallwellen, die durch den Gehörgang wandern, verursachen winzige Vibrationen im Trommelfell. Die Knochen des Mittelohrs verstärken die Schwingungen und leiten sie an die mit Flüssigkeit gefüllte Cochlea weiter. Die Schwingungen der Basilarmembran, die durch Druckveränderungen in der Cochlea-Flüssigkeit verursacht werden, führen zu einer Bewegung der winzigen Haarsinneszellen, wodurch neuronale Botschaften (über den Thalamus) an den auditorischen Kortex im Gehirn gesendet werden. Schallempfindungsschwerhörigkeit (oder Nerventaubheit) entsteht durch eine Schädigung der Haarzellen in der Cochlea oder der zugehörigen Nerven. Schallleitungsschwerhörigkeit entsteht durch eine Schädigung des mechanischen Systems, das die Schallwellen an die Cochlea weiterleitet. Cochlea-Implantate können bei einigen Menschen das Hörvermögen wiederherstellen.

 

7.18 – Wie können wir Lautstärke wahrnehmen, Tonhöhen unterscheiden und bestimmen, woher ein Geräusch kommt?

Die Lautstärke hängt nicht von der Intensität der Reaktion einer Hörzelle ab, sondern vielmehr von der Anzahl der aktivierten Hörzellen. Die Ortstheorie erklärt, wie wir hohe Töne hören, und die Frequenztheorie erklärt, wie wir tiefe Töne hören. Eine Kombination der beiden Theorien erklärt, wie wir Töne im mittleren Bereich hören. Die Schallwellen treffen auf das eine Ohr früher und intensiver als auf das andere. Um Töne zu lokalisieren, analysiert das Gehirn die winzigen Unterschiede zwischen den Tönen, die von den beiden Ohren empfangen werden, und berechnet die Schallquelle.

 

7.19 – Wie spüren wir Berührung?

Unser Tastsinn besteht eigentlich aus mehreren Sinnen - Druck, Wärme, Kälte und Schmerz -, die zusammen weitere Empfindungen hervorrufen, wie zum Beispiel „heiß“.

 

7.20 – Welche biologischen, psychologischen und soziokulturellen Faktoren beeinflussen unser Schmerzerleben? Wie helfen Placebos, Ablenkung und Hypnose bei der Schmerzkontrolle?

Die biopsychosoziale Sichtweise betrachtet unsere Schmerzwahrnehmung als die Summe biologischer, psychologischer und sozial-kultureller Einflüsse. Schmerz spiegelt Empfindungen top-down und Prozesse bottom-up wider. Eine Schmerztheorie besagt, dass sich ein „Tor“ im Rückenmark entweder öffnet, damit Schmerzsignale über kleine Nervenfasern zum Gehirn gelangen können, oder sich schließt, um die Weiterleitung zu verhindern. Bei der Schmerzbehandlung werden häufig physische und psychologische Elemente kombiniert. Die Kombination eines Placebos mit Ablenkung und die Verstärkung der Wirkung durch Hypnose (die unsere Reaktion auf Suggestionen erhöht) kann zur Schmerzlinderung beitragen. Posthypnotische Suggestion wird von einigen Ärzt:innen eingesetzt, um unerwünschte Symptome und Verhaltensweisen zu kontrollieren.

 

7.21 – Auf welche Weise ähneln sich Geschmack und Geruch und wie unterscheiden sie sich?

Geschmack und Geruch sind beides chemische Sinnesorgane. Der Geschmackssinn setzt sich aus fünf Grundempfindungen zusammen - süß, sauer, salzig, bitter und umami - sowie aus den Aromen, die mit den Informationen der Geschmacksrezeptorzellen der Geschmacksknospen interagieren. Für den Geruchssinn (Olfaktorik) gibt es keine Grundempfindungen. Wir riechen etwas, wenn Luftmoleküle eine winzige Ansammlung von 20 Millionen Rezeptorzellen in der oberen Hälfte jeder Nasenhöhle erreichen. Geruchsmoleküle lösen Kombinationen von Rezeptorzellen in einer bestimmten Anordnung aus, die der Geruchskortex interpretiert. Die Rezeptorzellen senden Nachrichten an den Riechkolben des Gehirns, dann an den Temporallappen und an Teile des limbischen Systems.

 

7.22 – Wie spüren wir Lage und Bewegung unseres Körpers im Raum?

Durch Kinästhetik nehmen wir Haltung und Bewegung unserer Körperteile wahr. Wir überwachen die Haltung und Bewegung unseres Kopfes (und damit unseres Körpers) und halten unser Gleichgewicht mit unserem Vestibulärsinn, der sich auf die Bogengänge und die vestibulären Kammern stützt, um die Neigung oder Drehung unseres Kopfes zu erfassen.

 

7.23 – Auf welche Weise beeinflusst sensorische Interaktion unsere Wahrnehmung und was ist Embodied Cognition?

Unsere Sinne beeinflussen sich gegenseitig. Zu dieser sensorischen Interaktion kommt es zum Beispiel, wenn der Geruch eines Lieblingsessens dessen Geschmack verstärkt. Embodied Cognition ist der Einfluss von Körperempfindungen, Gesten und anderen Befindlichkeiten auf kognitive Präferenzen und Urteile.

 

7.24 – Was wird im Rahmen der außersinnlichen Wahrnehmung behauptet und welchen Schluss ziehen die meisten forschenden Psycholog:innen, wenn sie diese Behauptungen auf den Prüfstand gestellt haben?

Die Parapsychologie ist die Lehre von den paranormalen Phänomenen, einschließlich der übersinnlichen Wahrnehmung (ESP) und der Psychokinese. Die drei am besten überprüfbaren Formen der ESP sind Telepathie (Kommunikation von Geist zu Geist), Hellsehen (Wahrnehmung entfernter Ereignisse) und Vorahnung (Wahrnehmung zukünftiger Ereignisse). Der Glaube an ESP bedeutet, dass das Gehirn in der Lage ist, ohne Sinneseindrücke wahrzunehmen; forschenden Psycholog:innen ist es nicht gelungen, ESP-Phänomene unter kontrollierten Bedingungen zu reproduzieren.

 

 

zurück zur Inhaltsübersicht

 

Kapitel 8 - Lernen

 

8.1 – Wie definieren wir Lernen und welches sind die grundlegenden Formen des Lernens?

Lernen ist der Prozess der Aneignung neuer Informationen oder Verhaltensweisen durch Erfahrung. Beim assoziativen Lernen lernen wir, dass bestimmte Ereignisse zusammen auftreten. Bei der klassischen Konditionierung lernen wir, zwei oder mehr Reize miteinander zu verbinden. Das natürliche Reagieren auf Reize, die wir nicht selbst kontrollieren, wird als respondentes Verhalten bezeichnet. Bei der operanten Konditionierung lernen wir, eine Reaktion und ihre Folgen zu assoziieren. Diese Assoziationen erzeugen operante Verhaltensweisen. Durch kognitives Lernen erwerben wir mentale Kenntnisse, die unser Verhalten steuern. Beim Beobachtungslernen beispielsweise lernen wir neue Verhaltensweisen, indem wir Ereignisse und andere beobachten.

 

8.2 – Wie lässt sich Lernen aus der Sicht des Behaviorismus beschreiben?

Iwan Pawlows Arbeiten über das klassische Konditionieren legten den Grundstein für den Behaviorismus, der die Auffassung vertritt, dass die Psychologie eine objektive Wissenschaft sein soll, die das Verhalten ohne Bezugnahme auf mentale Prozesse untersucht. Die Behavioristen glauben, dass die grundlegenden Gesetze des Lernens für alle Arten, einschließlich des Menschen, gleich sind.

 

8.3 – Wer war Pawlow, und was sind die Hauptbestandteile klassischer Konditionierung?

Iwan Pawlow, ein russischer Physiologe, entwickelte neue Experimente zum Lernen. Seine Forschungen in den letzten drei Jahrzehnten seines Lebens zu Beginn des 20. Jahrhunderts zeigten, dass die klassische Konditionierung eine grundlegende Form des Lernens ist. Das klassische Konditionieren ist eine Form des Lernens, bei der ein Organismus Reize miteinander in Verbindung bringt und Ereignisse vorwegnimmt. Ein UR ist ein Ereignis, das auf natürliche Weise als Reaktion auf einen Reiz auftritt (z. B. Speichelfluss). Ein US ist ein Reiz, der auf natürliche Weise und automatisch (ohne Lernen) die nicht gelernte Reaktion auslöst (z. B. löst Essen im Mund den Speichelfluss aus). Ein CS ist ursprünglich ein NS (neutraler Reiz, z. B. ein Ton), der nach Assoziation mit einem US (z. B. Essen) eine CR auslöst. Ein CR ist die erworbene Reaktion (Speichelfluss) auf den ursprünglich neutralen (aber nun konditionierten) Reiz.

 

8.4 – Was versteht man in der klassischen Konditionierung unter folgenden Prozessen: Erwerb, Löschung, Spontanerholung, Reizgeneralisierung und Reizdiskrimination?

Bei der klassischen Konditionierung ist die erste Phase der Erwerb, d. h. die Assoziation eines NS mit dem US, so dass der NS die CR auslöst. Der Erwerb erfolgt am leichtesten, wenn der NS kurz vor einem US präsentiert wird (idealerweise etwa eine halbe Sekunde zuvor), um den Organismus auf das bevorstehende Ereignis vorzubereiten. Dieses Phänomen unterstützt die Ansicht, dass die klassische Konditionierung biologisch adaptiv ist. Durch Konditionierung höherer Ordnung kann ein neuer NS zu einem neuen CS werden. Extinktion ist ein nachlassendes Reagieren, das auftritt, wenn der CS wiederholt von selbst ohne den US auftritt. Spontane Erholung ist das Auftreten einer zuvor ausgelöschten konditionierten Reaktion nach einer Ruhephase. Generalisierung ist die Tendenz, auf Stimuli zu reagieren, die einem CS ähnlich sind. Diskrimination ist die erlernte Fähigkeit, zwischen einem CS und anderen irrelevanten Reizen zu unterscheiden.

 

8.5 – Warum ist Pawlows Arbeit immer noch so wichtig?

Pawlow lehrte uns, dass bedeutende psychologische Vorgänge objektiv untersucht werden können und dass das klassische Konditionieren eine grundlegende Form des Lernens ist, die für alle Arten gilt.

 

8.6 – Wie wurde Pawlows Arbeit auf die Bereiche menschliche Gesundheit und Wohlbefinden angewandt? Wie wandte Watson Pawlows Prinzipien auf erlernte Ängste an?

Techniken der klassischen Konditionierung werden in vielen Bereichen zur Förderung der menschlichen Gesundheit und des Wohlbefindens eingesetzt, so auch in der Verhaltenstherapie bei bestimmten psychischen Störungen. Auch das Immunsystem des Körpers kann auf klassische Konditionierung reagieren. Pawlows Arbeit bildete auch die Grundlage für Watsons Idee, dass menschliche Emotionen und Verhaltensweisen zwar biologisch beeinflusst sind, aber hauptsächlich ein Zusammenspiel von konditionierten Reaktionen darstellen. Watson wandte die Prinzipien der klassischen Konditionierung in seiner Studie über „Little Albert“ an, um zu zeigen, wie bestimmte Ängste konditioniert werden können.

 

8.7 – Was ist operante Konditionierung?

Bei der operanten Konditionierung handelt es sich um eine Art des Lernens, bei der ein Verhalten gefestigt wird, wenn es von einem Verstärker gefolgt wird, oder vermindert wird, wenn es von einer Bestrafung gefolgt ist.

 

8.8 – Wer war Skinner, und wie wird operantes Verhalten verstärkt und geformt?

B. F. Skinner war ein Universitätsstudent der Anglistik und aufstrebender Schriftsteller, der später ein Psychologiestudium aufnahm. Er wurde zur einflussreichsten und umstrittensten Figur des modernen Behaviorismus. Auf der Grundlage von Edward Thorndikes Gesetz der Wirkung fanden Skinner und andere heraus, dass das Verhalten von Ratten oder Tauben, die in einer sogenannten Skinner-Box platziert wurden, durch Verstärker geformt werden kann, um eine sukzessive Annäherung an das gewünschte Verhalten zu bewirken.

 

8.9 – Wie unterscheiden sich positive und negative Verstärkung und was sind die grundlegenden Arten von Verstärkern?

Unter Verstärkung versteht man jede Konsequenz, die ein Verhalten intensiviert. Positive Verstärkung fügt einen erwünschten Anreiz hinzu, um die Häufigkeit eines Verhaltens zu erhöhen. Negative Verstärkung reduziert oder entfernt einen aversiven Reiz, um die Häufigkeit eines Verhaltens zu erhöhen. Primäre Verstärker (z. B. die Aufnahme von Nahrung bei Hunger oder das Ende der Übelkeit während einer Krankheit) sind von Natur aus befriedigend - es ist kein Lernen erforderlich. Konditionierte (oder sekundäre) Verstärker (wie Bargeld) sind befriedigend, weil wir gelernt haben, sie mit grundlegenderen Belohnungen (wie dem Essen oder den Medikamenten, die wir damit kaufen) in Verbindung zu bringen. Unmittelbare Verstärker (z. B. ein gekaufter Leckerbissen) bieten eine sofortige Belohnung; verzögerte Verstärker (z. B. ein Gehaltsscheck) erfordern die Fähigkeit, die Belohnung aufzuschieben.

 

8.10 – Wie beeinflussen die unterschiedlichen Verstärkungspläne das Verhalten?

Ein Verstärkungsplan legt fest, wie oft eine Reaktion verstärkt werden soll. Bei kontinuierlicher Verstärkung (d. h. die gewünschte Reaktion wird jedes Mal verstärkt, wenn sie auftritt) erfolgt das Lernen schnell, aber ebenso die Extinktion, wenn die Belohnungen ausbleiben. Bei partieller (intermittierender) Verstärkung (Reaktionen werden nur manchmal verstärkt) ist das anfängliche Lernen langsamer, aber das Verhalten ist viel robuster gegen Auslöschung. Verstärkungspläne mit festem Verhältnis verstärken das Verhalten nach einer bestimmten Anzahl von Reaktionen; Verstärkungspläne mit variablem Verhältnis nach einer unvorhersehbaren Anzahl. Bei Plänen mit festem Intervall wird das Verhalten nach einer bestimmten Zeitspanne verstärkt, bei Plänen mit variablem Intervall nach einer unvorhersehbaren Zeitspanne.

 

8.11 – Wie unterscheiden sich Bestrafung und negative Verstärkung und wie wirkt sich Bestrafung auf das Verhalten aus?

Bestrafung führt zu einer unerwünschten Konsequenz (z. B. Prügel) oder entzieht etwas Erwünschtes (z. B. Wegnahme eines Lieblingsspielzeugs), um die Auftretenswahrscheinlichkeit eines Verhaltens (Ungehorsam des Kindes) zu verringern. Negative Verstärkung (Einnahme einer Aspirintablette) beseitigt einen unerwünschten Reiz (Kopfschmerzen). Diese erwünschte Konsequenz (Schmerzfreiheit) erhöht die Wahrscheinlichkeit, dass das Verhalten (die Einnahme von Aspirin, um die Schmerzen zu beenden) wiederholt wird. Bestrafung kann unerwünschte Nebeneffekte haben, wie z. B. die Unterdrückung unerwünschter Verhaltensweisen, anstatt sie zu ändern, die Förderung von Diskrimination (so dass das unerwünschte Verhalten auftritt, wenn die bestrafende Person nicht anwesend ist), die Erzeugung von Angst, das Erlernen von Aggression und die Begünstigung von Depression und Gefühlen der Hilflosigkeit.

 

8.12 – Warum werden Skinners Auffassungen zum Verhalten des Menschen kontrovers diskutiert und wie könnte man seine Prinzipien der operanten Konditionierung in der Schule, im Sport, bei der Arbeit, in der Erziehung und in der persönlichen Weiterentwicklung anwenden?

Die Kritik an Skinners Prinzipien besagt, dass dieser Ansatz die Menschen entmenschlicht, indem er ihre persönliche Freiheit vernachlässigt und versucht, ihre Handlungen zu kontrollieren. Skinner entgegnete, dass die Handlungen der Menschen bereits durch externe Konsequenzen kontrolliert werden und dass Verstärkung als Mittel zur Verhaltenskontrolle humaner ist als Bestrafung. Die Lehrenden können die Verhaltensweisen der Schülerinnen und Schüler mit Hilfe von Shaping-Techniken steuern und interaktive Medien wie z. B. adaptive Online-Tests einsetzen, um ein unmittelbares Feedback zu geben. (Das Learning-Curve-System, das mit diesem Text erhältlich ist, bietet ein solches Feedback und ermöglicht es den Lernenden, das Tempo ihres Lernens selbst zu bestimmen). Trainer:innen können die Fähigkeiten und das Selbstvertrauen der Spieler:innen stärken, indem sie sie für kleine Verbesserungen belohnen. Manager:innen können Produktivität und Moral steigern, indem sie klar definierte und erreichbare Verhaltensweisen belohnen. Eltern können erwünschte Verhaltensweisen belohnen, nicht aber unerwünschte. Wir können unsere eigenen Verhaltensweisen formen, indem wir realistische Ziele setzen, planen, wie wir auf diese Ziele hinarbeiten, die Häufigkeit der gewünschten Verhaltensweisen überwachen, gewünschte Verhaltensweisen verstärken und die Belohnungen allmählich reduzieren, wenn Verhaltensweisen zur Gewohnheit werden.

 

8.13 – Wie unterscheiden sich klassische und operante Konditionierung?

Bei der operanten Konditionierung lernt ein Organismus Assoziationen zwischen seinem eigenen Verhalten und den daraus resultierenden Folgen; diese Form der Konditionierung beinhaltet operantes Verhalten (Verhalten, das auf die Umwelt einwirkt und belohnende oder bestrafende Konsequenzen nach sich zieht). Bei der klassischen Konditionierung bildet der Organismus Assoziationen zwischen Reizen und nicht von ihm beeinflussbaren Ereignissen; bei dieser Form der Konditionierung handelt es sich um respondentes Verhalten (automatische Reaktionen auf einen bestimmten Reiz).

 

8.14 – Wie beeinflussen biologische Beschränkungen das Lernen durch klassische und operante Konditionierung?

Die Fähigkeit eines jeden Lebewesens, ein Verhalten zu konditionieren, ist durch biologische Zwänge begrenzt, so dass manche Assoziationen leichter zu lernen sind. Jede Spezies erlernt Verhaltensweisen, die ihr Überleben sichern – ein Phänomen, das als „Preparedness“ bezeichnet wird. Diejenigen, die Geschmacksabneigungen rasch erlernten, aßen wahrscheinlich nicht noch einmal das gleiche giftige Nahrungsmittel und überlebten eher und hinterließen Nachkommen. Die Natur schränkt die Fähigkeit jeder Spezies sowohl zur klassischen als auch zur operanten Konditionierung ein. Unsere Bereitschaft, einen CS mit einem US zu assoziieren, der vorhersehbar und unmittelbar folgt, ist oft (aber nicht immer) adaptiv. Während des operanten Lernens können die Tiere eine instinktive Abweichung zeigen, indem sie zu biologisch veranlagten Mustern zurückkehren.

 

8.15 – Welche Bedeutung haben kognitive Prozesse bei der klassischen und operanten Konditionierung?

Bei der klassischen Konditionierung können die Tiere lernen, wann sie einen US zu erwarten haben, und sie können sich des Zusammenhangs zwischen Reizen und Reaktionen bewusst sein. Bei der operanten Konditionierung zeigen die Forschungen zum kognitiven Kartografieren und zum latenten Lernen die Bedeutung der kognitiven Prozesse beim Lernen. Andere Untersuchungen zeigen, dass übermäßige Belohnungen (die die extrinsische Motivation fördern) die intrinsische Motivation untergraben können.

 

8.16 – Was ist der Unterschied zwischen Beobachtungslernen und assoziativem Lernen? Wie kann Beobachtungslernen durch neuronale Spiegelung ermöglicht werden?

Beim Beobachtungslernen lernen wir durch Beobachtung und Nachahmung, anstatt Assoziationen zwischen verschiedenen Ereignissen zu erwerben. Wir lernen, die Folgen eines Verhaltens vorherzusehen, weil wir stellvertretende Verstärkung oder stellvertretende Bestrafung wahrnehmen. Die Frontallappen unseres Gehirns sind nachweislich in der Lage, die Aktivität des Gehirns eines anderen Menschen zu spiegeln. Einige Psycholog:innen glauben, dass dies durch Spiegelneurone ermöglicht wird; andere argumentieren, dass dies eher auf die verteilten Netzwerke des Gehirns zurückzuführen ist.

 

8.17 – Welchen Einfluss haben prosoziale Vorbilder und antisoziale Vorbilder?

Kinder neigen dazu, das nachzuahmen, was ein Vorbild tut und sagt, unabhängig davon, ob das Verhalten des Vorbilds prosozial (positiv, konstruktiv und hilfreich) oder antisozial ist. Wenn die Handlungen und Worte eines Vorbilds widersprüchlich sind, ahmen Kinder möglicherweise die beobachtete Heuchelei nach.

 

8.18 – Welche Wirkung erzeugt das Betrachten von Gewalt?

Gewalt in den Medien kann zu Aggression beitragen. Dieser Effekt der Gewaltdarstellung kann durch Nachahmung und Desensibilisierung hervorgerufen werden. Korrelation ist nicht gleich Kausalität, aber die Versuchspersonen haben aggressiver reagiert, wenn sie Gewalt gesehen haben (anstatt sich mit Gewaltfreiheit zu beschäftigen).

 

 

zurück zur Inhaltsübersicht

 

Kapitel 9 - Gedächtnis

 

9.1 – Was versteht man unter Gedächtnis, und wie wird es gemessen?

Das Gedächtnis ist ein dauerhaftes Lernen durch die Kodierung, Speicherung und den Abruf von Informationen. Ein Indiz für ein Erinnerungsvermögen ist die Fähigkeit, Informationen abzurufen, sie wiederzuerkennen oder sie bei einem erneuten Lernvorgang leichter wieder zu lernen.

 

9.2 – Wie beschreiben Psycholog:innen das menschliche Gedächtnissystem?

Psycholog:innen verwenden Gedächtnismodelle, um über das Gedächtnis zu reflektieren und zu kommunizieren. Informationsverarbeitungsmodelle umfassen drei Prozesse: Kodierung, Speicherung und Abruf. Unser agiles Gehirn verarbeitet viele Dinge gleichzeitig, indem es sie parallel verarbeitet. Das konnektionistische Informationsverarbeitungsmodell konzentriert sich auf diese mehrspurige Verarbeitung und betrachtet Erinnerungen als Produkte miteinander verbundener neuronaler Netzwerke. Die drei Verarbeitungsstufen im Modell nach Atkinson-Shiffrin sind das sensorische Gedächtnis, das Kurzzeitgedächtnis und das Langzeitgedächtnis. Dieses Modell wurde inzwischen aktualisiert und um zwei wichtige Konzepte erweitert: (1) Arbeitsgedächtnis, um die aktive Verarbeitung in der zweiten Gedächtnisphase zu betonen, und (2) automatische Verarbeitung, um die Verarbeitung von Informationen außerhalb des bewussten Bewusstseins zu berücksichtigen.

 

9.3 – Wie lassen sich explizite und implizite Erinnerungen unterscheiden?

Das menschliche Gehirn verarbeitet Informationen auf zwei Wegen: bewusst und unbewusst. Explizite (deklarative) Erinnerungen - unsere bewussten Erinnerungen an Fakten und Erlebnisse - entstehen durch aufwändige Verarbeitung, die bewusste Anstrengung und Aufmerksamkeit erfordert. Implizite (nicht deklarative) Erinnerungen - an Fertigkeiten und klassisch konditionierte Assoziationen - entstehen ohne unser Bewusstsein, durch automatische Verarbeitung.

 

9.4 – Welche Informationen verarbeiten wir automatisch?

Zusätzlich zu Fertigkeiten und klassisch konditionierten Assoziationen verarbeiten wir automatisch auch zufällige Informationen über Raum, Zeit und Frequenz.

 

9.5 – Wie funktioniert das sensorische Gedächtnis?

Das sensorische Gedächtnis übermittelt einige Informationen an das Arbeitsgedächtnis, um sie dort aktiv zu verarbeiten. Ein ikonisches Gedächtnis ist ein sehr kurzes (einige Zehntelsekunden) sensorisches Gedächtnis für visuelle Reize; ein echoisches Gedächtnis ist ein drei- oder viersekündiges sensorisches Gedächtnis für auditive Reize.

 

9.6 – Wie groß ist die Kapazität unseres Kurzzeitgedächtnisses?

Die Kapazität des Kurzzeitgedächtnisses liegt bei etwa sieben Items, plus oder minus zwei, aber diese Informationen verschwinden ohne Wiederholung sehr schnell aus dem Gedächtnis. Die Kapazität unseres Arbeitsgedächtnisses für eine aktive Verarbeitung variiert je nach Alter, Intelligenzniveau und anderen Faktoren.

 

9.7 – Welche bewussten Verarbeitungsstrategien können uns beim Erinnern an neue Informationen helfen?

Zu den wirksamen Strategien für eine effektive Verarbeitung gehören Chunking, Mnemotechniken, Hierarchien und verteilte Übungssitzungen (der Spacing-Effekt). Der Testeffekt besagt, dass das bewusste Abrufen von Informationen das Gedächtnis verbessert, anstatt die Informationen nur zu wiederholen.

 

9.8 – Welche Informationsverarbeitungsebenen gibt es und wie beeinflussen sie die Enkodierung?

Die Tiefe der Verarbeitung wirkt sich auf das Langzeitgedächtnis aus. Bei der oberflächlichen Verarbeitung kodieren wir Wörter auf der Grundlage ihrer Struktur oder ihres Erscheinungsbilds. Die Gedächtnisleistung ist am besten, wenn wir eine tiefe Verarbeitung nutzen und Wörter auf der Grundlage ihrer Bedeutung kodieren. Wir erinnern uns auch leichter an Inhalte, die für uns persönlich von Bedeutung sind der Selbstbezugseffekt.

 

9.9 – Wie groß ist die Kapazität des Langzeitgedächtnisses und wo ist es lokalisiert? Werden unsere Langzeiterinnerungen an bestimmten Orten verarbeitet und gespeichert?

Die Kapazität unseres Langzeitgedächtnisses ist im Wesentlichen unbegrenzt. Erinnerungen werden im Gehirn nicht an einzelnen Orten abgelegt, sondern an vielen Stellen. Beim Kodieren, Speichern und Abrufen von Erinnerungen wirken viele Teile des Gehirns zusammen.

 

9.10 – Welche Rolle spielen der Frontallappen und der Hippocampus beim Speichern von Erinnerungen?

Die Frontallappen und der Hippocampus sind Teile des Gehirnnetzwerks, das für die explizite Gedächtnisbildung zuständig ist. Viele Hirnregionen senden Informationen zur Verarbeitung an die Frontallappen. Der Hippocampus nimmt mit Hilfe der umliegenden Kortexbereiche Elemente expliziter Erinnerungen (semantischer oder episodischer Art) auf und speichert sie vorübergehend, bevor er sie zur langfristigen Speicherung an andere Gehirnregionen weiterleitet. Die neuronale Speicherung von Langzeiterinnerungen wird als Gedächtniskonsolidierung bezeichnet.

 

9.11 – Welche Rolle spielen das Zerebellum und die Basalganglien bei unseren Gedächtnisprozessen?

Das Zerebellum und die Basalganglien sind Teile des Gehirnnetzwerks, das für die Bildung impliziter Erinnerungen zuständig ist. Das Zerebellum ist wichtig für die Speicherung klassisch konditionierter Gedächtnisinhalte. Die Basalganglien sind an der Motorik beteiligt und helfen bei der Bildung von prozeduralen Speicherinhalten für Fähigkeiten. Viele der in den ersten vier Lebensjahren erlernten Reaktionen und Fertigkeiten bleiben bis ins Erwachsenenalter erhalten, auch wenn wir uns nicht bewusst an das Erlernen dieser Assoziationen und Fähigkeiten erinnern können (infantile Amnesie).

 

9.12 – Wie beeinflussen Emotionen unsere Verarbeitung von Erinnerungen?

Emotionale Erregung bewirkt eine Ausschüttung von Stresshormonen, die zu einer Aktivität in den gedächtnisbildenden Bereichen des Gehirns führt. Erheblich belastende Ereignisse können sehr starke Flashbulb-Erinnerungen auslösen.

 

9.13 – Wie wirken sich Veränderungen auf der Ebene der Synapsen auf die Verarbeitung von Erinnerungen aus?

Die Langzeitpotenzierung (LTP) ist die neuronale Grundlage des Lernens. Bei der LTP werden die Neuronen effizienter bei der Freisetzung und Wahrnehmung von Neurotransmittern, und es entstehen mehr Verbindungen zwischen den Neuronen.

 

9.14 – Wie beeinflussen externe Hinweise, innere Empfindungen und die Reihenfolge der Präsentation den Abruf von Informationen aus dem Gedächtnis?

Äußere Reize aktivieren Assoziationen, die uns helfen, Erinnerungen abzurufen; dieser Prozess kann ohne unser Bewusstsein ablaufen, wie es beim Priming der Fall ist. Das Prinzip der Enkodierungsspezifität besagt, dass uns für eine bestimmte Erinnerung spezifische Hinweise und Kontexte am effektivsten helfen, sie wieder abzurufen. Die Wiederherstellung desselben physischen Kontextes oder emotionalen Zustands (Stimmungskongruenz), in dem wir eine Erinnerung gebildet haben, kann uns helfen, sie abzurufen. Der Reihenpositions-Effekt erklärt unsere Tendenz, die letzten Elemente einer Liste (die sich noch im Arbeitsgedächtnis befinden können) und die ersten Elemente (die wir länger einstudiert haben) am besten abzurufen.

 

9.15 – Warum vergessen wir?

Anterograde Amnesie ist eine mangelnde Fähigkeit, neue Erinnerungen zu bilden. Retrograde Amnesie ist die fehlende Fähigkeit, alte Erinnerungen abzurufen. Normales Vergessen kann auftreten, weil wir Informationen nie kodiert haben (Kodierungsversagen), weil die physische Spur zerfallen ist (Speicherzerfall) oder weil wir nicht abrufen können, was wir kodiert und gespeichert haben (Abrufversagen). Abrufprobleme können durch proaktive (vorwärtsgerichtete) Interferenz entstehen, wenn früheres Lernen den Abruf neuer Informationen beeinträchtigt, oder durch retroaktive (rückwärtsgerichtete) Interferenz, wenn neues Lernen den Abruf alter Informationen stört. Motiviertes Vergessen tritt auf, aber Forschende haben kaum Hinweise auf eine Verdrängung gefunden.

 

9.16 – Wie beeinflussen Fehlinformationen, Imagination und Quellenamnesie das Gedächtnis und wie stellen wir fest, ob eine Erinnerung wahr oder falsch ist?

Erinnerungen können nach dem Abruf ständig revidiert werden, ein Prozess, den Gedächtnisforscher als Rekonsolidierung bezeichnen. In Experimenten, die den Fehlinformations-Effekt belegen, haben Menschen falsche Erinnerungen mit irreführenden Details gebildet, nachdem sie nach einem Ereignis falsche Informationen erhalten oder sich wiederholt etwas vorgestellt und eingeprägt hatten, was nie passiert war. Wenn wir eine Erinnerung während des Abrufs wieder zusammensetzen, schreiben wir sie möglicherweise der falschen Quelle zu (Quellenamnesie). Die Quellenamnesie kann eine Erklärung für Déjà-vu-Erlebnisse liefern. Falsche Erinnerungen fühlen sich wie echte Erinnerungen an und können anhaltend sein, beschränken sich aber in der Regel auf den Kern des Ereignisses.

 

 9.17 – Warum sind Berichte über verdrängte und wieder aufgedeckte Erinnerungen so heftig diskutiert worden?

Im Mittelpunkt der Debatte steht die Frage, ob Erinnerungen an frühkindlichen Missbrauch verdrängt werden und in einer Therapie wieder zugänglich gemacht werden können. Inzest und Missbrauch kommen häufiger vor, als früher angenommen wurde. Sofern das Opfer nicht zu jung war, um sich daran erinnern zu können, werden solche Traumata in der Regel nicht verdrängt, sondern lebhaft in Erinnerung behalten. Psycholog:innen sind sich einig, dass sexueller Missbrauch vorkommt, dass Ungerechtigkeit vorkommt, dass Vergessen vorkommt, dass wiedererlangte Erinnerungen häufig sind, dass Erinnerungen an Ereignisse, die vor dem vierten Lebensjahr passiert sind, unzuverlässig sind, dass Erinnerungen, die unter Hypnose "wiederhergestellt" wurden, besonders unzuverlässig sind und dass Erinnerungen, ob echt oder falsch, emotional aufwühlend sein können.

 

 9.18 – Wie verlässlich sind Augenzeugenaussagen jüngerer Kinder?

Die Schilderungen von Kindern als Augenzeug:innen unterliegen denselben Gedächtniseinflüssen, die auch die Berichte von Erwachsenen verzerren. Wenn sie kurz nach einem Ereignis mit neutralen Worten, die sie verstehen, befragt werden, können sich Kinder genau an die Ereignisse und die daran beteiligten Personen erinnern.

 

 9.19 – Wie können Sie die Erkenntnisse der Gedächtnisforschung nutzen, um im Studium bessere Leistungen zu zeigen?

Die Ergebnisse der Gedächtnisforschung legen die folgenden Strategien zur Verbesserung des Gedächtnisses nahe: Lernen Sie regelmäßig, verleihen Sie dem Stoff eine Bedeutung, aktivieren Sie Abrufhinweise, verwenden Sie Gedächtnisstützen, minimieren Sie proaktive und retroaktive Interferenzen, schlafen Sie ausreichend, und testen Sie sich selbst, um sicher zu sein, dass Sie den Stoff abrufen und wiedergeben können.

 

 

zurück zur Inhaltsübersicht

 

Kapitel 10 - Denken und Sprache

 

10.1 – Was ist Kognition und was sind die Funktionen von Begriffen?

Kognition bezieht sich auf alle geistigen Aktivitäten, die mit Denken, Wissen, Erinnern und Kommunizieren verbunden sind. Wir verwenden Konzepte, mentale Gruppierungen ähnlicher Objekte, Ereignisse, Ideen oder Menschen, um die Welt um uns herum zu vereinfachen und zu organisieren. Die meisten Konzepte bilden wir um Prototypen oder Musterbeispiele für eine Kategorie herum.

 

10.2 – Welche kognitiven Strategien helfen uns beim Problemlösen und welche Hindernisse stehen uns im Weg?

Ein Algorithmus ist eine methodische, logische Regel oder Prozedur (z. B. eine Schritt-für-Schritt-Beschreibung für die Evakuierung eines Gebäudes während eines Brandes), die eine Lösung für ein Problem sicherstellt. Eine Heuristik ist eine einfachere Strategie (z. B. zum Ausgang rennen, wenn man Rauch riecht), die in der Regel schneller als ein Algorithmus ist, aber auch fehleranfälliger. Einsicht ist keine strategiebasierte Lösung, sondern eher ein plötzlicher Geistesblitz, der ein Problem löst. Zu den Hindernissen bei der Problemlösung gehören der Confirmation Bias, der uns dazu veranlasst, unsere Hypothesen eher zu bestätigen als zu hinterfragen, und Fixierungen, wie z. B. mentale Festlegungen, die uns daran hindern können, die neue Perspektive einzunehmen, die zu einer Lösung führen würde.

 

10.3 – Was ist Intuition und wie können die Verfügbarkeits- und Repräsentativitätsheuristik unsere Entscheidungsfindung und Urteilsbildung beeinflussen?

Intuition ist das einfache, unmittelbare, automatische Gefühl oder Denken, das wir oft anstelle von systematischem Denken einsetzen. Heuristiken, wie z. B. die Repräsentativitätsheuristik, ermöglichen schnelle Urteile. Mit der Verfügbarkeitsheuristik beurteilen wir die Wahrscheinlichkeit von Ereignissen danach, wie leicht sie uns in den Kopf kommen.

 

10.4 – Welche Faktoren verstärken unsere Ängste vor unwahrscheinlichen Ereignissen?

Wir neigen dazu, uns vor dem zu fürchten, worauf uns die Evolution vorbereitet hat, vor dem, was wir nicht kontrollieren können, was greifbar ist und was am leichtesten verfügbar ist. Wir fürchten zu wenig die ständigen Bedrohungen, die ein Leben nach dem anderen fordern, wie Verkehrsunfälle und Krankheiten.

 

10.5 – Wie werden unsere Entscheidungen und Urteile durch Selbstüberschätzung, Beharren auf Überzeugungen und Framing beeinflusst?

Selbstüberschätzung kann dazu führen, dass wir die Wahrheitsgehalte unserer Überzeugungen überbewerten. Wenn eine Überzeugung, die wir gebildet und erklärt haben, in Zweifel gezogen wurde, kann uns das Festhalten an dieser Überzeugung dazu veranlassen, an ihr zu verharren. Ein Mittel gegen das Beharren auf einer Überzeugung besteht darin, zu überlegen, wie wir ein gegenteiliges Ergebnis hätten erklären können. Framing ist die Art und Weise, wie eine Frage oder Aussage präsentiert wird. Subtile Unterschiede in der Darstellung können unsere Reaktionen erheblich verändern.

 

10.6 – Wie nutzen kluge Denker:innen ihre Intuition?

Kluge Denker:innen begrüßen ihre Intuitionen (die in der Regel lernfähig sind), wissen aber auch, wann sie diese ignorieren sollten. Bei komplexen Entscheidungen kann es von Vorteil sein, so viele Informationen wie möglich zu sammeln und sich dann Zeit zu nehmen, um sie mit unserem zweigleisigen Verstand zu verarbeiten.

 

10.7 – Was ist Kreativität, und was fördert sie?

Kreativität, die Fähigkeit, neue und wertvolle Ideen zu entwickeln, korreliert in gewissem Maße mit der Begabung, ist aber mehr als Schulwissen. Eignungstests erfordern konvergentes Denken, aber Kreativität erfordert divergentes Denken. Robert Sternberg hat vorgeschlagen, dass zur Kreativität Fachwissen, phantasievolle Denkfähigkeiten, eine unternehmungslustige Persönlichkeit, intrinsische Motivation und ein kreatives Umfeld, das kreative Ideen anregt, unterstützt und weiterentwickelt, gehören.

 

10.8 – Was wissen wir darüber, wie andere Spezies denken?

Forschende ziehen aus dem Verhalten anderer Spezies Rückschlüsse auf deren Bewusstsein und Intelligenz. Studien an verschiedenen Spezies zeigen, dass viele andere Tiere Konzepte, Zahlen und Werkzeuge verwenden und dass sie das Gelernte von einer Generation zur nächsten weitergeben (kulturelle Übertragung). Und wie der Mensch zeigen auch einige andere Spezies Einsicht, Selbsterkenntnis, Altruismus, Kooperation und Trauer.

 

10.9 – Welche Gehirnregionen sind an Sprachverarbeitung und Sprachproduktion beteiligt?

Aphasie ist eine Beeinträchtigung der Sprache, die in der Regel durch eine Schädigung der linken Hemisphäre verursacht wird. Zwei wichtige Sprachverarbeitungsbereiche sind das Broca-Areal, eine Region des linken Frontallappens, die den Sprachexpressionen dient, und das Wernicke-Areal, eine Region im linken Temporallappen, die die Sprachrezeption steuert. Die Sprachverarbeitung ist auch auf andere Hirnareale verteilt, wobei verschiedene neuronale Netze für spezifische sprachliche Teilaufgaben zuständig sind.

 

10.10 – Was wissen wir über die Sprachfähigkeit anderer Spezies?

Forschende ziehen aus dem Verhalten anderer Spezies Rückschlüsse auf deren Bewusstsein und Intelligenz. Studien an verschiedenen Spezies zeigen, dass viele andere Tiere Konzepte, Zahlen und Werkzeuge verwenden und dass sie das Gelernte von einer Generation zur nächsten weitergeben (kulturelle Übertragung). Und wie der Mensch zeigen auch einige andere Arten Einsicht, Selbstbewusstsein, Altruismus, Kooperation und Trauer.

 

10.11 – Welche Beziehung besteht zwischen Sprache und Denken, und welchen Nutzen hat es, in Bildern zu denken?

Obwohl Benjamin Lee Whorfs Hypothese des sprachlichen Determinismus besagt, dass die Sprache das Denken bestimmt, ist es zutreffender zu sagen, dass die Sprache das Denken beeinflusst (sprachlicher Relativismus). Verschiedene Sprachen beinhalten verschiedene Arten des Denkens, und das Eintauchen in eine zweisprachige Erziehung kann das Denkvermögen verbessern. Wir denken oft in Bildern, wenn wir das implizite (nicht deklarative, prozedurale) Gedächtnis nutzen – unser automatisches Gedächtnissystem für motorische und kognitive Fähigkeiten und klassisch konditionierte Assoziationen. Das Denken in Bildern kann unsere Fähigkeiten verbessern, wenn wir kommende Ereignisse mental durchspielen. Prozesssimulation (Konzentration auf die Schritte, die zum Erreichen eines Ziels erforderlich sind) ist effektiv, aber Ergebnissimulation (Vorstellungen vom Erreichen des Ziels) bringt wenig.

 

 

zurück zur Inhaltsübersicht

 

Kapitel 11 - Intelligenz

11.1 – Wie definieren Psycholog:innen Intelligenz und welche Argumente sprechen für eine allgemeine Intelligenz?

Intelligenz ist die Fähigkeit, aus Erfahrungen zu lernen, Probleme zu lösen und Wissen zu nutzen, um sich an neue Situationen anzupassen. Charles Spearman schlug vor, dass wir eine allgemeine Intelligenz (g) haben, die allen geistigen Fähigkeiten zugrunde liegt. Durch seine Arbeit mit der Faktorenanalyse, einem statistischen Verfahren zur Ermittlung von Clustern verwandter Fähigkeiten, stellte er fest, dass Personen, die in einem Bereich hohe Werte erzielen, in der Regel auch in anderen Bereichen überdurchschnittlich gut abschneiden. L. L. Thurstone war anderer Meinung und identifizierte sieben verschiedene Bereiche für geistige Fähigkeiten. Dennoch blieb die Beobachtung bestehen, dass Personen, die in einem Cluster hohe Werte erzielten, auch in anderen Clustern hohe Werte erreichten, was ein weiterer Beweis für einen g-Faktor ist.

 

11.2 – Wie unterscheiden sich die Theorien multipler Intelligenzen von Gardner und Sternberg und welcher Kritik waren sie ausgesetzt?

Howard Gardner schlug acht unabhängige Intelligenzen vor (sprachliche, logisch-mathematische, musikalische, räumliche, körperlich-kinästhetische, intrapersonale, interpersonale und naturalistische Intelligenz) sowie eine mögliche neunte (existenzielle Intelligenz). Die unterschiedlichen Intelligenzen von Menschen mit Savant-Syndrom, Autismus-Spektrum-Störung (ASD) und bestimmten Arten von Hirnschäden scheinen seine Ansicht zu bestätigen. Robert Sternbergs triarchische Theorie geht von drei Intelligenzbereichen aus, die die Fähigkeiten in der realen Welt vorhersagen: analytisch (akademische Problemlösung), kreativ (bahnbrechende Intelligenz) und praktisch (Straßenintelligenz). Kritische Stimmen verweisen auf Untersuchungen, die einen allgemeinen Intelligenzfaktor bestätigt haben, der die Leistung in hohem Maße vorhersagt. Sehr erfolgreiche Menschen neigen jedoch auch dazu, gewissenhaft, gut vernetzt und hartnäckig zu sein, wobei sowohl Fähigkeiten als auch Motivation zählen.

 

11.3 – Was sind die vier Komponenten der emotionalen Intelligenz?

Emotionale Intelligenz, ein Aspekt der sozialen Intelligenz, umfasst die Fähigkeit, Emotionen wahrzunehmen, zu verstehen, zu steuern und zu nutzen. Emotional intelligente Menschen sind in der Regel glücklich, gesund und persönlich und beruflich erfolgreicher. Einige kritische Stimmen bezweifeln, dass die Bezeichnung dieser Fähigkeiten als „Intelligenz“ den Begriff zu weit ausdehnt.

 

11.4 – Was ist ein Intelligenztest, und wie unterscheiden sich Leistungs- und Eignungstests?

Ein Intelligenztest beurteilt die geistigen Fähigkeiten einer Person und vergleicht sie mit denen anderer, indem numerische Werte ermittelt werden. Eignungstests messen die Fähigkeit zu lernen, während Leistungstests messen, was wir bereits gelernt haben.

 

11.5 – Wann und warum wurden Intelligenztests entwickelt, und wie unterscheiden sich die heutigen Tests von frühen Intelligenztests?

Francis Galton, der fasziniert war von der Messung dessen, was er für erbliche Genialität hielt (um diejenigen mit außergewöhnlichen Fähigkeiten zur Fortpflanzung zu ermutigen), versuchte Ende des 19. Jahrhunderts einen einfachen Intelligenztest zu konstruieren, scheiterte jedoch. Alfred Binet, der zu einer umweltbedingten Erklärung der Intelligenzunterschiede neigte, gab Anfang des 19. Jahrhunderts den Anstoß für die moderne Intelligenztestbewegung in Frankreich, als er Fragen entwickelte, mit deren Hilfe sich die künftigen Leistungen der Kinder im Pariser Schulsystem vorhersagen ließen. Binet hoffte, dass sein Test die Schulbildung der Kinder verbessern würde, befürchtete aber, dass er dazu benutzt werden könnte, sie zu stigmatisieren. Zu Beginn des zwanzigsten Jahrhunderts überarbeitete Lewis Terman von der Stanford University die Arbeit von Binet für den Einsatz in den Vereinigten Staaten. Terman war der Meinung, dass sein Stanford-Binet-Test dazu beitragen könnte, Menschen die richtigen Chancen zu eröffnen, aber sein Glaube an eine Intelligenz, die von Geburt an festgelegt ist und sich je nach ethnischer Gruppe unterscheidet, machte Binets Befürchtung wahr, dass Intelligenztests dazu benutzt werden könnten, die Chancen von Kindern zu beschränken. William Stern steuerte das Konzept des IQ (Intelligenzquotient) bei. Die heute am weitesten verbreiteten Intelligenztests sind die Wechsler Adult Intelligence Scale (WAIS) und die Wechsler-Tests für Kinder. Diese Tests unterscheiden sich von ihren Vorgängerversionen dadurch, dass sie einen Gesamtintelligenzwert sowie Werte für das sprachliche Verständnis, die Wahrnehmungsorganisation, das Arbeitsgedächtnis und die Verarbeitungsgeschwindigkeit liefern.

 

11.6 – Was bedeutet der Begriff Normalverteilung, und was ist unter der Aussage zu verstehen, dass ein Test die Kriterien Normierung, Reliabilität und Validität erfüllt?

Die Verteilung von Testergebnissen bildet oft eine normalverteilte (glockenförmige) Kurve um den zentralen Durchschnittswert, mit immer weniger Ergebnissen an den Extremen. Die Standardisierung schafft eine Grundlage für aussagekräftige Leistungsvergleiche, indem ein Test an eine repräsentative Stichprobe künftiger Testpersonen vergeben wird. Zuverlässigkeit ist das Ausmaß, in dem ein Test konsistente Ergebnisse liefert (bei zwei Testhälften, bei alternativen Formen des Tests oder bei Wiederholungstests). Validität ist das Ausmaß, in dem ein Test das misst oder vorhersagt, was er messen soll. Ein Test hat inhaltliche Gültigkeit, wenn er das relevante Verhalten erfasst (so wie ein Fahrtest die Fahrfähigkeit misst). Er hat prädiktive Validität, wenn er ein Verhalten vorhersagt, das er vorhersagen soll. (Eignungstests haben prädiktive Validität, wenn sie künftige Leistungen vorhersagen können; ihre Vorhersagekraft ist am besten für die ersten Schuljahre).

 

11.7 – Was versteht man unter Querschnittstudien, was unter Längsschnittstudien, und warum ist es wichtig zu wissen, welche Methode verwendet wurde?

Die unterschiedlichen Ergebnisse von Querschnitts- und Längsschnittstudien zur Intelligenz dass die geistigen Fähigkeiten mit dem Alter abnehmen oder dass sie stabil bleiben (oder sogar zunehmen) verdeutlichen die Tatsache, dass in Querschnittsstudien Menschen aus verschiedenen Phasen und Lebensumständen verglichen werden. Dies kann eine hervorragende Momentaufnahme eines bestimmten Zeitpunkts liefern, aber Längsschnittstudien sind besser geeignet, um die Entwicklung von Merkmalen über einen längeren Zeitraum zu verfolgen.

 

11.8 – Wie beeinflusst das Altern die kristalline und fluide Intelligenz?

Querschnitts- und Längsschnittstudien haben gezeigt, dass die fluide Intelligenz bei älteren Erwachsenen abnimmt, was zum Teil darauf zurückzuführen ist, dass die neuronale Verarbeitung langsamer wird. Die kristalline Intelligenz nimmt hingegen tendenziell zu.

 

11.9 – Wie stabil sind Intelligenztestwerte über die Lebensspanne hinweg?

Die Stabilität der Ergebnisse von Intelligenztests nimmt mit dem Alter zu. Im Alter von 4 Jahren schwanken die Ergebnisse etwas, beginnen aber, die Ergebnisse von Jugendlichen und Erwachsenen vorherzusagen. Im frühen Jugendalter sind die Ergebnisse sehr stabil und prädiktiv.

 

11.10 – Welche Eigenschaften haben Menschen, die sich im unteren bzw. oberen Extrembereich der Intelligenz befinden?

Ein Intelligenztestergebnis von 70 oder weniger ist ein diagnostisches Kriterium für die Diagnose einer geistigen Behinderung; weitere Kriterien sind eingeschränkte konzeptionelle, soziale und praktische Fähigkeiten. Menschen mit einem hohen Intelligenzniveau sind in der Regel gesund und ausgeglichen und haben in der Regel auch besondere akademische Erfolge.

 

11.11 – Welche Befunde sprechen für die genetische Bedingtheit der individuellen Intelligenz, und was versteht man unter Erblichkeit?

Studien an Zwillingen, Familienmitgliedern und Adoptiveltern und -geschwistern weisen auf einen signifikanten erblichen Beitrag zum Intelligenzquotienten hin. Intelligenz ist polygenetisch. Die Erblichkeit ist der Anteil der Variation zwischen Individuen einer Gruppe, der auf die Gene zurückgeführt werden kann.

 

11.12 – Welche Befunde gibt es zu Umwelteinflüssen auf die individuelle Intelligenz?

Studien über Kinder, die in einem verarmten Umfeld mit geringer sozialer Interaktion aufgewachsen sind, zeigen, dass persönliche Erfahrungen die Leistung in Intelligenztests erheblich beeinflussen. Es gibt keine Belege dafür, dass normale, gesunde Kinder zu Genies geformt werden können, wenn sie in einer außergewöhnlich reichen Umgebung aufwachsen.

 

11.13 – Wie und warum unterscheiden sich die Geschlechter in Bezug auf geistige Fähigkeiten?

Männer und Frauen haben im Durchschnitt die gleichen Intelligenztestergebnisse, aber sie unterscheiden sich in einigen spezifischen Fähigkeiten. Mädchen können besser buchstabieren, sich flüssiger ausdrücken, Objekte besser lokalisieren, Gefühle besser erkennen und sind empfindlicher für Berührungen, Geschmack und Farben. Jungen übertreffen Mädchen im räumlichen Vorstellungsvermögen und in der damit zusammenhängenden Mathematik, obwohl sich Jungen und Mädchen im mathematischen Rechnen und in der allgemeinen mathematischen Leistung kaum unterscheiden. Auch bei den unteren und oberen Extremen der geistigen Fähigkeiten sind Jungen den Mädchen überlegen. Für diese geschlechtsspezifischen Unterschiede wurden evolutionäre und kulturelle Erklärungen vorgeschlagen.

 

11.14 – Wie und warum unterscheiden sich ethnische Gruppen in Bezug auf geistige Fähigkeiten?

Ethnische Gruppen unterscheiden sich in ihren durchschnittlichen Intelligenztestergebnissen. Es gibt Hinweise darauf, dass Umweltunterschiede für diese Gruppenunterschiede verantwortlich sind.

 

11.15 – Können Intelligenztests zu verzerrenden Ergebnissen führen? Wie wirkt sich Bedrohung durch Stereotype auf die Leistung der Testteilnehmenden aus?

Einstellungstests zielen darauf ab, vorherzusagen, wie gut eine Testperson in einer bestimmten Situation abschneiden wird. Daher sind sie zwangsläufig „verzerrt“ in dem Sinne, dass sie auf Leistungsunterschiede reagieren, die durch kulturelle Erfahrungen bedingt sind. Mit „unangemessen voreingenommen“ meinen Psycholog:innen, dass ein Test für eine Gruppe eine weniger genaue Vorhersage trifft als für eine andere. In diesem Sinne halten die meisten Expert:innen die wichtigsten Eignungstests für unvoreingenommen. Die Bedrohung durch Stereotypen, d. h. die Befürchtung, dass man aufgrund eines negativen Stereotyps bewertet wird, beeinträchtigt die Leistung bei allen Testarten.

 

 

zurück zur Inhaltsübersicht

 

Kapitel 12 - Quellen der Motivation: Hunger, Sex, Zugehörigkeit und Erfolg

 

12.1 – Wie definiert die Psychologie Motivation? Aus welchen Perspektiven betrachtet sie motiviertes Verhalten?

Motivation ist ein Bedürfnis oder ein Wunsch, der das Verhalten anregt und steuert. Die Instinkt- bzw. Evolutionsperspektive untersucht die genetischen Einflüsse auf komplexe Verhaltensweisen. Die Theorie der Triebreduktion untersucht, wie physiologische Bedürfnisse erregte Spannungszustände (Triebe) erzeugen, die uns dazu bringen, diese Bedürfnisse zu befriedigen. Umweltanreize können die Triebe verstärken. Das Ziel der Triebreduktion ist die Homöostase, die Aufrechterhaltung eines stabilen inneren Zustands. Die Arousal-Theorie geht davon aus, dass einige Verhaltensweisen (z. B. solche, die durch Neugierde ausgelöst werden) nicht die physiologischen Bedürfnisse reduzieren, sondern vielmehr durch die Suche nach einem optimalen Erregungsniveau ausgelöst werden. Das Yerkes-Dodson-Gesetz beschreibt die Beziehung zwischen Erregung und Leistung. Abraham Maslows Bedürfnispyramide schlägt eine Hierarchie der menschlichen Bedürfnisse vor, die von den Grundbedürfnissen bis zu den übergeordneten Bedürfnissen reicht.

 

12.2 – Welche physiologischen Faktoren rufen Hunger hervor?

Hungergefühle werden durch Magenkontraktionen ausgelöst, aber Hunger hat auch andere Ursachen. Nervenzentren im Gehirn, einige davon im Hypothalamus, überwachen die Blutwerte (einschließlich des Glukosespiegels) und die eingehenden Informationen über den Zustand des Körpers. Zu den Appetithormonen gehören Ghrelin (wird bei leerem Magen ausgeschüttet), Insulin (kontrolliert den Blutzuckerspiegel), Leptin (wird von den Fettzellen ausgeschüttet), Orexin (wird vom Hypothalamus ausgeschüttet) und PYY (wird vom Verdauungstrakt ausgeschüttet). Der Grundumsatz ist der Ruheumsatz des Körpers in Bezug auf den Energieverbrauch. Der Körper kann einen Sollwert haben (eine biologisch festgelegte Tendenz, ein optimales Gewicht zu halten) oder einen freieren Einstellwert (der auch von der Umwelt beeinflusst wird).

 

12.3 – Welche kulturellen und situativen Faktoren beeinflussen Hunger?

Hunger spiegelt auch unsere Erinnerung daran wider, wann wir zuletzt gegessen haben, und unsere Erwartung, wann wir erneut etwas essen sollten. Der Mensch als Spezies bevorzugt bestimmte Geschmacksrichtungen (z. B. süß und salzig), aber unsere individuellen Vorlieben werden auch durch Konditionierung, Kultur und Situation beeinflusst. Einige Geschmackspräferenzen haben einen Überlebenswert.

 

12.4 – Wie wirkt sich Adipositas auf die physische und psychische Gesundheit aus und welche Faktoren spielen beim Gewichtsmanagement eine Rolle?

Fettleibigkeit (Adipositas), definiert durch einen Body-Mass-Index (BMI) von 30 oder mehr, wird mit vermehrten Depressionen (insbesondere bei Frauen), Mobbing und vielen körperlichen Gesundheitsrisiken in Verbindung gebracht. Gene und Umwelt wirken bei der Entstehung von Fettleibigkeit zusammen. Die Speicherung von Fett war für unsere Vorfahren eine adaptive Eigenschaft, und Fett erfordert eine geringere Nahrungsaufnahme, um es zu erhalten, als wenn es aufgebaut werden soll. Sollwert und Stoffwechsel spielen eine Rolle; schlanke Menschen neigen dazu, sich mehr zu bewegen. Zwillings- und Adoptionsstudien zeigen, dass das Körpergewicht auch genetisch beeinflusst ist. Zu den Umwelteinflüssen gehören Schlafmangel, soziale Einflüsse sowie das Ernährungs- und Aktivitätsniveau. Denjenigen, die abnehmen wollen, wird geraten, ihre Gewohnheiten ein Leben lang zu ändern: Beginnen Sie erst mit dem Abnehmen, wenn Sie sich motiviert und diszipliniert fühlen; treiben Sie Sport und schlafen Sie ausreichend; setzen Sie sich möglichst wenig verlockenden Nahrungsmitteln aus; begrenzen Sie die Auswahl und essen Sie gesunde Lebensmittel; verkleinern Sie die Portionsgrößen; verteilen Sie die Mahlzeiten über den Tag; hüten Sie sich vor Heißhungerattacken; planen Sie im Voraus, um das Essen bei gesellschaftlichen Anlässen zu kontrollieren; verzeihen Sie gelegentliche Ausrutscher; halten Sie Ihre Fortschritte öffentlich fest und schließen Sie sich einer Selbsthilfegruppe an.

 

12.5 – Wie beeinflussen Hormone die menschliche sexuelle Motivation?

Für alle außer den Menschen, die als asexuell gelten, haben Partnersuche und Paarung ab der Pubertät eine hohe Priorität. Die weiblichen Östrogen- und männlichen Testosteronhormone beeinflussen das menschliche Sexualverhalten weniger direkt als bei anderen Spezies. Die Sexualität von Frauen reagiert stärker auf den Testosteronspiegel als auf den Östrogenspiegel. Kurzfristige Schwankungen des Testosteronspiegels sind bei Männern normal, teilweise als Reaktion auf Stimulation.

 

12.6 – Was ist der sexuelle Reaktionszyklus beim Menschen und wie unterscheiden sich sexuelle Funktionsstörungen von Paraphilien?

William Masters und Virginia Johnson beschrieben vier Phasen im menschlichen sexuellen Reaktionszyklus: Erregung, Plateau, Orgasmus und Entspannung. Während der Entspannung gibt es eine Refraktärphase, in der eine erneute Erregung und ein Orgasmus unmöglich sind. Sexuelle Funktionsstörungen sind Probleme, die die sexuelle Erregung oder das sexuelle Funktionieren dauerhaft beeinträchtigen. Dazu gehören Erektionsstörungen bei Männern und Orgasmusstörungen bei Frauen, die oft erfolgreich durch verhaltenstherapeutische Maßnahmen oder medikamentöse Behandlung behandelt werden können. Paraphilien gelten als Störung, wenn eine Person durch ein ungewöhnliches sexuelles Interesse in Bedrängnis gerät oder wenn dieses Interesse Schaden oder die Gefahr von Schaden für andere mit sich bringt.

 

12.7 – Wie lassen sich sexuell übertragbare Krankheiten verhindern?

Sichere Sexualpraktiken tragen dazu bei, sexuell übertragbare Infektionen zu verhindern. Kondome sind besonders wirksam bei der Verhinderung der Übertragung von HIV, dem Virus, welches AIDS verursacht. Eine vor dem Sexualkontakt verabreichte Impfung kann die meisten Infektionen mit dem Papillomavirus verhindern.

 

12.8 – Wie tragen äußere und in der Fantasie vorgestellte Reize zur sexuellen Erregung bei?

Äußere Reize können sowohl bei Männern als auch bei Frauen sexuelle Erregung auslösen. Sexuell explizites Bildmaterial kann dazu führen, dass Menschen ihre Partner als vergleichsweise weniger attraktiv wahrnehmen und ihre Beziehungen abwerten. Die Betrachtung von Inhalten mit sexuellem Zwang kann zu einer erhöhten Akzeptanz von Gewalt gegen Frauen führen. Der ausgiebige Konsum von Internetpornografie kann junge Männer gegenüber normaler Sexualität desensibilisieren, was zu Erektionsproblemen und vermindertem sexuellen Verlangen im wirklichen Leben führen kann. Auch imaginäre Reize (Träume und Fantasien) beeinflussen die sexuelle Erregung.

 

12.9 – Welche Faktoren beeinflussen Teenagerschwangerschaften und die Nutzung von Verhütungsmitteln bei Jugendlichen?

Die Sexualität von Jugendlichen ist von Kultur zu Kultur und von Epoche zu Epoche unterschiedlich. Zu den Faktoren, die zu diesen Unterschieden beitragen, gehören die Kommunikation über Empfängnisverhütung, Impulsivität, Alkoholkonsum und Massenmedien. Eine hohe Intelligenz, religiöses Handeln, die Anwesenheit eines Vaters und die Teilnahme am Freiwilligendienst sagen sexuelle Zurückhaltung bei Jugendlichen voraus.

 

12.10 – Was hat uns die Forschung über sexuelle Orientierungen gelehrt?

Sexuelle Orientierung ist eine dauerhafte sexuelle Anziehung zu Angehörigen des eigenen Geschlechts, des anderen Geschlechts oder beider Geschlechter. Etwa 3 bis 4 Prozent der Männer und 2 Prozent der Frauen in Europa und den Vereinigten Staaten bezeichnen sich als ausschließlich homosexuell. Männer neigen dazu, eher auf sexuelle Bilder zu reagieren, in denen ihr bevorzugtes Geschlecht vorkommt (bei Heterosexuellen sind es Bilder von Frauen); Frauen reagieren eher unspezifisch auf Bilder von Männern und Frauen. Es gibt keine Hinweise darauf, dass Umwelteinflüsse die sexuelle Orientierung bestimmen. Zu den Belegen für biologische Einflüsse gehören gleichgeschlechtliche Anziehung bei anderen Spezies, Unterschiede zwischen homosexuellen und heterosexuellen Merkmalen und Gehirnen, genetische Einflüsse und pränatale Einflüsse.

 

12.11 – Welche Rolle spielen soziale Faktoren für unsere Sexualität?

Die wissenschaftliche Forschung zur sexuellen Motivation befasst sich nicht mit der persönlichen Bedeutung von Sex in unserem Leben, die von vielen sozialen Faktoren beeinflusst wird. Sex ist ein gesellschaftlich bedeutsamer Akt und ein Ausdruck unserer zutiefst sozialen Natur. Sex in seiner menschlichsten Form ist lebensverbindend und erneuert die Liebe.

 

12.12 – Welchen Beleg gibt es für das menschliche Bedürfnis nach Zugehörigkeit?

Unser Bedürfnis nach Zugehörigkeit – sich mit anderen verbunden und identifiziert zu fühlen – hatte für unsere Vorfahr:innen Überlebenswert, was erklären könnte, warum Menschen in jeder Gesellschaft in Gruppen leben. Soziale Bindungen tragen dazu bei, dass wir gesünder und glücklicher sind, und das Gefühl, geliebt zu werden, aktiviert Gehirnregionen, die mit Belohnungs- und Sicherheitsaspekten verbunden sind. Ausgrenzung ist der absichtliche Ausschluss von Einzelpersonen oder Gruppen. Soziale Isolation kann uns geistig und körperlich schwächen.

 

12.13 – Wie beeinflussen uns soziale Netzwerke?

Über soziale Netzwerke knüpfen wir Kontakte zu anderen und stärken unsere Beziehungen zu denjenigen, die wir bereits kennen. Beim Networking neigen die Menschen dazu, mehr von sich preiszugeben. Menschen mit ausgeprägtem Narzissmus sind in sozialen Netzwerken besonders aktiv. Die Erarbeitung von Strategien zur Selbstkontrolle und disziplinierten Nutzung kann Menschen helfen, ein gesundes Gleichgewicht zwischen ihrer realen und ihrer online verbrachten Zeit zu bewahren.

 

12.14 – Was ist Leistungsmotivation?

Leistungsmotivation ist der Wunsch nach bedeutenden Leistungen, nach der Beherrschung von Fähigkeiten oder Ideen, nach Kontrolle und nach dem Erreichen eines hohen Standards. Eine hohe Leistungsmotivation führt zu größerem Erfolg, vor allem, wenn sie mit entschlossenem, beständigem Durchhaltevermögen kombiniert wird.

 

 

zurück zur Inhaltsübersicht

 

Kapitel 13 - Emotionen, Stress und Gesundheit

 

13.1 – Welches Wechselspiel zwischen Erregung, Ausdrucksverhalten und Kognition gibt es bei Emotionen?

Emotionen sind Reaktionen des gesamten Organismus, die physiologische Erregung, Ausdrucksverhalten und bewusstes Erleben umfassen. Emotionstheorien befassen sich im Allgemeinen mit zwei wichtigen Fragen: (1) Kommt die physiologische Erregung vor oder nach den Emotionen? Und (2) wie interagieren Emotion und Kognition? Die James-Lange-Theorie besagt, dass Emotionen der Reaktion unseres Körpers auf emotionsauslösende Reize folgen (wir beobachten unser Herzklopfen und fühlen Angst). Die Cannon-Bard-Theorie besagt, dass unsere physiologische Reaktion auf einen emotionsauslösenden Stimulus zur gleichen Zeit erfolgt wie unser subjektives Gefühl der Emotion (das eine verursacht das andere nicht).

 

13.2 – Müssen wir Emotionen bewusst interpretieren und benennen, um sie erleben zu können?

Die Zwei-Faktoren-Theorie von Schachter-Singer besagt, dass unsere Emotionen zwei Komponenten umfassen: körperliche Erregung und eine kognitive Bewertung; die kognitiven Bewertungen, die wir unseren Erregungszuständen geben, sind ein wesentlicher Bestandteil der Emotion. Lazarus stimmte zu, dass viele wichtige Emotionen aus unseren Interpretationen oder Schlussfolgerungen entstehen. Zajonc und LeDoux haben jedoch behauptet, dass einige einfache emotionale Reaktionen sofort auftreten, nicht nur außerhalb unseres Bewusstseins, sondern bevor irgendeine kognitive Verarbeitung stattfindet. Dieses Zusammenspiel zwischen Emotion und Kognition veranschaulicht unseren zweigleisigen Verstand.

 

13.3 – Was ist das Bindeglied zwischen emotionaler Erregung und dem autonomen Nervensystem?

Die Erregungskomponente der Emotionen wird von den Teilsystemen des autonomen Nervensystems, dem Sympathikus (Erregung) und dem Parasympathikus (Beruhigung), gesteuert. In einer Krise mobilisiert die „Kampf-oder-Flucht“-Reaktion Ihren Körper automatisch zum Handeln.

 

13.4 – Rufen verschiedene Emotionen auch verschiedene physiologische Reaktionen und Hirnaktivitätsmuster hervor?

Die weitreichenden körperlichen Veränderungen, die mit Angst, Wut und sexueller Erregung einhergehen, sind einander sehr ähnlich (vermehrtes Schwitzen, Atmung und Herzfrequenz), obwohl sie sich unterschiedlich anfühlen. Emotionen können ähnlich erregend sein, aber einige subtile physiologische Reaktionen, wie die Bewegungen der Gesichtsmuskeln, unterscheiden sie voneinander. Bedeutsamere Unterschiede wurden bei der Aktivität in einigen Hirnbahnen und kortikalen Bereichen festgestellt.

 

13.5 – Wie effektiv können Lügendetektoren (Polygrafen) Lügen anhand körperlicher Zustände erkennen?

Polygraphen (Lügendetektoren) versuchen, verschiedene physiologische Indikatoren für Emotionen zu messen, sind aber nicht präzise genug, um einen umfassenden Einsatz in der Wirtschaft und bei der Strafverfolgung zu rechtfertigen. Die Verwendung von Fragen zum Schuldbewusstsein und neue Formen der Technologie können bessere Hinweise auf eine Lüge liefern.

 

13.6 – Wie kommunizieren wir nonverbal?

Ein Großteil unserer Kommunikation erfolgt über Körperbewegungen, Gesichtsausdruck und Stimmlage. Selbst einige Sekunden lang gefilmte Ausschnitte aus dem Verhalten können Gefühle offenbaren.

 

13.7 – Wie unterscheiden sich die Geschlechter in ihrer Fähigkeit nonverbal zu kommunizieren?

Frauen neigen dazu, emotionale Anzeichen leichter zu erkennen und empathischer zu sein. Sie äußern auch mehr Emotionen.

 

13.8 – Wie werden Gestik und Mimik in einer Kultur und über kulturelle Grenzen hinaus verstanden?

Die Bedeutung von Gesten variiert je nach Kultur, aber Gesichtsausdrücke wie Freude und Traurigkeit sind auf der ganzen Welt ähnlich. Die Kulturen unterscheiden sich auch darin, wie sehr sie ihre Emotionen ausdrücken.

 

13.9 – Wie beeinflusst unser Gesichtsausdruck unsere Gefühle?

Forschungsarbeiten zum Gesichtsfeedback-Effekt zeigen, dass unsere Mimik emotionale Gefühle auslösen kann und unserem Körper signalisiert, entsprechend zu reagieren. Wir ahmen auch die Mimik anderer nach, was uns hilft, uns in andere einzufühlen. Ein ähnlicher Effekt der Verhaltensrückkopplung ist die Neigung, unsere eigenen Gedanken, Gefühle und Handlungen und die der anderen durch unser Verhalten zu beeinflussen.

 

13.10 – Welche Grundemotionen gibt es?

Die 10 grundlegenden Emotionen von Carroll Izard sind Freude, Begeisterung, Überraschung, Traurigkeit, Wut, Abscheu, Verachtung, Angst, Scham und Schuld.

 

13.11 – Was sind die Auslöser und Folgen von Wut?

Wut wird am häufigsten durch Fehlverhalten hervorgerufen, das wir als vorsätzlich, ungerechtfertigt und vermeidbar ansehen. Aber auch kleinere Frustrationen und unverschuldete Unannehmlichkeiten können Wut auslösen. Chronische Feindseligkeit ist eine der wichtigsten negativen Emotionen, die mit Herzkrankheiten in Verbindung gebracht werden. Emotionale Abregung kann zwar vorübergehend beruhigend wirken, aber sie reduziert die Wut nicht; wenn wir unsere Wut ausdrücken, können wir noch wütender werden. Fachleute empfehlen, das physiologische Erregungsniveau der Wut zu reduzieren, indem man wartet, sich eine gesunde Ablenkung oder Unterstützung sucht und versucht, sich mental von der Situation zu distanzieren. Kontrolliertes Ausdrücken von Gefühlen kann Konflikte lösen, und Vergebung kann uns von wütenden Gefühlen befreien.

 

13.12 – Was ist das Phänomen „Fühl dich gut, und du tust etwas Gutes“ und wo liegt der Schwerpunkt der Forschung in der Positiven Psychologie?

Glückliche Menschen sind in der Regel gesünder, energiegeladener und zufriedener mit ihrem Leben, so dass sie eher bereit sind, anderen zu helfen (das Wohlfühlphänomen). Die positive Psychologie setzt wissenschaftliche Methoden ein, um das menschliche Wohlbefinden zu erforschen, um Stärken und Fähigkeiten zu entdecken und zu fördern, die dem Einzelnen und der Gemeinschaft helfen, zu einem erfolgreichen Leben zu gelangen.

 

13.13 – Inwieweit sind Zeit, Reichtum, Anpassung und Vergleich Gradmesser unseres Glücks?

Die Stimmungen, die durch gute oder schlechte Ereignisse ausgelöst werden, halten selten über den Tag hinaus an. Selbst bedeutsame gute Ereignisse, wie plötzlicher Reichtum, erhöhen das Glück selten für lange Zeit. Glück ist relativ zu unseren eigenen Erfahrungen (das Phänomen der Anpassungsebene) und zum Erfolg anderer (das Prinzip der relativen Deprivation).

 

13.14 – Was determiniert das Glück, und wie können wir glücklicher werden?

Manche Menschen sind aufgrund ihrer genetischen Veranlagung und ihrer persönlichen Erfahrungen glücklicher als andere. Auch die Kultur, die sich in den von ihr geschätzten Eigenschaften und den von ihr erwarteten und belohnten Verhaltensweisen unterscheidet, beeinflusst das persönliche Glücksniveau. Tipps zur Steigerung des Glücksniveaus: Nehmen Sie Ihren Zeitplan selbst in die Hand, verhalten Sie sich so, als wären Sie glücklich, suchen Sie nach einer sinnvollen Arbeit und Freizeitbeschäftigung, investieren Sie eher in gemeinsame Erlebnisse als in Dinge, treiben Sie Sport, schlafen Sie ausreichend, pflegen Sie Freundschaften, konzentrieren Sie sich nicht nur auf sich selbst, und entwickeln Sie Dankbarkeit und Spiritualität.

 

13.15 – Wie beeinflusst unsere Bewertung eines Ereignisses unsere Stressreaktion und was sind die drei wichtigsten Arten von Stressoren?

Stress ist der Prozess, durch den wir Faktoren, die uns herausfordern oder bedrohen, einschätzen und auf sie reagieren. Wenn wir ein Ereignis als Herausforderung einschätzen, sind wir erregt und konzentrieren uns auf den Erfolg; wenn wir es als Bedrohung empfinden, erleben wir eine Stressreaktion, und unsere Gesundheit kann darunter leiden. Die drei wichtigsten Arten von Stressoren sind Katastrophen, einschneidende Lebensveränderungen sowie alltägliche Probleme inklusive sozialem Stress.

 

13.16 – Wie reagieren wir auf Stress und wie können wir uns an ihn anpassen?

Walter Cannon betrachtete die Stressreaktion als ein Kampf-oder-Flucht-System. Hans Selye schlug ein allgemeines Anpassungssyndrom mit drei Phasen (Alarm, Widerstand, Erschöpfung) vor. Angesichts von Stress können Frauen eine „tend-and-befriend“-Reaktion zeigen; Männer können sich sozial zurückziehen, sich dem Alkohol zuwenden oder gefühlsmäßig abstumpfen.

 

13.17 – Wodurch macht uns Stress anfälliger für Krankheiten?

Die Psychoneuroimmunologie befasst sich mit der Frage, wie psychologische, neuronale und endokrine Prozesse das Immunsystem und die daraus resultierende Gesundheit beeinflussen. Stress entzieht dem Immunsystem Kraft und hemmt die Aktivitäten der B- und T-Lymphozyten, Makrophagen und NK-Zellen. Stress verursacht nicht die Entstehung von Krankheiten, aber durch die Veränderung unserer Immunfunktion kann er uns anfälliger für Krankheiten machen und deren Verlauf beeinflussen.

 

13.18 – Warum neigen manche Menschen mehr zu koronaren Herzkrankheiten als andere?

Koronare Herzkrankheiten werden mit der reaktiven, zu Ärger neigenden Typ-A-Persönlichkeit in Verbindung gebracht. Im Vergleich zu entspannten, gelassenen Typ-B-Persönlichkeiten, bei denen die Wahrscheinlichkeit einer Herzerkrankung geringer ist, schütten Menschen vom Typ A mehr Stresshormone aus. Chronischer Stress trägt auch zu anhaltenden Entzündungsprozessen bei, die mit Herz- und anderen Gesundheitsproblemen, einschließlich Depressionen, in Verbindung gebracht werden.

 

13.19 – Kann Stress also krank machen?

Stress ist zwar nicht direkt krankheitsauslösend, aber er macht uns anfälliger für Krankheiten, indem er unsere Verhaltensweisen und unsere Physiologie beeinflusst.

 

13.20 – Auf welche zwei Arten versuchen Menschen Stress abzubauen?

Mit problemorientierter Bewältigung versuchen wir, den Stress oder die Art und Weise, wie wir damit umgehen, zu verändern. Wir verwenden emotionsorientierte Bewältigung, um Stressoren zu vermeiden oder zu ignorieren und emotionale Bedürfnisse im Zusammenhang mit Stressreaktionen zu erfüllen.

 

13.21 – Wie beeinflusst ein wahrgenommener Mangel an Kontrolle die Gesundheit?

Das Gefühl, keine Kontrolle zu haben, führt zu einer Ausschüttung von Hormonen, die die Gesundheit der Menschen gefährden. Die Unfähigkeit, wiederholte aversive Ereignisse zu vermeiden, kann zu erlernter Hilflosigkeit führen. Menschen, die Kontrolle internal attribuieren, erreichen mehr, erfreuen sich einer besseren Gesundheit und sind glücklicher als Menschen, die einen Kontrollzwang external attribuieren.

 

13.22 – Warum ist Selbstkontrolle wichtig, und kann unsere Selbstkontrolle erschöpfen?

Selbstkontrolle erfordert Aufmerksamkeit und Energie, sagt aber eine bessere Gesundheit, ein höheres Einkommen und bessere schulische Leistungen voraus; sie ist ein besserer Prädiktor für zukünftigen akademischen und Lebenserfolg als die Ergebnisse von Intelligenztests. Die Selbstkontrolle variiert im Laufe der Zeit. Die Forschung ist sich uneinig über die Faktoren, die die Selbstkontrolle beeinflussen, aber eine Stärkung der Selbstkontrolle kann zu einem gesünderen, glücklicheren und erfolgreicheren Leben führen.

 

13.23 – Wie wirkt sich eine optimistische Grundeinstellung auf Gesundheit und Langlebigkeit aus?

Studien mit optimistisch eingestellten Menschen zeigen, dass deren Immunsystem stärker ist, ihr Blutdruck als Reaktion auf Stress nicht so stark ansteigt, sie sich schneller von einer Bypass-Operation am Herzen erholen und ihre Lebenserwartung höher ist als die ihrer pessimistischen Mitmenschen.

 

13.24 – Wie begünstigt soziale Unterstützung die Gesundheit?

Soziale Unterstützung fördert die Gesundheit, indem sie uns beruhigt, den Blutdruck und die Stresshormone senkt und eine bessere Immunfunktion fördert. Wir können unseren Stress deutlich reduzieren und unsere Gesundheit verbessern, indem wir Beziehungen aufbauen und pflegen und auch in schwierigen Situationen einen Sinn sehen.

 

13.25 – Wie effektiv ist aerobes Training als Möglichkeit, um mit Stress umzugehen und das Wohlbefinden zu steigern?

Aerobes Training ist eine anhaltende, sauerstoffverbrauchende Aktivität, die die Fitness von Herz und Lunge erhöht. Sie erhöht die Erregung, führt zu Muskelentspannung und ruhigerem Schlaf, regt die Produktion von Neurotransmittern an und verbessert das Selbstbild. Sie kann Depressionen lindern und wird im Erwachsenenalter mit besseren kognitiven Funktionen und einem längeren Leben in Verbindung gebracht.

 

13.26 – Wie können Entspannung und Meditation Stress und Gesundheit beeinflussen?

Es ist erwiesen, dass Entspannung und Meditation Stress abbauen, indem sie die Muskeln entspannen, den Blutdruck senken, das Immunsystem stärken und Ängste sowie Depressionen lindern. Achtsamkeitsmeditation ist eine Übung, bei der man aktuelle Erfahrungen vorurteilsfrei und akzeptierend wahrnimmt. Auch eine Massagetherapie entspannt die Muskeln und lindert Depressionen.

 

13.27 – Was ist der Glaubensfaktor und welche möglichen Erklärungen gibt es für den dadurch beschriebenen Zusammenhang?

Der Glaubensfaktor ist die Feststellung, dass religiös lebende Menschen tendenziell länger leben als solche, die nicht religiös leben. Zu den möglichen Erklärungen gehören intervenierende Variablen wie gesunde Verhaltensweisen, soziale Unterstützung oder positive Emotionen, die häufig bei Menschen zu finden sind, die regelmäßig an Gottesdiensten teilnehmen.

 

 

zurück zur Inhaltsübersicht

 

Kapitel 14 - Sozialpsychologie

 

14.1 – Was erforschen Sozialpsycholog:innen? Wie erklären wir das Verhalten anderer und unser eigenes?

Sozialpsycholog:innen setzen wissenschaftliche Methoden ein, um zu untersuchen, wie Menschen übereinander denken, sich gegenseitig beeinflussen und miteinander in Beziehung treten. Sie untersuchen die sozialen Einflüsse, die erklären, warum ein und dieselbe Person in verschiedenen Situationen unterschiedlich handelt. Wenn wir das Verhalten anderer Personen erklären, begehen wir – vor allem, wenn wir aus einer individualistischen westlichen Kultur kommen – möglicherweise den fundamentalen Attributionsfehler, indem wir den Einfluss der Situation unterschätzen und die Auswirkungen stabiler, dauerhafter Eigenschaften der Person überschätzen. Wenn wir unser eigenes Verhalten erklären, führen wir es eher auf den Einfluss der Situation zurück.

 

14.2 – Wie wirken Einstellungen und Handlungen zusammen?

Die periphere Route der Überzeugung nutzt beiläufige Hinweise (wie z. B. die Unterstützung durch einen Prominenten), um schnelle, aber relativ gedankenlose Änderungen in der Einstellung zu bewirken. Die zentrale Route der Überzeugung hingegen nutzt Beweise und Argumente, um nachdenkliche Reaktionen auszulösen. Wenn andere Einflüsse minimal sind, können Einstellungen, die stabil, spezifisch und leicht abrufbar sind, unsere Handlungen beeinflussen. Handlungen können Einstellungen verändern, wie beim Fuß-in-der-Tür-Phänomen und bei Rollenspielen. Wenn unsere Einstellungen nicht mit unseren Handlungen übereinstimmen, schlägt die Theorie der kognitiven Dissonanz vor, dass wir Spannungen abbauen, indem wir unsere Einstellungen so ändern, dass sie mit unseren Handlungen zusammenpassen.

 

14.3 – Was ist soziale Ansteckung? Wie können Experimente zur Konformität die Macht von sozialem Einfluss demonstrieren?

Soziale Ansteckung (Chamäleoneffekt) – unsere Neigung, das Verhalten, die Mimik, die Körperhaltung, den Tonfall und die Stimmung anderer unbewusst zu imitieren – ist eine Form der Konformität. Soziale Netzwerke dienen als Wege der Übertragung von Stimmungen, sowohl von guten als auch von schlechten. Solomon Asch und andere haben herausgefunden, dass wir unser Verhalten oder Denken am ehesten an eine Gruppennorm anpassen, wenn wir uns inkompetent oder unsicher fühlen, wenn unsere Gruppe aus mindestens drei Personen besteht, wenn alle anderen zustimmen, wenn wir den Status und die Attraktivität der Gruppe bewundern, wenn wir uns noch nicht auf eine bestimmte Reaktion festgelegt haben, wenn wir wissen, dass wir beobachtet werden, und wenn unsere Kultur den Respekt vor sozialen Normen fördert. Wir können uns anpassen, um Zustimmung zu erhalten (normativer sozialer Einfluss) oder weil wir bereit sind, die Ansichten anderer als neue Informationen zu akzeptieren (informativer sozialer Einfluss).

 

14.4 – Was lehrten uns die Experimente von Milgram zum Gehorsam über die Macht von sozialen Einflüssen?

Die Experimente von Stanley Milgram – bei denen Menschen Befehle befolgten, selbst wenn sie annahmen, dass sie einer anderen Person Schaden zufügten – zeigten, dass starke soziale Einflüsse Menschen dazu bringen können, falsche Erklärungen zu akzeptieren oder Grausamkeiten zuzulassen. Der Gehorsam war am größten, wenn die Person, die die Befehle gab, in der Nähe war und als legitime Autoritätsperson wahrgenommen wurde, wenn die Forschung von einer angesehenen Institution unterstützt wurde, wenn das Opfer entpersonalisiert oder weit entfernt war und wenn es keine Modelle für die Verweigerung der Befehle gab.

 

14.5 – Was können wir aus den Studien zum sozialen Einfluss über uns selbst erfahren? Wie viel Macht haben wir als Individuen?

Diese Experimente haben gezeigt, dass starke soziale Einflüsse Menschen dazu bringen können, Unwahrheiten zu glauben oder gegenüber Grausamkeiten zu kapitulieren. Die Macht des Einzelnen (persönliche Kontrolle) und die Macht der Situation (soziale Kontrolle) wirken zusammen. Eine kleine Minderheit, die konsequent ihre Meinung vertritt, kann die Mehrheit beeinflussen, ebenso wie ein einzelner engagierter Mensch.

 

14.6 – Wie beeinflusst die Anwesenheit anderer unser Handeln durch soziale Erleichterung, soziales Faulenzen und Deindividuation?

Bei sozialer Erleichterung werden wir durch die bloße Anwesenheit anderer angeregt, so dass sich unsere Leistung bei leichten oder gut erlernten Aufgaben verbessert, bei schwierigen Aufgaben jedoch abnimmt. Bei sozialem Faulenzen fühlen wir uns durch die Gruppenarbeit weniger verantwortlich und nutzen die Bemühungen der anderen als Trittbrettfahrer. Wenn die Anwesenheit anderer uns sowohl erregt als auch das Gefühl gibt, anonym zu sein, kann es zu einer Deindividuation kommen – dem Verlust des Ich-Bewusstseins und der Selbstbeherrschung.

 

14.7 – Wie kann die Interaktion in der Gruppe Gruppenpolarisierung bewirken?

Bei der Gruppenpolarisierung verstärken Gruppendiskussionen mit Gleichgesinnten die vorherrschenden Überzeugungen und Haltungen der Gruppenmitglieder.

 

14.8 – Welche Rolle spielt das Internet bei der Gruppenpolarisierung?

Die Kommunikation über das Internet verstärkt die Wirkung der Verbindung von Gleichgesinnten, im Guten wie im Schlechten. Die Menschen finden Unterstützung, die ihre Ideen stärkt, aber oft auch Isolation von Menschen mit anderen Meinungen. Isolation und Konversation können so zu einer Polarisierung der Gruppe führen.

 

14.9 – Wie kann Gruppeninteraktion Gruppendenken ermöglichen?

Gruppendenken wird durch den Wunsch nach Harmonie innerhalb einer Gruppe, die Entscheidungen trifft, angetrieben und hat Vorrang vor einer realistischen Bewertung von Alternativen. Die Gruppenleitung kann sich die Vorteile der Gruppeninteraktion zunutze machen, indem sie Personen mit der Ermittlung möglicher Probleme beauftragt und verschiedene Meinungen und Kritik von Fachleuten zulässt.

 

14.10 – Was sind Vorurteile? Wie unterscheiden sich explizite und implizite Vorurteile?

Ein Vorurteil ist eine ungerechtfertigte, meist negative Einstellung gegenüber einer Gruppe und ihren Mitgliedern. Die drei Komponenten von Vorurteilen sind Überzeugungen (oft Stereotype), Emotionen und Prädispositionen für Handlungen (Diskriminierung). Explizite (offene) Vorurteile haben in Nordamerika im Laufe der Zeit abgenommen, aber implizite Vorurteile – eine unreflektierte, unbewusste Antwort – sind nach wie vor vorhanden.

 

14.11 – Welche Gruppen sind häufig Ziel von Vorurteilen?

Vorurteile umfassen sowohl explizite als auch implizite negative Einstellungen gegenüber Menschen einer bestimmten ethnischen Gruppe, eines bestimmten Geschlechts, einer bestimmten sexuellen Ausrichtung oder eines bestimmten Glaubens.

 

14.12 – Was sind einige soziale, emotionale und kognitive Wurzeln von Vorurteilen, und welche Möglichkeiten gibt es, Vorurteile zu beseitigen?

Zu den sozialen Hintergründen von Vorurteilen gehören soziale Ungleichheiten und Trennungen. Gruppen mit höherem Status rechtfertigen ihre privilegierte Stellung oft mit dem Phänomen der gerechten Welt. Wir neigen dazu, unsere eigene Gruppe zu bevorzugen (Ingroup Bias), da wir zwischen „uns“ (der Ingroup) und „ihnen“ (der Outgroup) unterscheiden. Vorurteile können auch ein Mittel sein, um unser emotionales Wohlbefinden zu schützen, beispielsweise wenn wir unsere Wut auf einen Sündenbock schieben können. Die kognitiven Ursprünge von Vorurteilen ergeben sich aus unserer natürlichen Informationsverarbeitung: Wir bilden Kategorien, erinnern uns an eindrückliche Fälle und glauben, dass die Welt gerecht sei (und dass unsere eigene Vorgehensweise und die unserer Gruppe die richtige sei). Die Beobachtung unserer Gefühle und Handlungen sowie die Entwicklung neuer Freundschaften können uns helfen, uns von unseren Vorurteilen zu lösen.

 

14.13 – Inwiefern unterscheidet sich die psychologische Definition von Aggression von der Verwendung des Begriffs im Alltag? Welche biologischen Faktoren bringen uns dazu, anderen wehzutun?

In der spezifischeren Bedeutung der Psychologie ist Aggression jede Handlung, die darauf abzielt, jemandem körperlich oder seelisch zu schaden. Unsere Schwelle für aggressives Verhalten wird von der Biologie auf drei Ebenen beeinflusst: genetisch (vererbte Eigenschaften), neuronal (Aktivität in Schlüsselregionen des Gehirns) und biochemisch (z. B. Alkohol oder Testosteronüberschuss in der Blutbahn). Aggression ist ein komplexes Verhalten, das sich aus dem Zusammenspiel von Biologie und Erfahrung ergibt.

 

14.14 – Welche psychologischen und soziokulturellen Faktoren können aggressives Verhalten auslösen?

Frustration (das Frustrations-Aggressions-Prinzip), vorherige Verstärkung für aggressives Verhalten, die Beobachtung aggressiver Modelle und mangelnde Selbstkontrolle tragen allesamt zur Aggression bei. Gewalt in den Medien liefert soziale Skripte, an denen sich Kinder orientieren lernen. Der Anblick von sexueller Gewalt trägt zu größerer Aggression gegenüber Frauen bei. Das Spielen von gewalttätigen Videospielen verstärkt aggressive Gedanken, Gefühle und Verhaltensweisen.

 

14.15 – Wieso freunden wir uns mit bestimmten Personen an oder verlieben uns in sie, aber nicht in andere?

Nähe (räumlich) erhöht die Sympathie, zum Teil aufgrund des bloßen Expositionseffekts die Exposition gegenüber neuen Reizen erhöht die Sympathie für diese Reize. Körperliche Attraktivität erhöht die sozialen Chancen und verbessert die Art und Weise, wie wir wahrgenommen werden. Ähnliche Einstellungen und Interessen erhöhen die Sympathie erheblich, insbesondere wenn sich Beziehungen entwickeln. Wir mögen auch diejenigen, die uns mögen.

 

14.16 – Wie verändert sich romantische Liebe im Laufe der Zeit?

Intime Liebesbeziehungen beginnen mit leidenschaftlicher Liebe einem Zustand intensiver Erregung. Mit der Zeit kann sich die starke Zuneigung der partnerschaftlichen Liebe entwickeln, insbesondere wenn sie durch eine partnerschaftliche Beziehung, intime Selbstauskünfte und positive Unterstützung gefördert wird.

 

14.17 – Was ist Altruismus? Wann ist es am wahrscheinlichsten bzw. am unwahrscheinlichsten, dass Menschen helfen?

Altruismus ist die uneigennützige Rücksichtnahme auf das Wohl der anderen Menschen. Wir helfen am ehesten, wenn wir einen Zwischenfall bemerken, ihn als Notfall interpretieren und die Verantwortung für die Hilfsmaßnahmen übernehmen. Andere Faktoren, wie unsere Stimmung und unsere Ähnlichkeit mit dem Opfer, beeinflussen ebenfalls unsere Bereitschaft zu helfen. Die Wahrscheinlichkeit, dass wir helfen, ist am geringsten, wenn andere Zuschauende anwesend sind (Bystander-Effekt).

 

14.18 – Wie erklären die Theorie des sozialen Austauschs und soziale Normen Hilfeverhalten?

Die Theorie des sozialen Austauschs geht davon aus, dass wir anderen helfen, weil es in unserem eigenen Interesse liegt; nach dieser Auffassung besteht das Ziel des sozialen Verhaltens darin, den persönlichen Nutzen zu maximieren und die Kosten zu minimieren. Andere glauben, dass Helfen aus der Sozialisation resultiert, in der uns Richtlinien für das erwartete Verhalten in sozialen Situationen beigebracht werden, wie etwa die Reziprozitätsnorm und die Norm der sozialen Verantwortung.

 

14.19 – Wie können soziale Fallen und spiegelbildliche Wahrnehmung soziale Konflikte anheizen?

Soziale Fallen sind Situationen, in denen Menschen in Konflikten ihre eigenen individuellen Interessen verfolgen und damit dem kollektiven Wohl schaden. Einzelpersonen und Kulturen in Konflikten neigen auch dazu, spiegelbildliche Wahrnehmungen zu entwickeln: Jede Partei sieht die andere als unzuverlässig und böswillig an und sich selbst als ethisches, friedliches Opfer. Wahrnehmungen können zu einer sich selbst erfüllenden Prophezeiung werden.

 

14.20 – Was können wir tun, um Frieden zu fördern?  

Frieden kann entstehen, wenn Einzelpersonen oder Gruppen sich gemeinsam für die Erreichung übergeordneter (gemeinsamer) Ziele einsetzen. Die Forschung zeigt, dass Kontakt, Kooperation, Kommunikation und Aussöhnung den Frieden fördern.

 

 

zurück zur Inhaltsübersicht

 

Kapitel 15 - Persönlichkeit

 

15.1 – Was ist Persönlichkeit und welche Theorien formen unser Verständnis von Persönlichkeit?

Die Persönlichkeit ist das charakteristische Muster des Denkens, Fühlens und Handelns einer Person. Die psychoanalytische (und später die psychodynamische) Theorie und die humanistische Theorie sind Teil unseres kulturellen Erbes geworden. Außerdem legten sie den Grundstein für spätere Theorien, wie die Persönlichkeitstheorie und die sozial-kognitive Theorie.

 

15.2 – Wie führten Sigmund Freuds Behandlungen psychischer Störungen zu seiner Sichtweise über das Unbewusste?

Die psychodynamischen Theorien betrachten die Persönlichkeit aus der Perspektive, dass das Verhalten eine dynamische Interaktion zwischen dem bewussten und dem unbewussten Teil des Geistes ist. Diese Theorien gehen auf die Theorie der Psychoanalyse von Sigmund Freud zurück. Bei der Behandlung von Menschen, deren Störungen keine eindeutige körperliche Erklärung hatten, kam Freud zu dem Schluss, dass diese Probleme inakzeptable Gedanken und Gefühle widerspiegeln, die im Unbewussten verborgen sind. Um diesen unbewussten Teil der Psyche eines Menschen zu erforschen, setzte Freud die freie Assoziation und die Traumanalyse ein.

 

15.3 – Wie betrachtete Freud Persönlichkeit?

Freud ging davon aus, dass die Persönlichkeit aus einem Konflikt resultiert, der sich aus der Interaktion zwischen den drei psychischen Systemen ergibt: dem Es („lustorientierte Triebe“), dem Ich („realitätsorientierte Exekutive“) und dem Über-Ich („verinnerlichte Ideale“ oder „Gewissen“).

 

15.4 – Von welchen Entwicklungsphasen ging Freud aus?

Freud glaubte, dass Kinder fünf psychosexuelle Phasen durchlaufen (oral, anal, phallisch, Latenzphase und genital). Nach dieser Auffassung können ungelöste Konflikte in jeder Phase dazu führen, dass die lustbetonten Triebe einer Person in dieser Phase fixiert („abgewürgt“) werden.

 

15.5 – Wie wehren sich Menschen laut Freud gegen Angst?

Für Freud war die Angst das Ergebnis von Spannungen zwischen den Anforderungen des Es und des Über-Ichs. Das Ich bewältigt diese Spannungen mit Hilfe unbewusster Abwehrmechanismen wie der Verdrängung, die er als den grundlegenden Mechanismus ansah, der allen anderen zugrunde liegt und diese erst ermöglicht.

 

15.6 – Welche seiner Ideen verwarfen bzw. akzeptierten Freuds Anhänger:innen?

Die frühen Anhänger:innen Freuds, die Neo-Freudianer:innen, übernahmen viele seiner Ideen. Sie unterschieden sich darin, dass sie den Schwerpunkt stärker auf das Bewusstsein legten und soziale Motive stärker betonten als Sex oder Aggression. Die meisten gegenwärtigen psychodynamischen Theoretiker:innen und Therapeut:innen lehnen Freuds Betonung der sexuellen Motivation ab. Sie betonen, unterstützt von aktuellen Forschungsergebnissen, dass ein Großteil unseres Seelenlebens unbewusst abläuft, und sie glauben, dass unsere Kindheitserfahrungen unsere Persönlichkeit und unsere Bindungsmuster im Erwachsenenalter beeinflussen. Viele glauben auch, dass die gemeinsame Evolutionsgeschichte der Menschen einige universelle Veranlagungen geprägt hat.

 

15.7 – Was sind projektive Tests, wie werden sie eingesetzt und welche Kritikpunkte gibt es an diesen?

Mit projektiven Tests wird versucht, die Persönlichkeit zu beurteilen, indem Personen Reizmaterial gezeigt wird, das viele mögliche Interpretationen zulässt, und ihre Antworten als Hinweise auf unbewusste Motive betrachtet werden. Der Thematische Apperzeptionstest (TAT) und der Rorschach-Tintenklecks-Test sind zwei solcher Tests. Der TAT liefert eine valide und reliable Darstellung der impliziten Motive von Menschen, die über die Zeit hinweg konsistent ist. Der Rorschach-Test hat eine geringe Reliabilität und Validität, aber einige Kliniker:innen schätzen ihn als Quelle für suggestive Hinweise, als Kennenlernhilfe („Eisbrecher“) oder als aufschlussreiche Interviewtechnik.

 

15.8 – Wie beurteilt die heutige Psychologie Freuds Psychoanalyse?

Die heutige Psychologie sieht es als ihr Verdienst an, dass Freud die Aufmerksamkeit auf das große Unbewusste, auf den Kampf mit Angst und Sexualität, auf den Konflikt zwischen biologischen Impulsen und sozialen Zwängen und auf einige Formen von Abwehrmechanismen gelenkt hat. Doch sein Konzept der Verdrängung und seine Auffassung vom Unbewussten als einer Ansammlung von verdrängten und inakzeptablen Gedanken, Wünschen, Gefühlen und Erinnerungen halten einer wissenschaftlichen Überprüfung nicht stand. Freud bot nachträgliche Erklärungen an, die wissenschaftlich schwer zu überprüfen sind. Die Forschung stützt viele von Freuds spezifischen Ideen nicht, wie etwa die Ansicht, dass die Entwicklung in der Kindheit festgelegt wird. (Wir wissen heute, dass Entwicklung ein Leben lang andauert.)

 

15.9 – Wie hat die moderne Forschung unser Verständnis des Unbewussten geprägt?

Die Forschung bestätigt, dass wir keinen vollständigen Zugang zu allem haben, was in unserem Kopf vor sich geht, obwohl die heutige Wissenschaft das Unbewusste als eine separate und parallele Spur der Informationsverarbeitung betrachtet, die außerhalb unseres Bewusstseins stattfindet. Zu dieser Verarbeitung gehören Schemata, die unsere Wahrnehmungen steuern, Priming, implizite Erinnerungen an erlernte Fähigkeiten, sofort aktivierte Emotionen und Stereotypen, die unsere Informationsverarbeitung von Eigenschaften und Merkmalen anderer filtern. Die Forschung unterstützt auch die Reaktionsbildung und Projektion (der Effekt des falschen Konsenses) und die Vorstellung, dass wir uns unbewusst vor Angst schützen (wie in Experimenten zur Terror-Management-Theorie gezeigt wurde).

 

15.10 – Was verstanden humanistische Psycholog:innen unter Persönlichkeit und welches Ziel verbarg sich hinter deren Erforschung von Persönlichkeit?

Die Sichtweise der humanistischen Psycholog:innen auf die Persönlichkeit konzentriert sich auf das Potenzial für gesundes persönliches Wachstum und das Streben des Menschen nach Selbstbestimmung und Selbstverwirklichung. Abraham Maslow schlug vor, dass die menschlichen Bedürfnisse eine Hierarchie bilden; wenn die Grundbedürfnisse erfüllt sind, streben die Menschen nach Selbstverwirklichung und Selbsttranszendenz. Carl Rogers glaubte, dass ein wachstumsförderndes Umfeld aus Akzeptanz (einschließlich bedingungsloser positiver Wertschätzung), Aufrichtigkeit und Empathie besteht. Das Selbstkonzept war sowohl für Maslow als auch für Rogers ein zentrales Merkmal der Persönlichkeit.

 

15.11 – Wie erfassten humanistische Psycholog:innen die Selbstwahrnehmung einer Person?

Einige humanistische Psycholog:innen lehnten jede standardisierte Beurteilung ab und verließen sich auf Interviews und Gespräche. Rogers verwendete gelegentlich Fragebögen, in denen die Menschen ihr ideales und tatsächliches Selbst beschrieben, was er später zur Beurteilung des Therapiefortschritts nutzte.

 

15.12 – Inwiefern haben humanistische Theorien die Psychologie beeinflusst und welches sind die Kritikpunkte an ihnen?

Die humanistische Psychologie trug dazu bei, das Interesse am Konzept des Selbst zu erneuern, und legte auch den Grundstein für den heutigen wissenschaftlichen Teilbereich der positiven Psychologie. Kritische Stimmen sagen, die Konzepte der humanistischen Psychologie seien vage und subjektiv, ihre Werte egozentrisch und ihre Annahmen naiv optimistisch.

 

15.13 – Wie bedienen sich Psycholog:innen sogenannter Traits, um Persönlichkeit zu beschreiben?

Eigenschaftstheoretiker:innen sehen die Persönlichkeit als ein stabiles und dauerhaftes Verhaltensmuster an. Sie sind mehr daran interessiert, unsere individuellen Besonderheiten zu beschreiben, als sie zu erklären. Mithilfe der Faktorenanalyse ermitteln sie Cluster von Verhaltenstendenzen, die gemeinsam auftreten. Genetische Veranlagungen beeinflussen viele Merkmale.

 

15.14 – Was sind einige verbreitete Missverständnisse über Introversion?

In westlichen Kulturen wird Extravertiertheit geschätzt, aber auch introvertierte Menschen haben andere, ebenso wichtige Fähigkeiten. Introvertiertheit ist nicht gleichbedeutend mit Schüchternheit, und Extravertierte sind nicht immer besser als Introvertierte in Führungspositionen oder im Verkaufserfolg. Introvertierte erzielen oft große Leistungen; viele Introvertierte sind erfolgreich.

 

15.15 – Was sind Persönlichkeitsinventare und was sind deren Stärken und Schwächen in ihrer Funktion als Werkzeuge zur Erfassung von Traits?

Persönlichkeitsinventare (wie beispielsweise das MMPI) sind Fragebögen, bei denen Personen auf Items antworten, mit denen ein breites Spektrum von Gefühlen und Verhaltensweisen erfasst werden soll. Die Testaufgaben werden empirisch abgeleitet, und die Tests werden objektiv ausgewertet. Objektivität ist jedoch keine Garantie für Validität; Menschen können ihre Antworten verfälschen, um einen guten Eindruck zu erwecken (sie können dann aber auf einer Lügenskala, die die Verfälschung bewertet, hohe Werte erreichen).

 

15.16 – Welche Traits scheinen die brauchbarste Information über Persönlichkeitsvarianten bereitzustellen?

Die Big-Five-Persönlichkeitsfaktoren – Offenheit, Gewissenhaftigkeit, Extraversion, Verträglichkeit und Neurotizismus (OCEAN) – bieten derzeit die beste Annäherung an die grundlegenden Merkmalsdimensionen. Diese Faktoren sind recht stabil und scheinen in allen Kulturen vorhanden zu sein. Viele Gene, von denen jedes einen kleinen Einfluss hat, beeinflussen unsere Eigenschaften, und die Heritabilität liegt im Allgemeinen bei etwa 40 Prozent je Dimension.

 

15.17 – Wird das Konzept der zeitlichen und situativen Stabilität der Persönlichkeitsmerkmale durch die Forschung gestützt?

Die durchschnittlichen Eigenschaften einer Person bleiben im Laufe der Zeit stabil und sind in vielen verschiedenen Situationen prognostizierbar. Die Eigenschaften können jedoch nicht das Verhalten in einer konkreten Situation vorhersagen.

 

15.18 – Wie interpretiert die sozial-kognitive Perspektive die Persönlichkeitsentwicklung und auf welche Weise untersucht sie das Verhalten?

Albert Bandura schlug als Erster die sozial-kognitive Perspektive vor, die die Wechselwirkungen zwischen unseren Merkmalen und Situationen betont. Sozial-kognitive Forschung wendet Prinzipien des Lernens, der Kognition und des sozialen Verhaltens auf die Persönlichkeit an. Der reziproke Determinismus beschreibt die Interaktion und den gegenseitigen Einfluss von Verhalten, interner Kognition und Umwelt. Bei Bewertungssituationen mit simulierten Bedingungen wird das Prinzip genutzt, dass die Handlungen einer Person in ähnlichen Situationen der beste Prädiktor für zukünftiges Verhalten sind.

 

15.19 – Welche Kritikpunkte wurden am sozial-kognitiven Ansatz geäußert?

Der sozial-kognitive Ansatz baut auf etablierten Konzepten des Lernens und der Kognition auf und sensibilisiert die Forschung für die Art und Weise, wie Situationen Individuen beeinflussen und von ihnen beeinflusst werden. Es wurde bemängelt, dass die Bedeutung unbewusster Motive, Emotionen und biologisch geprägter Eigenschaften unterschätzt wird.

 

15.20 – Wieso hat die Psychologie so viel Forschung über das Selbst hervorgebracht? Wie wichtig ist das Selbstwertgefühl für die Psychologie und das menschliche Wohlbefinden?

Das Selbst ist das Zentrum der Persönlichkeit und organisiert unsere Gedanken, Gefühle und Handlungen. Die Beschäftigung mit dem eigenen Selbst motiviert uns zu einer positiven Entwicklung, aber wenn wir uns zu sehr auf uns selbst konzentrieren, kann dies zu einem Spotlight-Effekt führen. Ein hohes Selbstwertgefühl korreliert mit weniger Anpassungsdruck, mit Ausdauer bei schwierigen Aufgaben und mit Glück. Die Richtung der Korrelation ist jedoch unklar. Anstatt den Selbstwert unrealistisch zu unterstützen, ist es besser, die Leistungen der Kinder zu belohnen und so das Gefühl der Kompetenz zu fördern.

 

15.21 – Auf welche Weise offenbaren exzessiver Optimismus, die Schwierigkeit, unsere eigene Inkompetenz zu erkennen, und selbstwertdienliche Verzerrungen den Preis des Selbstwertgefühls, und wie unterscheidet sich ein defensives Selbstwertgefühl von einem sicheren Selbstwertgefühl?

Übermäßiger Optimismus kann zu Selbstgefälligkeit führen und uns daran hindern, echte Risiken zu erkennen, während die Blindheit gegenüber der eigenen Inkompetenz dazu führen kann, dass wir immer wieder die gleichen Fehler machen. Die Tendenz, uns selbst als besser als der Durchschnitt zu betrachten oder unsere Erfolge zu würdigen, aber unsere Misserfolge nicht zu tadeln, wird als selbstsüchtig bezeichnet (selfserving bias). Ein negatives Selbstwertgefühl ist zerbrechlich, konzentriert sich darauf, sich selbst zu erhalten, und sieht Versagen oder Kritik als Bedrohung an. Ein gefestigtes Selbstwertgefühl gibt uns das Gefühl, so akzeptiert zu werden, wie wir sind.

 

 

zurück zur Inhaltsübersicht

 

Kapitel 16 - Klinische Psychologie: Psychische Störungen

 

16.1 – Wo ist die Grenze zwischen Normalität und Störung?

Nach Ansicht von Psycholog:innen und Psychiater:innen sind psychische Störungen durch eine klinisch signifikante Störung der Kognition, der Emotionsregulation oder des Verhaltens einer Person gekennzeichnet. Solche dysfunktionalen oder maladaptiven Gedanken, Emotionen oder Verhaltensweisen beeinträchtigen das tägliche Leben und sind deshalb dysfunktional.

 

16.2 – Wie erklären das medizinische Modell und der biopsychosoziale Ansatz psychische Störungen?

Das medizinische Modell geht davon aus, dass psychische Störungen körperliche Ursachen haben, die diagnostiziert, behandelt und in den meisten Fällen durch eine bestimmte Therapie, manchmal in einem Krankenhaus, geheilt werden können. Die biopsychosoziale Perspektive geht davon aus, dass gestörtes Verhalten aus dem Zusammenspiel von biologischen Merkmalen, psychologischer Dynamik und soziokulturellen Umständen entsteht. Aus diesem Ansatz ist das Vulnerabilitäts-Stress-Modell hervorgegangen, bei dem individuelle Merkmale und Umweltstressoren zusammenwirken, sodass sich die Wahrscheinlichkeit, eine psychische Störung zu entwickeln, erhöht oder verringert – ein Modell, das von der Epigenetikforschung unterstützt wird.

 

16.3 – Wie und warum klassifizieren Kliniker:innen psychische Störungen, und warum kritisieren einige Psycholog: innen die Verwendung diagnostischer Etikettierungen?

Das DSM-5 (Diagnostic and Statistical Manual of Mental Disorders, Fifth Edition) der American Psychiatric Association enthält diagnostische Bezeichnungen und Beschreibungen, die eine einheitliche Sprache und geteilte Konzepte für Kommunikation und Forschung bieten. Kritiker des DSM sagen, dass es zu breit gefächert sei und normale Verhaltensweisen pathologisiere. Ein ergänzender Klassifizierungsansatz ist das Research Domain Criteria (RDoC)-Projekt des U.S. National Institute of Mental Health, ein Rahmenwerk, das Störungen nach Verhaltensweisen und Gehirnaktivität entlang mehrerer Dimensionen gliedert. Jeder Klassifizierungsversuch führt zu diagnostischen Etiketten, die zu Vorurteilen führen können, die die Wahrnehmung des vergangenen und gegenwärtigen Verhaltens der mit dem Etikett versehenen Person verzerren.

 

16.4 – Warum gibt es eine Kontroverse über die Aufmerksamkeitsdefizit- Hyperaktivitätsstörung?

Bei einem Kind (oder seltener bei einem Erwachsenen), das extreme Unaufmerksamkeit und bzw. oder Hyperaktivität und Impulsivität zeigt, kann eine Aufmerksamkeitsdefizit-Hyperaktivitätsstörung (ADHS) diagnostiziert werden. Die Kontroverse dreht sich um die Frage, ob die steigende Zahl der ADHS-Fälle auf eine Überdiagnostik oder ein gesteigertes Bewusstsein für die Störung zurückzuführen ist, sowie um die langfristigen Auswirkungen der Behandlung mit Stimulanzien.

 

16.5 – Sind psychische Störungen Prädiktoren für gewalttätiges Verhalten?

Psychische Störungen führen nur selten zu Gewalt, aber wenn sie es tun, werfen sie moralische und ethische Fragen darüber auf, ob die Gesellschaft Menschen mit psychischen Störungen für ihre gewalttätigen Handlungen verantwortlich machen sollte. Die meisten Menschen mit Erkrankungen sind nicht gewalttätig und werden eher zu Opfern als zu Täter:innen.

 

16.6 – Wie viele Menschen leiden oder litten an einer psychischen Störung? Ist Armut ein Risikofaktor?

Die Häufigkeit psychischer Störungen variiert je nach Zeitpunkt und Ort der Erhebung. In einer länderübergreifenden Erhebung war die niedrigste Rate an gemeldeten psychischen Störungen in Nigeria (6 %), die höchste Rate in den Vereinigten Staaten (27 %). Armut ist ein Risikofaktor. Aber einige Störungen, wie z. B. Schizophrenie, können Menschen auch in die Armut treiben. Immigrant:innen, die in die Vereinigten Staaten einwandern, weisen im Durchschnitt eine bessere psychische Gesundheit auf als ihre US-Kolleg:innen mit derselben ethnischen Herkunft (ein Phänomen, das als Einwanderungsparadoxon bekannt ist).

 

16.7 – Wie unterscheiden sich generalisierte Angststörung, Panikstörung und Phobien?

Bei Angststörungen handelt es sich um psychische Störungen, die durch belastende, anhaltende Ängste oder unangepasste Verhaltensweisen zur Reduktion von Ängsten gekennzeichnet sind. Menschen mit einer generalisierten Angststörung fühlen sich ohne ersichtlichen Grund anhaltend und unkontrollierbar angespannt und ängstlich. Bei der extremeren Panikstörung eskaliert die Angst in periodischen Episoden intensiver Panik. Menschen mit einer Phobie haben irrationale Angst vor einem bestimmten Objekt, einer Aktivität oder einer Situation.

 

16.8 – Was ist OCD?

Anhaltende und sich wiederholende Gedanken (Obsessionen), Handlungen (Zwänge) oder beides sind kennzeichnend für eine Zwangsstörung (obsessive-compulsive disorder; OCD).

 

16.9 – Was ist PTBS?

Zu den Symptomen einer posttraumatischen Belastungsstörung (PTBS) gehören vier oder mehr Wochen lang quälende Erinnerungen, Albträume, Hypervigilanz, sozialer Rückzug, flottierende Ängste, Gefühllosigkeit und bzw. oder Schlafprobleme nach einem traumatischen Erlebnis.

 

16.10 – Inwiefern tragen Konditionierung, Kognition und Biologie zu den Empfindungen und Gedanken bei, die Angststörungen, Zwangsstörungen und posttraumatische Belastungsstörungen prägen?

Die Lerntheorie betrachtet Angststörungen, Zwangsstörungen und PTBS als Ergebnisse von Angstkonditionierung, Reizgeneralisierung, Verstärkung von Angstverhalten und Beobachtungslernen (in der Regel mit Hypervigilanz) von Ängsten und Kognitionen anderer. Die biologische Perspektive berücksichtigt genetische Prädispositionen für ein hohes Maß an emotionaler Reaktivität und Neurotransmitterproduktion, abnorme Reaktionen in den Angstschaltkreisen des Gehirns und die Rolle, die die Angst vor lebensbedrohlichen Gefahren bei der natürlichen Selektion und Evolution gespielt hat.

 

16.11 – Wie unterscheiden sich Major Depression, persistierende depressive Störung und bipolare Störung?

Eine Person mit einer schweren depressiven Störung leidet zwei oder mehr Wochen lang an mindestens fünf Symptomen einer Depression (einschließlich niedergeschlagener Stimmung oder Verlust von Interesse oder Freude). Eine anhaltende depressive Störung umfasst eine lang anhaltende, leicht depressive Stimmung sowie mindestens zwei weitere Symptome. Eine Person mit der selteneren bipolaren Störung leidet nicht nur an Depressionen, sondern auch an einer Manie, d. h. an hyperaktivem und übertrieben optimistischem, impulsivem Verhalten.

 

16.12 – Wie erklären biologische und sozial-kognitive Ansätze affektive depressive Störungen und die bipolare Störung?

Die biologische Sichtweise zu depressiven Störungen und bipolaren Störungen konzentriert sich auf genetische Prädispositionen, Anomalien in den Gehirnstrukturen und -funktionen (einschließlich derjenigen, die in den Neurotransmittersystemen zu finden sind) sowie auf die Auswirkungen von Ernährung (und Medikamenten). Die sozial-kognitive Perspektive betrachtet Depressionen als einen andauernden Kreislauf von belastenden Erfahrungen (interpretiert durch negative Überzeugungen, Zuschreibungen und Erinnerungen, oft mit unaufhörlichem Grübeln), der zu negativen Stimmungen, Gedanken und Handlungen führt und dadurch neue belastende Erfahrungen fördert.

 

16.13 – Welche Faktoren erhöhen das Risiko für Suizid, und was wissen wir über nicht suizidales selbstverletzendes Verhalten?

Die Suizidraten unterscheiden sich je nach Land, Ethnie, Geschlecht, Altersgruppe, Einkommen, Religionszugehörigkeit, Familienstand und anderen Faktoren. Menschen, denen es an sozialer Unterstützung mangelt, wie z. B. viele homosexuelle, transsexuelle und geschlechtsindifferente Jugendliche, sind einem erhöhten Risiko ausgesetzt, ebenso wie Menschen, die unter Angstzuständen oder Depressionen leiden. Anzeichen für einen Suizid können verbale Andeutungen, das Weggeben von Besitztümern, Rückzug und die Beschäftigung mit dem Tod sein. Menschen, die über Suizid sprechen, sollten ernst genommen werden: Hören Sie zu und zeigen Sie Mitgefühl, vermitteln Sie Hilfe und schützen Sie diejenigen, die unmittelbar gefährdet zu sein scheinen. Nicht-suizidale Selbstverletzung (nonsuicidal self-injury; NSSI) führt in der Regel nicht zu Suizid, kann aber unbehandelt zu Suizidgedanken und -handlungen eskalieren. Menschen, die NSSI begehen, können Stress nicht gut verkraften und neigen dazu, sich selbst zu kritisieren und verfügen über schlechte Kommunikations- und Problemlösungsfähigkeiten.

 

16.14 – Welche Muster des Denkens, Wahrnehmens, Fühlens und Verhaltens charakterisieren die Schizophrenie?

Schizophrenie ist eine psychotische Störung, die durch Wahnvorstellungen, Halluzinationen, desorganisierte Sprache und bzw. oder einen verminderten, unangemessenen emotionalen Ausdruck gekennzeichnet ist. Halluzinationen sind Sinneserfahrungen ohne sensorische Stimulation; Wahnvorstellungen sind falsche Überzeugungen. Die Symptome der Schizophrenie können positiv (das Vorhandensein unangemessener Verhaltensweisen) oder negativ (das Fehlen angemessener Verhaltensweisen) sein.

 

16.15 – Wie unterscheiden sich chronische und akute Schizophrenie?

Schizophrenie tritt typischerweise im späten Jugendalter auf, betrifft etwas häufiger Männer und kommt in allen Kulturen vor. Bei der chronischen (oder progredienten) Schizophrenie verläuft die Entwicklung schleichend, und eine Heilung ist unwahrscheinlich. Bei der akuten (oder reaktiven) Schizophrenie tritt die Erkrankung plötzlich auf – als Reaktion auf Stress – und die Aussichten auf Heilung sind günstiger.

 

16.16 – Welche Gehirnanomalien stehen in Zusammenhang mit Schizophrenie?

Bei Personen mit Schizophrenie gibt es eine übermäßige Anzahl von Dopaminrezeptoren, die die Signale im Gehirn verstärken und positive Symptome wie Halluzinationen und Paranoia hervorrufen können. Hirnscans haben eine abnorme Aktivität in den Frontallappen, im Thalamus und in der Amygdala sowie einen Verlust der neuronalen Verbindungen im gesamten Gehirnnetzwerk ergeben. Zu den mit Schizophrenie assoziierten Hirnanomalien gehören vergrößerte, mit Flüssigkeit gefüllte Areale und eine entsprechende Verkleinerung und Verdünnung des Hirngewebes.

 

16.17 – Bei welchen pränatalen Ereignissen besteht ein erhöhtes Risiko, eine Schizophrenie zu entwickeln?

Zu den möglichen Faktoren gehören Diabetes der Mutter, ein höheres Alter des Vaters, Virusinfektionen oder Mangelerscheinungen während der Schwangerschaft der Mutter sowie ein niedriges Gewicht oder Sauerstoffmangel bei der Geburt.

 

16.18 – Wie beeinflussen Gene die Schizophrenie? Welche Faktoren könnten frühe Warnsignale für Schizophrenie bei Kindern sein?

Zwillings- und Adoptionsstudien zeigen, dass die Veranlagung zur Schizophrenie erblich ist. Mehrere Gene wirken bei der Entstehung von Schizophrenie zusammen. Es gibt keine umweltbedingten Ursachen, die zwangsläufig zu Schizophrenie führen, aber Umwelteinflüsse (wie pränatale Viren oder mütterlicher Stress) können bei Menschen, die für diese Störung prädisponiert sind, Gene „aktivieren“. Zu den möglichen Frühwarnzeichen gehören sowohl biologische Faktoren (eine Mutter mit schwerer und lang anhaltender Schizophrenie; Geburtskomplikationen; Trennung von den Eltern; kurze Aufmerksamkeitsspanne und schlechte Muskelkoordination) als auch psychologische Faktoren (störendes oder zurückgezogenes Verhalten; emotionale Unberechenbarkeit; schlechte Beziehungen zu Gleichaltrigen und Alleingänge; Trennung von den Eltern; Missbrauch in der Kindheit).

 

16.19 – Was sind dissoziative Störungen, und warum sind sie umstritten?

Dissoziative Störungen sind umstrittene, seltene Erkrankungen, bei denen das Bewusstsein von früheren Erinnerungen, Gedanken und Gefühlen getrennt zu sein scheint. Skeptische Stimmen weisen darauf hin, dass die dissoziative Identitätsstörung (issociative identity disorder; DID) im späten zwanzigsten Jahrhundert dramatisch zugenommen hat, außerhalb Nordamerikas nur selten vorkommt und möglicherweise auf Rollenspiele von Menschen zurückzuführen ist, die auf die Suggestionen von Behandlern ansprechen. Andere sehen die DID als eine Manifestation von Angstgefühlen oder als eine erlernte Reaktion, wenn Verhaltensweisen durch Angstreduktion verstärkt werden.

 

16.20 – Was sind die drei Cluster der Persönlichkeitsstörungen? Welches Verhalten und welche Gehirnaktivität charakterisieren die antisoziale Persönlichkeit?

Persönlichkeitsstörungen sind starre und dauerhafte Verhaltensmuster, die das soziale Funktionieren beeinträchtigen. Die zehn DSM-5-Störungen werden in der Regel in drei Gruppen eingeteilt, die durch (1) Angst, (2) exzentrisches oder seltsames Verhalten und (3) dramatisches oder impulsives Verhalten gekennzeichnet sind. Die antisoziale Persönlichkeitsstörung (eine der Störungen des dritten Clusters) zeichnet sich durch Gewissenlosigkeit und manchmal auch durch aggressives und furchtloses Verhalten aus. Bei Menschen mit dieser Störung ist die Amygdala kleiner und der Frontallappen zeigt eine geringere Aktivität, was zu einer Beeinträchtigung der kognitiven Funktionen des Frontallappens und einer verminderten Reaktionsfähigkeit auf die Probleme anderer führt. Genetische Prädispositionen können in Wechselwirkung mit der Umwelt zu diesen Merkmalen führen.

 

16.21 – Was sind die drei häufigsten Essstörungen, und wie beeinflussen biologische, psychologische und soziokulturelle Faktoren die Anfälligkeit für diese Erkrankungen?

Bei Menschen mit Essstörungen (meist Frauen oder homosexuelle Männer) überwältigen psychologische Faktoren die Neigung des Körpers, ein normales Gewicht zu halten. Menschen mit Anorexia nervosa (in der Regel weibliche Jugendliche) halten trotz erheblichen Untergewichts weiterhin Diät und treiben manchmal übermäßig viel Sport, weil sie sich selbst als zu dick betrachten. Menschen mit Bulimia nervosa (in der Regel Frauen in ihren späten Teenagerjahren und frühen Zwanzigern) haben heimlich Fressattacken und kompensieren diese durch Fasten, Abführen oder exzessive sportliche Betätigung. Menschen mit einer Binge-Eating-Störung haben zwar Essanfälle, aber keine anschließenden Abführmaßnahmen, kein Fasten oder Sport als Gegenmaßnahme. Kultureller Druck, geringes Selbstwertgefühl und negative Emotionen in Verbindung mit stressigen Lebenserfahrungen und genetischen Faktoren führen zu Essstörungen.

 

 

zurück zur Inhaltsübersicht

 

Kapitel 17 - Klinische Psychologie: Therapie

 

17.1 – Wie unterscheiden sich Psychotherapie und die biomedizinische Therapie?

Psychotherapie ist eine Behandlung mit psychologischen Techniken, die aus der Interaktion zwischen einem speziell ausgebildeten Therapeuten bzw. einer speziell geschulten Therapeutin und einer Person besteht, die psychologische Schwierigkeiten überwinden oder persönliches Wachstum erreichen möchte. Die wichtigsten Psychotherapien entstammen den psychodynamischen, humanistischen, verhaltenstherapeutischen und kognitiven Perspektiven der Psychologie. Bei der biomedizinischen Therapie werden psychische Störungen mit Medikamenten oder Verfahren behandelt, die direkt auf die Physiologie des bzw. der Betroffenen einwirken. Ein eklektischer Ansatz kombiniert Techniken aus verschiedenen Therapieformen.

 

17.2 – Welches sind die Ziele und Techniken der Psychoanalyse und wie wurden sie in der psychodynamischen Therapie angepasst?

Mit der Psychoanalyse versuchte Sigmund Freud, Personen Selbsterkenntnis zu vermitteln und sie von ihren Störungen zu befreien, indem er angstbesetzte Gefühle und Gedanken ins Bewusstsein brachte. Zu den psychoanalytischen Techniken gehörten die freie Assoziation und die Interpretation von Widerständen und Übertragungen. Die psychodynamische Therapie wurde von der traditionellen Psychoanalyse beeinflusst, unterscheidet sich jedoch in vielerlei Hinsicht von dieser, u. a. durch das Fehlen des Konzepts von Es, Ich und Über-Ich. Diese moderne Therapie ist kürzer, kostengünstiger und konzentriert sich mehr darauf, Klient:innen zu helfen, aktuell Symptome zu lindern. Psychodynamische Therapeut:innen helfen ihren Klient:innen zu verstehen, wie vergangene Beziehungen Themen hervorgebracht haben, die sich in gegenwärtigen Beziehungen auswirken können.

 

17.3 – Was sind die grundlegenden Themen der humanistischen Therapie? Welches sind die speziellen Ziele und Techniken der klientenzentrierten Therapie nach Carl Rogers?

Sowohl psychodynamische als auch humanistische Therapien sind Einsichtstherapien sie versuchen, das Leben der Menschen zu verbessern, indem sie ihnen ihre Motive und Abwehrmechanismen bewusst machen. Zu den Zielen der humanistischen Therapie gehört es, den Betroffenen zu helfen, ihre Selbstwahrnehmung und Selbstakzeptanz zu verbessern, ihr persönliches Wachstum zu fördern, anstatt Krankheiten zu heilen, ihnen zu helfen, die Verantwortung für ihr eigenes Wachstum zu übernehmen, sich auf bewusste Gedanken statt auf unbewusste Motivationen zu konzentrieren und die Gegenwart und Zukunft als wichtiger zu betrachten als die Vergangenheit. Carl Rogers' klientenzentrierte Therapie schlug vor, dass der wichtigste Beitrag des Therapeuten bzw. der Therapeutin darin besteht, durch aktives Zuhören als psychologischer Spiegel zu fungieren und ein wachstumsförderndes Umfeld mit bedingungsloser positiver Wertschätzung zu schaffen.

 

17.4 – Wie unterscheidet sich die Grundannahme der Verhaltenstherapie von denen der psychodynamischen und humanistischen Therapien? Welche Techniken werden bei den Expositionstherapien und der Aversionskonditionierung genutzt?

Verhaltenstherapien sind keine Einsichtstherapien, sondern gehen davon aus, dass dysfunktionale Verhaltensweisen das Problem sind. Ihr Ziel ist die Anwendung von Lernprinzipien, um diese Problemverhaltensweisen zu verändern. Klassische Konditionierungstechniken, einschließlich Expositionstherapien (z. B. systematische Desensibilisierung oder Virtual-Reality-Expositions-Therapie) und aversive Konditionierung, versuchen, Verhaltensweisen durch Gegenkonditionierung zu verändern, d. h. durch das Hervorrufen neuer Reaktionen auf alte Reize, die unerwünschte Verhaltensweisen auslösen.

 

17.5 – Was ist die Grundannahme der Verhaltensmodifikation, und was sind die Auffassungen derer, die diesen Ansatz befürworten bzw. kritisieren?

Die operante Konditionierung geht von der Annahme aus, dass willkürliche Verhaltensweisen stark von ihren Konsequenzen beeinflusst werden. In der auf operanter Konditionierung basierenden Therapie werden daher Techniken zur Verhaltensänderung eingesetzt, um unerwünschte Verhaltensweisen zu ändern, indem erwünschte Verhaltensweisen positiv verstärkt und unerwünschte Verhaltensweisen ignoriert oder bestraft werden. Kritiker:innen behaupten, dass (1) Techniken, wie sie in der Token-Ökonomie eingesetzt werden, zu Verhaltensänderungen führen können, die wieder verschwinden, wenn die Belohnungen wegfallen, und dass (2) die Entscheidung, welche Verhaltensweisen geändert werden sollen, autoritär und unethisch ist. Befürworter:innen argumentieren, dass eine Behandlung mit positiven Belohnungen humaner ist als die Bestrafung oder Einweisung von Menschen wegen unerwünschter Verhaltensweisen.

 

17.6 – Welches sind die Ziele und Techniken der kognitiven Therapie und der kognitiven Verhaltenstherapie?

Die kognitive Therapie, wie z. B. Aaron Becks kognitive Therapie bei Depressionen, geht davon aus, dass unser Denken unsere Gefühle beeinflusst und dass die Rolle der Behandelnden darin besteht, das selbstschädigende Denken der Klient:innen zu verändern, indem sie die Betroffenen darin schult, Ereignisse auf konstruktivere Weise wahrzunehmen und zu interpretieren. Die weithin erforschte und praktizierte kognitiv-behaviorale Therapie (CBT) kombiniert kognitive Therapie und Verhaltenstherapie, wobei den Klient:innen geholfen wird, ihre neuen Denk- und Verhaltensweisen regelmäßig im Alltag auszuprobieren. Eine neuere CBT-Variante, die dialektische Verhaltenstherapie (DBT), bringt Klient:innen kognitive Taktiken zur Stressbewältigung und Emotionsregulation bei und schult sie in sozialen Fähigkeiten und Achtsamkeitsmeditation. 

 

17.7 – Welches sind die Ziele und die Vorteile der Gruppentherapie und der Familientherapie?

Gruppentherapien können mehr Betroffenen zu geringeren Kosten helfen als Einzeltherapien. Die Klient:innen können davon profitieren, dass sie in einer Gruppensituation ihre Gefühle erkunden und soziale Fähigkeiten entwickeln können, indem sie erfahren, dass andere Menschen ähnliche Probleme haben, und indem sie Feedback zu neuen Verhaltensweisen erhalten. Die Familientherapie betrachtet die Familie als ein wechselseitiges System. Sie hilft den Familienmitgliedern, ihre Rollen zu erkennen und zu lernen, offener und direkter zu kommunizieren.

 

17.8 – Funktioniert Psychotherapie? Wie können wir das herausfinden?

Die positiven Berichte von Betroffenen und Behandlern sind noch kein Beweis für die Wirksamkeit einer Psychotherapie, und der Placebo-Effekt macht es schwierig zu beurteilen, ob eine Verbesserung tatsächlich auf die Behandlung zurückzuführen ist. Mithilfe von Meta-Analysen, die die Ergebnisse von Hunderten von randomisierten Studien zu den Ergebnissen von Psychotherapien statistisch zusammenfassen, haben Forschende herausgefunden, dass sich bei Personen, die sich keiner Behandlung unterziehen, häufig eine Besserung einstellt, während bei Personen, die sich einer Psychotherapie unterziehen, die Wahrscheinlichkeit einer Besserung zunimmt – und zwar in kürzerer Zeit und mit geringerem Risiko eines Rückfalls.

 

17.9 – Sind manche Psychotherapien bei bestimmten Störungen wirksamer als andere?

Es gibt keine bestimmte Psychotherapieform, die generell allen anderen überlegen wäre. Eine Therapie ist am wirksamsten für Menschen mit eindeutigen, spezifischen Problemen. Einige Therapien – wie die Verhaltenskonditionierung zur Behandlung von Phobien und Zwängen – sind bei bestimmten Störungen wirksamer. Kognitive und kognitiv-behaviorale Therapien haben sich bei der Bewältigung von Angstzuständen, posttraumatischen Belastungsstörungen, Schlaflosigkeit und Depressionen als wirksam erwiesen; Verhaltenskonditionierungstherapien bei spezifischen Verhaltensproblemen; psychodynamische Therapie bei Depressionen und Angstzuständen; und nicht-direktive (klientenzentrierte) Beratung bei leichten bis mittelschweren Depressionen. In der evidenzbasierten Praxis werden die besten vorhandenen Forschungsergebnisse mit dem Fachwissen der Kliniker:innen und den Eigenschaften und Präferenzen der Betroffenen kombiniert.

 

17.10 – Wie schneiden alternative Therapien unter dem prüfenden Blick der Wissenschaft ab?

Krankhafte Zustände neigen dazu, sich von selbst zu normalisieren, und der Placebo-Effekt kann den Eindruck erwecken, eine Behandlung sei wirksam gewesen. Diese beiden Tendenzen erschweren die Bewertung nicht-traditioneller Therapien, die behaupten, bestimmte Beschwerden zu heilen. Eye Movement Desensitization and Reprocessing (EMDR) hat sich als durchaus wirksam erwiesen – allerdings nicht aufgrund der Augenbewegungen, sondern aufgrund des expositionstherapeutischen Charakters der Behandlung. Die Lichtexpositionstherapie scheint Menschen mit jahreszeitlich bedingten Depressionssymptomen zu helfen, indem sie die Hirnregion aktiviert, die Erregung und Hormone beeinflusst.

 

17.11 – Welche drei Merkmale haben alle Formen der Psychotherapie gemein?

Alle Psychotherapieverfahren bieten entmutigten Menschen neue Hoffnung, eine neue Perspektive und (wenn der bzw. die Therapeut:in effektiv ist) eine einfühlsame, vertrauensvolle und fürsorgliche Beziehung. Das Vertrauen und die emotionale Beziehung zwischen Therapeut:in und Klient:in – das therapeutische Bündnis – bilden ein wichtiges Element einer wirksamen Therapie.

 

17.12 – Wie beeinflussen Kultur und Wertvorstellungen die Beziehung zwischen Therapeut:in und Klient:in?

Die Wertvorstellungen der Therapeut:innen, die sich auf Therapieziele und die Auffassung von Fortschritt auswirken, sind unterschiedlich. Diese Unterschiede können zu Problemen führen, wenn sich Therapeut:in und Klient:in in ihren kulturellen oder religiösen Ansichten unterscheiden.

 

17.13 – Auf was sollte eine Person achten, die eine:n Therapeut:in auswählen will?

Gesundheitszentren sind in der Regel eine gute Anlaufstelle für Beratungsangebote, und sie können einige kostenlose Dienste anbieten. Eine Person, die eine Therapie sucht, sollte sich nach dem Behandlungsansatz, den Werten, den Referenzen und den Gebühren des Therapierenden erkundigen. Ein wichtiger Aspekt ist, ob sich der bzw. die Therapiesuchende wohl fühlt und eine Bindung zum Therapeuten bzw. zur Therapeutin aufbauen kann. Die American Psychological Association hat die Bedeutung eines starken therapeutischen Bündnisses erkannt und akkreditiert Programme, die eine Ausbildung in kultureller Sensibilität anbieten und unterrepräsentierte kulturelle Gruppen einbeziehen.

 

17.14 – Warum wird eine therapeutische Lebensstiländerung als effektive biomedizinische Therapie angesehen und wie funktioniert sie?

Die therapeutische Lebensstiländerung wird als biomedizinische Therapie betrachtet, weil sie die Art und Weise beeinflusst, wie das Gehirn reagiert. Geist und Körper sind eine Einheit; beeinflusst man das eine, beeinflusst man auch das andere. Bewegung, Ernährung, Beziehungen, Erholung, Entspannung und religiöses oder spirituelles Engagement wirken sich auf die psychische Gesundheit aus. Menschen, die sich einem Programm aus aerobem Training, ausreichendem Schlaf, Lichtexposition, sozialem Engagement, Reduzierung des Grübelns und besserer Ernährung unterziehen, konnten ihre depressiven Symptome lindern.

 

17.15 – Was sind medikamentöse Therapien? Wie helfen Doppelblindstudien Forschenden dabei, die Wirksamkeit eines Medikaments zu beurteilen?

Die Psychopharmakologie hat dazu beigetragen, dass die medikamentöse Therapie die am häufigsten eingesetzte biomedizinische Therapie ist. Antipsychotika werden zur Behandlung von Schizophrenie eingesetzt; einige blockieren die Dopaminaktivität. Zu den Nebenwirkungen können Spätdyskinesien (unwillkürliche Zuckungen von Gesichtsmuskeln, Zunge und Gliedmaßen) oder ein erhöhtes Risiko für Adipositas und Diabetes gehören. Angstlösende Medikamente, die die Aktivität des zentralen Nervensystems dämpfen, werden zur Behandlung von Angststörungen, Zwangsneurosen und posttraumatischen Belastungsstörungen eingesetzt und können süchtig machen. Antidepressiva, die häufig die Verfügbarkeit von Serotonin und Noradrenalin erhöhen, werden zur Behandlung von Depressionen, Angststörungen, Zwangsstörungen und posttraumatischen Belastungsstörungen mit moderater Wirksamkeit eingesetzt. Angesichts ihrer breit gefächerten Anwendung (von Depressionen über Angstzustände bis hin zu Schlaganfällen) bevorzugen einige Fachleute die Bezeichnung SSRI (selektive Serotonin-Wiederaufnahmehemmer) anstelle von Antidepressiva. Lithium und Valproinsäure sind Stimmungsstabilisatoren, die bei bipolaren Störungen verschrieben werden. In Studien kann ein Doppelblindverfahren eingesetzt werden, um den Placebo-Effekt und die Voreingenommenheit von Untersuchenden zu vermeiden.

 

17.16 – Wie werden die Stimulation des Gehirns und die Psychochirurgie bei der Behandlung bestimmter Störungen genutzt?

Die Elektrokrampftherapie (EKT), bei der ein kurzer elektrischer Strom durch das Gehirn eines narkotisierten Menschen geschickt wird, ist ein wirksames letztes Mittel zur Behandlung von Personen mit schweren Depressionen, die auf andere Therapien nicht angesprochen haben. Zu den neueren alternativen Behandlungsmethoden für Depressionen gehören die transkranielle Elektrostimulation (tDCS; wird auch für wissenschaftlich nicht nachgewiesene kognitive Vorteile eingesetzt), die repetitive transkranielle Magnetstimulation (rTMS) und die Tiefenhirnstimulation (deep-brain stimulation; DBS; soll durch Beruhigung einer überaktiven Hirnregion wirken, die mit negativen Emotionen in Verbindung gebracht wird, und hat bei einigen Patienten Vorteile gezeigt). Bei der Psychochirurgie wird Hirngewebe entfernt oder zerstört, in der Hoffnung, das Verhalten zu ändern. Radikale psychochirurgische Verfahren wie die Lobotomie werden nicht mehr durchgeführt. Die heutige mikrochirurgische Psychochirurgie und die MRT-gesteuerte Präzisionshirnchirurgie sind seltene Behandlungen, die nur noch als letztes Mittel in Frage kommen, da die Auswirkungen irreversibel sind.

 

17.17 – Was ist der Grundgedanke von Präventionsprogrammen für psychische Gesundheit und warum ist es wichtig, Resilienz aufzubauen?

Präventionsprogramme für die psychische Gesundheit basieren auf der Idee, dass viele psychische Störungen verhindert werden können, wenn repressive, das Selbstwertgefühl zerstörende Umgebungen in wohlwollende, förderliche Umgebungen geändert werden, die das Wachstum, das Selbstvertrauen und die Widerstandsfähigkeit begünstigen. Die Bewältigung von Herausforderungen kann zu posttraumatischem Wachstum führen. Gemeindepsycholog:innen arbeiten daran, psychischen Störungen vorzubeugen, indem sie destruktive Umgebungen in günstigere Umgebungen verwandeln, die Kompetenz, Gesundheit und Wohlbefinden fördern.

 

 

zurück zur Inhaltsübersicht

 

Kapitel 18 - Pädagogische Psychologie

[die Antworten zu Kapitel 18 folgen]

18.1 – Wie gelingt Erziehung? Wie können aus Kindern und Jugendliche verantwortliche Erwachsene werden, die sich an sozialen und leistungsbezogenen Standards orientieren?

18.2 – Wie kann man Fehlverhalten modifizieren und angemessenes Verhalten wahrscheinlicher machen?

18.3 – Wie kann man mehr Disziplin und eine gute Lernatmosphäre im Klassenzimmer fördern?

18.4 – Wie kann man Mobbing entgegenwirken?

18.5 – Welche Aussagen machen Atkinson und Weiner zu den motivationsrelevanten Emotionen Stolz und Scham?

18.6 – Was bestimmt laut Dweck, ob Personen motiviert sind?

18.7 – Depression im Kindesalter: Was sind die zentralen Merkmale?

18.8 – Maßnahmen bei Depression?

 

zurück zur Inhaltsübersicht

 

Anhang A

 

A.1 – Was bedeutet Flow?

Flow ist ein vollständig konzentrierter Bewusstseinszustand, in dem das Bewusstsein für sich selbst und die Zeit reduziert ist. Er entsteht durch den uneingeschränkten Einsatz der eigenen Fähigkeiten. Interessen sagen sowohl Leistung als auch Ausdauer voraus, daher sollten Menschen Berufe finden, die gut zur Person und zum Umfeld passen.

 

A.2 – Welches sind die drei wichtigsten Studienbereiche im Zusammenhang mit der Arbeits- und Organisationspsychologie?

Die drei wichtigsten Studienbereiche der Arbeits- und Organisationspsychologie (AO) sind die Personal-, Organisations- und Human-Factors-Psychologie. Jeder dieser Bereiche nutzt psychologische Erkenntnisse, um das breite Spektrum der heutigen Arbeitskräfte, Arbeitsplätze und Arbeitstätigkeiten zu untersuchen und zu nutzen.

 

A.3 – Wie unterstützen Personalpsycholog:innen die Arbeitssuche, Personalauswahl, Arbeitsvermittlung und Leistungsbeurteilung?

Personalpsycholog:innen bieten Schulungsprogramme für Arbeitssuchende an, entwickeln Auswahlmethoden für neue Beschäftigte, rekrutieren und bewerten Bewerber:innen, konzipieren und bewerten Schulungsprogramme, ermitteln die Interessen und Stärken von Menschen, analysieren Arbeitsinhalte und bewerten die individuelle und organisatorische Leistung. Unstrukturierte, subjektive Interviews fördern die Illusion des Interviewenden; strukturierte Interviews zeigen arbeitsrelevante Stärken auf und sind bessere Prädiktoren für die Leistung. Checklisten, grafische Bewertungsskalen und Verhaltensbewertungsskalen sind nützliche Methoden zur Leistungsbeurteilung.

 

A.4 – Was ist die Aufgabe von Organisationspsycholog: innen?

Organisationspsycholog:innen untersuchen die Einflüsse auf die Zufriedenheit und Produktivität der Beschäftigten und erleichtern den organisatorischen Wandel. Mitarbeiterzufriedenheit und -engagement korrelieren in der Regel mit dem Unternehmenserfolg; tatsächlich sagt die Einstellung der Beschäftigten den künftigen Unternehmenserfolg voraus.

 

A.5 – Wie können Führungskräfte am effektivsten sein?

Effektive Führungskräfte setzen sich konkrete, anspruchsvolle Ziele und wählen einen adäquaten Führungsstil. Der Führungsstil kann zielorientiert (Aufgabenführung), gruppenorientiert (soziale Führung) oder eine Kombination aus beidem sein. Wirksames Management beinhaltet oft positive Verstärkung, die Erfüllung des Bedürfnisses nach Zugehörigkeit und partizipatives Management.

 

A.6 – Welche kulturellen Einflüsse müssen bei der Wahl eines effektiven Führungsstils berücksichtigt werden?

Das Projekt GLOBE untersucht kulturelle Unterschiede bei den Führungserwartungen. Führungskräfte, die die Erwartungen erfüllen (in einigen Kulturen eher direktiv, in anderen eher partizipativ), sind in der Regel erfolgreich. Erfolgreiche Unternehmen auf der ganzen Welt konzentrieren sich jedoch darauf, die Stärken ihrer Beschäftigten zu erkennen und zu fördern; eine auf Stärken basierende Führung zahlt sich überall aus.

 

A.7 – Wie arbeiten Human-Factors-Psycholog:innen, um Maschinen und Arbeitsumgebungen benutzerfreundlich zu gestalten?

Human-Factors-Psycholog:innen tragen zur menschlichen Sicherheit und zur Verbesserung des Konzepts bei, indem sie Entwickler:innen und Gestalter:innen ermutigen, menschliche Fähigkeiten und Verhaltensweisen zu berücksichtigen, ihre Arbeit vor der Produktion und Verbreitung zu testen und Flüchtigkeitsfehler im Auge zu behalten.

 

 

zurück zur Inhaltsübersicht

 

 

 

Zurück zur Übersicht